Are human influences on the climate really small?

by Steven E. Koonin

My Wall Street Journal article of 9/20/14, Climate Science is Not Settled , contains the paragraph

Even though human influences could have serious consequences for the climate, they are physically small in relation to the climate system as a whole. For example, human additions to carbon dioxide in the atmosphere by the middle of the 21st century are expected to directly shift the atmosphere’s natural greenhouse effect by only 1% to 2%. Since the climate system is highly variable on its own, that smallness sets a very high bar for confidently projecting the consequences of human influences.

 

Unfortunately, the necessary brevity and non-technical nature of a newspaper article has created some confusion about what I meant and how I arrived at the 1-2% figure. I write here to explain something that’s quite simple, but has profound implications.

AR5 WG1 Figure 2.11 shows the global radiative balance, with the total downward flux on the Earth’s surface estimated as 503 ± 7 W/m2 (161 W/m2 solar + 342 W/m2 thermal aka “Greenhouse effect”).

Slide1

AR5 Figure SPM.4 shows the total anthropogenic direct perturbation (i.e., absent feedbacks) of this balance (the “forcing”)to be some 2.3 ± 1 W/m2, less than 1% of the downward thermal flux or less than 0.5% of the total downward flux. If the atmospheric concentration of CO2 were to rise to 550 ppm with all other anthropogenic effects unchanged, this perturbation would rise to be 3.9 W/m2.  However one quantifies it, we have a percentish exogenous perturbation, which will result in percentish shifts of the energy flows, even after feedbacks (which are thought to roughly double the response).

Slide2

An alternative way of seeing the physical smallness of anthropogenic influences is to look at how the long-wave absorptivity of the clear sky increases with CO2 concentration – this is the physical input to GCMs that varies directly. Figure 4 from Harde shows that a doubling from the pre-industrial 280 ppm to 560 ppm increases the absorptivity by about 1% on a base of 82%, or, again a percentish shift. An even simpler indication of the percentish influence is to note that a 3 C mean global surface temperature increase on a base of 288 K is also about a 1% effect.

Slide3

The physical smallness of anthropogenic influences, which comes as a surprise to many non-climate-expert scientists, has profound implications for climate understanding and modeling.   First, it means precision observations are required to see the climate response. Second, it means that natural variations can easily overwhelm human influences, at least on multidecadal scales (witness the current stasis in global mean surface temperature). And finally, because life at the 1% level is rich, the models have to get many small phenomena right to confidently isolate and project the response to anthropogenic effects. Indeed, if the anthropogenic perturbation weren’t small, the detection/attribution discussion would be much more convincing than it is (see, for example, the APS Workshop transcript). Of course, since CO2 is an enduring perturbation to the climate system, at least on the scale of centuries, its effects may eventually grow large enough to clarify the situation.

The annotated version of my WSJ article contains supporting material for some of other points I made. Virtually all of the references are from IPCC AR5 WGI, so they are surely settled science.

JC note:  This post arose from my email discussions with Steve Koonin, and issues raised in discussion on the previous thread about Koonin’s WSJ op-ed.  As with all guest posts, keep your comments relevant and civil.

 

 

 

661 responses to “Are human influences on the climate really small?

  1. As far as I can tell we don’tr even know if the human influences will be net beneficial or net detrimental. I’ve asked many times and no one has been able to answer whether the human influences are increasing or decreasing the risk (consequence and/or probability) of the next abrupt climate change.

    I tend to think we are reducing the risk of the next abrupt cooling event and, since this would be far more damaging that an abrupt warming event, we are actually, inadvertently, reducing rather than increasing the risk of climate change.

    • The cooling and warming events happen in a cycle.

      A warming event, then a cooling event, then a warming event, then a cooling event, then a warming event, then a cooling event, then a warming event, then a cooling event, then a warming event, then a cooling event, then a warming event.

      These natural events will continue to cycle, no matter what we do. The warm events are much nicer for our life on earth. The cold events are harsher for our life on earth. Both will repeat, over and over. If we do anything to change the heat balance, it may change these natural warm and cool event timing some, but not a lot and it will not stop the natural cycles. These cycles have been in place for millions of years.

      We cannot change the risk of future climate change. Climate change works 100% of the time. The chance of climate change not happening is 0.00%
      We cannot change that.

      Will human influences will be net beneficial or net detrimental. It will not matter much. We most likely cannot measure data to prove either way.

      Up to now, in the history of the world, human influences have been more beneficial, more of the time. Humans have multiplied and lived well.

      Human influences were detrimental in many times. Often, beneficial to some while detrimental to others.

      We are not reducing or increasing the risk of the next climate change. We are just watching to see WHEN it happens, it will happen, and we are taking more data to understand it better.

    • It seems to me we’ve spent 30 years arguing about temperatures, forcings, climate sensitivity, etc, and negligible effort on reducing the uncertainty of the damage function. Is the damage function simply an assumption based on fear-mongering or is there any genuine hard science behind it?

      In fact is the social cost of carbon positive or negative? Reading the analyses by Stern, Nordhaus, EPA give little confidence that the SCC is based on anything more than belief:
      https://www.whitehouse.gov/sites/default/files/omb/assets/inforeg/technical-update-social-cost-of-carbon-for-regulator-impact-analysis.pdf
      http://www.economics-ejournal.org/special-areas/special-issues/the-social-cost-of-carbon
      http://www.gao.gov/assets/670/665016.pdf

      • Peter: The long term increase in sea level worries me. But other than that the forthcoming crisis caused by resource depletion, including oil and gas, seems to be a much bigger problem. The proposed effort to control the climate will fail, in part because it will use resources we can use to secure sufficient energy for a growing population, and in part because it makes climate change into a political hot potato. I sure wish we could focus on energy and other resources, respect for human rights, and other serious issues.

      • Fernando Leanme,

        Thank you. I agree with you on some points but don’t share some of your concerns.

        I strongly agree with you on this:

        I sure wish we could focus on energy …, respect for human rights, and other serious issues.

        Regarding “other resources” I share your concerns about biosphere resources but I am not concerned about minerals or fossil fuels. Market will handle that, as they always have. It’s best for governments, bureaucrats, NGO’s and other ignoramuses to keep their noses out of it and allow the markets to respond to resource constraints. They are by far the best informed.

        I am not concerned about sea level rise. It has a trivial economic impact and that is what counts. Economic impact is the way to measure impact. A 1 m sea level rise by 2100 is estimate to reduce global GDP by $1 trillion in a cumulative $30,000 trillion global GDP to 2100. That’s trivial. A 0.5 m sea level rise would reduce global GDP by just $200 billion – that’s even trivialer :)

        I strongly agree with you on this:

        The proposed effort to control the climate will fail, in part because it will use resources we can use to secure sufficient energy for a growing population, and in part because it makes climate change into a political hot potato.

        But my main reason is that the policies proposed by the alarmists are economically irrational. Therefore they cannot survive. I encourage people to do this mental exercise: consider that if the whole world could vote, what percentage would vote to have energy prices increased for some promised intangible gain in “reduced climate damages” in 100 years from now. I expect not 1% (70 million) people would vote for it. If John Cook understood this he might write a paper saying that 99% of the world population does not believe climate change is worth wasting money on. :)

        To emphasise this point, William Nordhaus’s DICE model shows that the world would be significantly poorer for all this century and beyond if it implemented optimal carbon pricing policies. Such policies would be net negative for all this century and beyond. Who’s going to support that once they realise? Furthermore, that is from the default values in DICE-2103R (the latest version). And the default values are CAGW values, e.g.:
        • ECS= 3.3
        • RCP = 8.5
        • Highly optimistic assumptions about the participation rate
        • Damage function is at the high end and then increased by another 25% to get the alarmists off his back
        The red curve in the chart below is ½ the assumed Copenhagen participation rate. It is still highly optimistic, IMO. Note that it is net negative GDP for all this century.
        http://catallaxyfiles.com/files/2014/10/Lang-3.jpg

      • We should not worry about sea level rise. They believe it is rising and they adjust the data to match what the models and theory tell them. Leap second data shows that the earth is spinning faster than it was 40 years ago. If sea level rose in the past 40 years, the earth would be spinning slower with more water around the equator.
        The earth does move and make it look like sea level is rising in some places and sinking in other places. Overall, it is now going down. More snow is falling on land and staying there because that is always what happens when polar oceans thaw and turn on the snow monster. This puts ice closer to the spin axis and the as the skater spins faster with arms in, the earth spins faster with more ice in and less water out.

      • What else changes spin rate? You mention the earth moving around. Obviously, well apparently, the water can move around more rapidly than the earth. Can the effects of these two movements be distinguished?
        =========

      • What else changes spin rate? Can the effects of these two movements be distinguished?

        less ice on land and more water in oceans does slow down spin rate.
        more ice on land and less water in oceans does speed up spin rate.

        Spin rate is speeding up. Leap second data is the most accurate measurement in any data that relates to sea level.

        There are larger errors in sea level measurements, of all kinds, than there are in leap second data.

        They quote sea level changes in millimeters per year.
        Their measurements are not good in feet per year.

        They do a lot of questionable “adjustments” to get a result that always increases by a similar but increasing amount.

        The work the data toward the output from the models.
        That don’t work for sea level and it don’t work for Temperature.

    • “I tend to think we are reducing the risk of the next abrupt cooling event..”

      Abrupt cooling can only be regional, and it happens at the same rate irrespective of small variations in global mean temperature.

      • Abrupt cooling can only be regional,

        Source please. Are you saying that abrupt climate change is all BS?

        Tell NAS: http://www.nap.edu/openbook.php?record_id=10136

        Tell Broeker: http://www.slc.ca.gov/division_pages/DEPM/Reports/BHP_Port/ERRATA_CSLC/Vol%20II/EDC%20Attachments%20Vol%20II-02.pdf

      • “Are you saying that abrupt climate change is all BS?”

        As I said regional, as your links describe, even if there follow on global effects they not all cooling, in glacial conditions near permanent El Nino states exist, and desert expansion is massive. And the most abrupt events are Arctic warming events. The point is whether we are mitigating potential cooling or not. It simply comes down to whether increased GHG’s can keep the North Atlantic and Arctic Oscillations forced positive. They have done a lousy job of that since 1995, and we have seen extreme negative NAO/AO values in recent years, not seen since previous solar (grand) minima, with regional temperatures to match.

      • Abrupt cooling can only be regional, [ … ]

        A completely unjustified assertion directly contradicted by climate history. There are many recorded instances of abrupt global cooling.
        The onsets of ice ages were in general both global and abrupt, as was for example the younger Dryas event.

        [ … ] and it happens at the same rate irrespective of small variations in global mean temperature.

        Another completely unjustified assertion. We do not fully understand the triggers for ice ages and events like the younger Dryas, so there is no basis for you to make such ridiculously confident assertions about what influence on their rate (likelihood?) of onset a small variation in global mean temperature might have.

      • “The onsets of ice ages were in general both global and abrupt,”

        Can you show me how abrupt in the tropics?

        “..so there is no basis for you to make such ridiculously confident assertions about what influence on their rate (likelihood?) of onset a small variation in global mean temperature might have.”

        Yes you can by seeing if global warming reduces extreme negative NAO/AO episodes and negative NAO/AO trends.

      • Ice ages tend to start slowly and end abruptly, rather like DO events.

      • Or rather the warming of DO events are rapid, and the cooling tends to be slower.

      • Ian H wrote:
        The onsets of ice ages were in general both global and abrupt, as was for example the younger Dryas event.

        We do not fully understand the triggers for ice ages and events like the younger Dryas.

        Ewing and Donn did explain that, more than a half century ago.
        The consensus alarmists do not understand, but I can explain it.

      • The link to my Pope’s Climate Theory was the wrong one.
        I do hope this fixes it

      • For the past ten thousand years, abrupt climate change has been regional, with the average in the Tropics and in the North and in the South well bounded in the same narrow bounds.

  2. daveandrews723

    So, there could be “serious consequences” but they will be “small.” As a layperson I have to shake my head at science speak. It sounds like political speak to me.

    • He’s not describing the consequences as “small”, rather the relative impact of human influences on various physical properties. Obviously, relatively small influences can have large consequences on the climate as we experience it.

      As I read it, the point of the piece is not to say anything in particular about consequences, but rather about the difficulties of getting precise measures of relatively small human influences on physical properties, and the uncertainties these difficulties entail.

      • daveandrews723

        “Even though human influences could have serious consequences for the climate, they are physically small in relation to the climate system as a whole.” How do you read that any other way?

      • daveandrews723:

        As I read it, “they” refers to “human influences”, not “serious consequences”.

        Say you believed “human influences” were going to increase avg surface temps by 10 deg C. That’s less than 5% of the current ~288K avg, so arguably “small”, mathematically. But surely with “serious consequences” for the climate as we experience it.

      • daveandrews: “Even though human influences could have serious consequences for the climate, they are physically small in relation to the climate system as a whole.” How do you read that any other way?

        Why is that hard to understand? The human influences have effects on climate that are small relative to the natural variation and small relative to our ability to measure changes. In fact, they are smaller than the combined measurement and model error, and models are biased on the high side. There “could be” serious consequences, no consequences, beneficial consequences, or a mixture of good, no, and bad consequences in different regions.

      • Danny Thomas

        MatthewRMarler,
        This:”There “could be” serious consequences, no consequences, beneficial consequences, or a mixture of good, no, and bad consequences in different regions.” Sounds an awful lot like you just described yesterday, today, and tomorrow.

    • Why don’t you try to line your misrepresentation of the actual quote with the actual quote. See if it makes sense. then. If it doesn’t, you might consider spending your time on a food blog.

      “So, there could be “serious consequences” but they will be “small.””

      “Even though human influences could have serious consequences for the climate, they are physically small in relation to the climate system as a whole.”

      Think hard.

      • > Think hard.

        And think “small”:

        That’s not science, but it’s important.

        http://www.aps.org/policy/statements/upload/climate-seminar-transcript.pdf

      • I agree with you Don. The entire article could have been contained in one sentence. “A 3K shift on a base of 288K is only just over one per cent.” This is not the most significant statement ever uttered in the climate debate. All those graphs and sciencey language are just there to hide the fact that the article doesn’t really say anything.

      • The entire article could have been contained in one sentence. “A 3K shift on a base of 288K is only just over one per cent.”

        That the change is 1% of the baseline is irrelevant. The point is that although the change “might be” large enough to cause harmful biological changes, it is small relative to our ability to measure it and small relative to the natural variations. On the absolute scale, a fever of 105F is a similar percentage increase, but it is potentially harmful to the one experiencing it; significantly, we can measure the change accurately enough, unlike the case with climate change.

      • John Carpenter

        Yes, 3K increase may be approx 1% of 288K, however the habitable temperature range is more like 255 K to 311 K, with a range of approx 56 K which makes the percentage more like 5%. Still small, but more accurate to life on the planet

  3. Koonan the ruin
    Of coward complacency
    At the APS.
    ========

    • Eep, Koonin, abject apologies.
      =========

      • michael hart

        lol. Koonan the Librarian?
        —————

        It’s a nice clear, simple, article.

      • Koonin, what is best in life? Crush the certainties. See them driven before you. Hear the lamentations of the alarmists.
        =========================

    • Kim, lol I wondered where you were going with that ;-)

      • Kim is really showing her versatility with that one ; ) I doubt one woman out of a hundred would know that quote.

  4. What the federal judge said today to DOJ lawyers about their role in helping to push the Executive Branch’s overreach on immigration could easily be said about the pushers of the government-funded global warming agenda:

    ““Whether by ignorance, omission, purposeful misdirection, or because they were misled by their clients, the attorneys for the government misrepresented the facts… Fabrications, misstatements, half-truths, artful omissions and the failure to correct misstatements may be acceptable, albeit lamentable, in other aspects of life… is unacceptable.

    • Tribe’s rogue pupil is violating both law and nature.
      ======

    • That’s a bullseye. The parallel is unmistakable.

    • > … the federal judge

      For non-US readers, *please, please* specify WHICH exact court you are referring to. Trying to make sense of the Mann/Steyn imbroglio proved impossible – way too many courts,jurisdictions and general legal cross-purpose volleyballs. Same thing with a recent Canadian Weaver/??? case, with comments mixing what actually happened with what should/could/might have happened if someone or other had only been someone else or other

      I understand that Peabody has lobbed a case against the EPA ruling on coal-fired power stations, but in which court and what court trajectories may follow decisions either way, it is impossible for non-US citizens to grasp

      My view is that these legal proceedings will become critical (and no, I do not think the legal system in general is competent here but whether it is or not is irrelevant); the integrity of our power grids is at risk

      • “A radiant fool setting off a skyrocket by day.”

        H/t another merry round of Thurber’s Carnival, p. 91.
        ==================

      • That judge wants the origin of the deception unveiled even if only in camera to himself. The case is appealed already to another court, soon to render a decision. I’m nearly as confused as the aliens(grin) on the domestic legal details.
        =====================

      • @ kim

        > … I’m nearly as confused as the aliens(grin) on the domestic legal details

        I don’t find that encouraging. not at all. But then, I didn’t expect to :) :)

        Adding legal concrete pours (which may be interrupted at any time, for reasons only dimly perceived by us great unwashed) to the mix only reinforces my view that public human discourse resembles nothing better than a rookery in full throat

    • If you could supply a link for the quoted language, it would be useful to me.

      JD

      • It’s gonna be a blockbuster, and though the EPA may not have yet been illegally premature, there are marvelous parallels in the two cases.

        It’s executive overreach, trending into criminality. Judges are increasingly alarmed at this administration’s behaviour.
        ======================

      • Sorry, first part is page 6, second part is page 10.

      • KCH, Thanks a lot for the link. Looks like the government attorneys are in a heap of trouble for telling the court that they would hold proposed deferrals in abeyance and then granting 100,000 deferrals. Personally, I think the government attorneys, unless a real good explanation is forthcoming, should be reported to the legal ethical authority in the State or States where they are licensed.

        JD

      • @ JD

        > … should be reported to the legal ethical authority in the State or States where they are licensed

        Who does the reporting, and who decides if a report should be made, please ? And, of course, who decides on action (if any) to be taken on said report, if it is made ? And is such action as may be decided appealable ?

        This is what I mean by a legal concrete pour, interruptible for reasons we out here in baa-baa land can never discern in advance, not advancing knowledge

      • The judge wants the details of how the decision to deceive was made. I suspect he’ll follow this to the Gates of Hell.
        ==================

  5. It’s my understanding that solar variation results in only a 1% change and yet it produces significant change. Can they be compared?

    • When it comes to all climate change throughout the geophysical record of the Earth — Nominally, It’s the Sun, Stupid.

      A study of the Earth’s albedo (project “Earthshine”) shows that the amount of reflected sunlight does not vary with increases in greenhouse gases. The “Earthshine” data shows that the Earth’s albedo fell up to 1997 and rose after 2001.

      https://evilincandescentbulb.files.wordpress.com/2012/01/leonardo-davinci-crescent-moon.jpg?w=600

      What was learned is that climate change is related to albedo, as a result of the change in the amount of energy from the sun that is absorbed by the Earth. For example, fewer clouds means less reflectivity which results in a warmer Earth. And, this happened through about 1998. Conversely, more clouds means greater reflectivity which results in a cooler Earth. And this happened after 1998.

      https://evilincandescentbulb.files.wordpress.com/2012/01/lvinc-ba1.jpg?w=640

      It is logical to presume that changes in Earth’s albedo are due to increases and decreases in low cloud cover, which in turn is related to the climate change that we have observed during the 20th Century, including the present global cooling. However, we see that climate variability over the same period is not related to changes in atmospheric greenhouse gases.

      Obviously, the amount of `climate forcing’ that may be due to changes in atmospheric greenhouse gases is either overstated or countervailing forces are at work that GCMs simply ignore. GCMs fail to account for changes in the Earth’s albedo. Accordingly, GCMs do not account for the effect that the Earth’s albedo has on the amount of solar energy that is absorbed by the Earth.

      • Wagathon, thanks for the detailed response.

      • Very good and clear comment. Thanks.

      • [..]
        including the present global cooling.

        What present cooling? Year 2014 was probably one of the least cool years for a long time.

        I’m astonished.

      • “We are already in a cooling trend, which I think will continue for the next 15 years at least. There is no doubt the warming of the 1980s and 1990s has stopped.” ~Professor Anastasios Tsonis, University of Wisconsin

      • Ah, ‘expert evaluation’. Sad it is not that much seeable in gistemp.

      • Wagathon – Oceans are warming.
        Here is the link to the actual story the Examiner writer misinterpreted so clearly. It’s a good read.

        http://earthobservatory.nasa.gov/Features/OceanCooling/

        Also, NOAA tracks ocean heat content, which is rising and rising. Slides one and two on this page show the heat increase clearly.
        http://www.nodc.noaa.gov/OC5/3M_HEAT_CONTENT/

        The chart you linked (also from the Examiner) shows just sea surface temperature anomaly in the small area of the Pacific used to define El Niño/La Niña conditions – it is NOT “the ocean” and it is only the surface.

      • Read the comments found in the link to the Examiner article above and you will see that the workings of the ‘social cascade’ that has led you to believe that the oceans are not really cooling (because the heat is hiding deep in the ocean, although it’s too slight to be measurable). Meanwhile, we see record sea ice extent. When the oceans are cooling there is no global warming and that is what we seen even in the land-based record that is corrupted by the UHI effect: no global warming going on 2 decades (going on 3 decades depending on which dataset you wish to use).

      • No, Wagathon – comment threads on a newspaper article don’t disprove anything, especially when the article distorts what it reports on.

        Here’s the second paragraph of the article the newspaper writer so distorted:
        “The oceans are absorbing more than 80 percent of the heat from global warming,” he says. “If you aren’t measuring heat content in the upper ocean, you aren’t measuring global warming.”

        The title of the NASA article, by the way, is “Correcting Ocean Cooling”.

        And the measured ocean heat content is increasing. That’s why I posted a reference to one of the places that measures such things, not to some weatherman’s column in some newspaper.

        And what, specifically, is Dr. Tsonis referring to? Do you have citations to anything peer-reviewed that he’s published on the topic?

        Because it hasn’t been cooling. It’s been warming slower.

      • If you remember, Trenberth told us the heat is missing — we can’t find it — but, despite the many uncertainties, as Dr. Curry reminds us, “there is no particularly convincing evidence that the ‘missing heat’ is hiding in the ocean.” All we have is the changing story of the social cascade that has a vest interest in maintain the illusion of global warming certainty.

      • Well, Wagathon, if taking sentences out of context floats your boat, happy boating. I was just providing a clarification to the misleading article you linked, the chart you misidentified as “the ocean”, and followed up with measurements.

        “Social cascade”? What’s that, a detergent? No, evidence of warming oceans points to warming oceans. It’s an “evidence cascade” if you must.

      • The ‘cascade’ is a social mechanism recognized by psychologists that serve to perpetuate perceptions that when expressed over and over, “trigger chains of individual responses,” as Kuran et al., say, “that make these perceptions appear increasingly plausible through their rising availability in public discourse. Availability cascades may be accompanied by counter-mechanisms that keep perceptions consistent with the relevant facts… The resulting mass delusions may last indefinitely, and they may produce wasteful or even detrimental laws and policies” (see, Kuran T and Sunstein CR (1999). Availability cascades and risk regulation. Stanford Law Review, 51(4): 683-768).

        Mann’s fraudulent ‘hockey stick’ science and the continuing parroting of the nonsensical 97% consensus meme are examples of such social cascades. Believing the oceans are warming when there has been no global warming for going on up to 3 decades, depending on the data source used, is another cascade. Ask any Bostonian.

      • Thanks for the detail. The psychologists also are aware of people who repeatedly ignore all of the measurements in order to hold to an alternate view of the world. When shown measurements of oceans warming, ignore. When asked for documentations to support Dr. Tsonis’ claim – revert to a cascade of Michal Mann fixation. Carry on.

      • A Michael Mann fixation sounds serious. Is that perhaps something Al Gore may have?

        We as a society have a problem. We must address the Climate Change Paradox. If the predictors of global warming are correct, then they have solved the simulation problem –i.e., the computer models they possess must now be clear and accurate reproductions of nature. However, to believe there are so many — but, only in Western academia — who have successfully digitized faithful copies of nature is, irrational! Alan Turing may have invented the machine capable of deciphering a Nazi machine but none of us humans – not even Western school teachers – have yet to decode the secrets of Mother Nature. The enigma of climate change remains.

        Solar variations and volcanic eruptions are a wild card. Russian scientists argue that there was a Grand Solar Maximum that peaked in the late 20th century, and that we can expect a Grand Solar Minima (contributing to cooling) to peak around 2060… And finally, we can’t rule out unforeseen surprises. The hiatus in warming in the early 21st century was an unforeseen surprise. ~Judith Curry (House of Rep. Hearing, The President’s U.N. Climate Pledge, 15 Apr 2015

  6. Pingback: Are human influences on the climate really small? | Enjeux énergies et environnement

  7. Thank you Dr. Koonin for that clarification.

    • Dr. Koonin, do you have any thoughts you’re willing to share about the APS draft climate change statement ?

      • Danny Thomas

        HaroldW,
        Hope you get a response, but there is this on pg. 6 of the annotated WSJ article cited.
        “Individuals and countries can legitimately disagree about these matters, so the discussion should not be about “believing” or “denying” the science. Despite the statements of numerous scientific societies, the scientific community cannot claim any special expertise in addressing issues related to humanity’s deepest goals and values.”

  8. “…if the anthropogenic perturbation weren’t small, the detection/attribution discussion would be much more convincing than it is.”

    I kinda like what this Koonin guy has to say. It fits with my own personal bias regarding the trace gas radiative transfer model, biases which I have acquired from my own experiences in other areas of science.

    Just sayin’.

  9. Doug Badgero

    This in a system where the short term response is more dependent on its position on the attractor than on a percentish change in forcing.

  10. Paleoclimate tells us that the system is that delicate. Subtract 5 W/m2 and you get a deep ice age or even approach a snowball earth, while when you add 5 W/m2 you get an iceless hothouse with sea levels 70 meters higher.

    • Paleoclimate tells us warmer and warming is better than colder and cooling.

      Wamrer is better and less risk of dangerous climate change

      Colder is dangerous.

      • Certainly human mortality decreases in the warm season and increases in the cold season. Lots of other factors, of course, but the data say it:
        ‘Cold kills’.

        Think the president would be interested? Nah.

    • Little joshie will be along any minute now to demand your evidence, yimmy.

    • Paleoclimate tells us that the system is that delicate. Subtract 5 W/m2 and you get a deep ice age or even approach a snowball earth, while when you add 5 W/m2 you get an iceless hothouse with sea levels 70 meters higher.

      That is falacious.

      The ice ages were not caused by a global energy deficit.

      The ice ages were caused by a local energy change ( when there was still balance ) and the glacials IMPOSED the radiative imbalance.

      • I don’t think Koonin was referring to the cause of the change, just the magnitude. Plus or minus 5 W/m2 is a big deal in the last billion years, and it has been that type of swing between ice-dominated and ice-free climates.

      • Eddie,

        The ice ages were not caused by a global energy deficit.

        Well, that isn’t what JimD said. However, he is correct that paleoclimate studies suggest that the difference between a glacial period and an inter-glacial is a change in external forcing (CO2 plus ice-albedo) of about 5-6W/m^2. Of course, the overall radiative difference is probably 2-3 times this with feedbacks, but the point still stands. A change in external forcing of a few W/m^2 can have a substantial influence on our climate (unless you regard major changes in ice sheets as not being substantial).

      • Plus or minus 5 W/m2 is a big deal in the last billion years

        Given that solar forcing alone is of the order of 20W/m2 in the last billion years, due to the changing luminosity of the sun, and the solar forcing signal isn’t even visible in the geological proxies of the last billion years, I would suggest that 5W/m2 is actual relatively small beer in the context of radiation budgets of the earth over that timescale.

      • >>>The ice ages were not caused by a global energy deficit.<<<
        Well, that isn’t what JimD said.
        Uhmm… Yes it was. “Subtract 5 W/m2 and you get a deep ice age”

        This represents a gross misunderstanding of the glacials.

        The glacials are pretty poor place to compare anyway because of the uncertainty of proxy measurements, compounded with the uncertainty of the models imagining what went on, and further all the things that were different.
        Sea levels were 120m lower meaning the atmosphere over the oceans was thicker. Land surface including ice surface was 500m higher meaning the atmosphere over land was thinner. The tropics were warmer while the poles were colder.

      • Spence_UK, Royer has an interesting graphic that takes into account the general solar increase and changing CO2 levels as the main forcing terms, and the result is a surprisingly large offsetting effect between a weaker sun and more CO2 in the distant past. This graphic also illustrates that 5 W/m2 is a large change, given that 250 million years ago we also had icy conditions. Looking at projected changes for the future in this longer perspective is sobering.
        http://descentintotheicehouse.org.uk/wp-content/uploads/forcing1.jpg

      • Turbulent Eddie, in fact you could subtract much less than 5 W/m2 to get an Ice Age. The orbitally forced albedo changes translate to very weak forcing changes in the global average sense, and then feedbacks take over and it becomes hard to separate forcing from feedback by the time you have a full Ice Age.

      • Turbulent Eddie, in fact you could subtract much less than 5 W/m2 to get an Ice Age. The orbitally forced albedo changes translate to very weak forcing changes in the global average sense, and then feedbacks take over and it becomes hard to separate forcing from feedback by the time you have a full Ice Age.

        If Arctic summer temperatures are well enough above freezing, it doesn’t matter what the radiative forcing of the earth is, there will be no ice age.

      • TE, true, it is only below about 300 ppm that it is so easy to push us into an Ice Age. It gets much harder at 400 ppm because of the general warmth and its polar amplification, and by the time you reach 500 ppm, what little ice we have left will also be going away. The polar surfaces warm several times faster than the global mean.

    • it works the other way. if you add ice you reflect the extra watts per meter squares and if you melt and remove the ice you reflect less watts per meter squared. Albedo is adjusted and that changes temperature. They have it backwards. They use the Albedo as a feedback, but it is really the means to regulate temperature.

    • Actually at Ringberg, Annan had it at 8 W/m2 and 4K change in temperature. The ice ages I believe are caused in a change in distribution of forcing, while total forcing remains constant and then feedbacks cause albedo changes etc. The change in forcing in high latitudes during summer is I believe 50 W/m2, which is pretty large.

      • Ice Ages are triggered by albedo changes that initially occur with no other change in forcing, just orbital effects. The forcing change that starts an Ice Age is very small, but there is the ice albedo feedback that amplifies the forcing. Going in the negative direction, it doesn’t take much forcing to get an amplifying response in the surface ice area.

      • JimD, “Ice Ages are triggered by albedo changes that initially occur with no other change in forcing, just orbital effects.”

        The orbital effect is a change in the distribution of forcing from 65N maximum solar to just about anything else. 65N solar maximum is the point where land based ice has the least chance of survival. Since most interglacials tend to last about 1/2 of a precessional cycle, getting into an ice age is pretty easy but getting out of one generally requires a little extra boost from obliquity, at least for the past million years. None of the theories completely explain every glacial/interglacial transition, duration or depth (height) of temperature change. That would imply there is considerable internal variability that needs to be considered.

        funny how that works.

    • As I understand it the 8 W/m2 counts feedbacks. There is no change in total global forcing.

      • DY,
        According to these slides, the 8W/m^2 is the total forcing, which includes CO2 and the ice-albedo effect. It doesn’t, as I understand it, include the fast feedbacks.

      • It is difficult to separate cause from effect for the Ice Ages, so I might look at a 5 W/m2 negative forcing as the equivalent to reducing CO2 to about 110 ppm from 280 ppm. This would lead to an Ice Age being a much stronger effect than the Milankovitch effect.

    • Your statement *may* be true – the problem is that anyone who purports to know what the W/m2 was in the past, is either doing it through proxies or models.
      And neither proxies nor models have shown any skill with modern measured comparisons, so ass-u-me-ing past or future conditions based on these proxies and models seems highly speculative.

  11. So, the radiative forcing of CO2 ( plus feedback ) is small compared to natural variation.

    The presumed temperature change is small compared to synoptic variation ( cold fronts, ridges, and the like ), not to mention diurnal and seasonal variation.

    And the observations of temperature rise are less than the low end projections. They should be, because the actual forcing imposed has been less than the low end projection:
    http://www.columbia.edu/~mhs119/GHGs/dF_GHGs.gif

    Smallness indeed.

  12. Ah, but Steve K. Mosher and company can measure global temp to 1/100th of a degree. So, yes, our contribution is small, but BEST can definitely measure the global temperature of the Earth using thermometers that are widely scattered, read at different times of day, moved around, and weren’t really accurate to +/- 1 or 2 degrees to 1/100th of a degree. Of course they can.

  13. ” Second, it [smallness] means that natural variations can easily overwhelm human influences”

    It doesn’t mean that. You’d have to quantify natural variations. A 1% increase in water in the oceans would have big consequences, for example. In fact, with emissions we are far exceeding natural variability in non-condensing gas constitution of the atmosphere.

    A percent of something very big is big. The greenhouse effect is very big.

    • That would be a lot of water, nicky. Please illuminate us on the consequences. While you are at it, tell us what the consequences would be of removing 1% of the dirt from Mt Everest. Something really scary to think about, melting 1% of the Antarctic ice. Taking 1% of Warren Buffet’s money, while he isn’t looking. Would he feel that?

    • Don’t worry about the 101% ocean, nicky. We will build an ark.

    • A 1% increase in water in the oceans would have big consequences, for example.

      I’m not aware of any such prediction/projection.
      Thermal expansion appears to be a much larger predicted term.
      However, even there I have my doubts.
      Heat into the oceans seems to be the meme.
      But it’s pretty clear that the oceans more readily store colder than average waters, and not warmer than average:
      http://www.drroyspencer.com/wp-content/uploads/Ocean-temperature-vs-depth.png

      Is recent oceanic heat storage a natural fluctuation due to varying rates of bottom water formation?

      More Anarctic sea ice -> less deep water formation?

      If so, is thermal expansion also overstated?

    • “A percent of something very big is big. The greenhouse effect is very big.”

      Well according the greenhouse effect theory: http://en.wikipedia.org/wiki/Greenhouse_effect
      the greenhouse effect is caused by greenhouse gases and these greenhouses are suppose add 33 C.
      Not sure who exactly came up with the idea that only greenhouse gases warm earth and that warmth was precisely +33 C.
      But how it was done was to pretend Earth was an ideal blackbody which would cause this blackbody planet to have uniform temperature of 5.3 C Earth distance from the sun.
      And then if add white clouds it’s suppose to get much cooler.
      And only the greenhouse gas are suppose to make up that difference cause an average temperature of 15 C. Or Earth is quite cold now, and once upon a time earth had average temperature of 25 C, which obviously less cold, and dinosaurs were quite happy. If you believe that only greenhouse gases can make earth warmer. And if assume Earth did indeed once have average temperature of 25 C, then you forced to believe that the +10 C warming was caused by greenhouse gases.

      But no one actually believes this, yet that is what theory says.
      The theory also says that water vapor causes 36–70% of this very precise number of 33 C of warming added.
      Which means there a huge uncertainly of how much warming is caused by
      water vapor according to the theory.
      Though some believers say clouds actually cause a lot warming. But of course clouds are water droplets or ice particles, and therefore they are not gases or greenhouse gases.
      But seems one must be flexible and allow gases to mean liquid or solid water. Or look a older definitions of gases they meant clouds- because no one actually believe actual gases existed.
      So if look sideways, one can see that believer are not claiming something contrary to their doctrine and one should assume that clouds are actually greenhouse gases. Of course ice particles on the ground and droplets of water in the sky, were suppose to cause cooling. So best not to get into these details and it tends to confuse people.

      So 33 C is big.

    • > You’d have to quantify natural variations

      Go ahead, please

      • Ian

        That quantification is the greatest natural variations we can discern in the Holocene between warmth and cold that lasted any time (i.e not one off ‘spikes’)

        I tend to use the MWP and the LIA as the benchmarks, as those are reasonably possible to quantify without going into novel proxies, although periods earlier in the Holocene appeared to be warmer there appear to be no colder periods than the LIA..

        So, from that we can deduce periods somewhat warmer than now and periods much colder than now as the extreme ends of natural variability.

        tonyb

      • Thanks, tonyb

        I’d actually addressed that question to Nick Stokes (an exercise in futility, I know), but if you’re using the MWP and LIA from the last millennium as boundaries, then on your model we have no real issues to argue about

    • Nick Stokes | April 8, 2015 at 10:49 pm | Reply
      A 1% increase in water in the oceans would have big consequences,

      Isn’t that a 2 way street in that that change could come from natural or anthropogenic sources? And since natural sources are so much larger than anthropogenic, it would take a small natural change but a large anthropogenic change to have that 1% increase in water in the ocean.

    • Nick Stokes,

      A 1% increase in water in the oceans would have big consequences, for example.

      What do you mean by big consequences? How do you measure the consequence? The most widely used method is the economic impact. But the economic impact of the projected sea level rise to 2100 is trivial – e.g.$1 trillion in $30,000 trillion cumulative global GDP loss for a 1m sea level rise to 2100: http://www.springerlink.com/content/851112j434t26502/fulltext.pdf
      http://link.springer.com/article/10.1007%2Fs10584-012-0613-3#page-1

      Is a 1% increase in ocean volume realistic? (source please)

    • stevefitzpatrick

      Nick,
      40 meters of sea level would a big deal, but I think you know that is not going to happen in even 1000 years (Antarctica is not going to melt any time soon). The legitimate point is that a 1% change in total atmospheric resistance to heat flow to space is a relatively small change compared to the spacial and temporal changes in that resistance which are happening all the time (seasons, geography, weather, ENSO, PDO, AMO, solar cycle, etc). So accurately measuring the influence of a 1% net change in atmospheric resistance to heat flow is a non-trivial task. How much of the relatively slow change in temperature over the last dozen years was due to natural processes? How much of the more rapid 1975 to 2000 warming was due to natural processes? How much warming will take place between now and 2100? What will be the consequences of the warming that will take place, both positive and negative? These are all questions which are clearly very difficult to answer, and anyone who says they know the answers with any certainty is deluding themselves. IMO Koonin is just confirming there is a lot of delusion going around WRT global warming; sort of stating the obvious.

      • I got the notion that Nick was just addressing the use of percentages to describe the magnitude of an effect. A 1% increase in water volume in the ocean is a very, very big amount of water.

        Either way, I think that’s a valid point.

        Suppose you have a D/C-coupled z-axis accelerometer (so your device always reads an offset associated with “1 g”). If you use Dr. Koonin’s logic, you’d compute all of your measurements relative to that “1g” (offset) value. But that isn’t a meaningful approach to quantify “big versus small”.

        Big versus small measurements in the context of this accelerometer actually have to do with your dynamic range and the resolution of your device. A quantity is “big” relative to my measurement system if I can achieve at least 40-dB SNR.

        The fact that 40-dB SNR corresponds to only 0.01% relative to the DC offset of “1g” is a generally useless piece of information.

      • stevefitzpatrick

        Carrick,
        Sure, 40 dB S/N in any measurement is pretty clear. But that is not the case WRT atmospheric resistance to heat flow… more like 1 dB S/N….. and it is difficult to make an accurate measurement under those circumstances unless you do a lot more than collect a bit of data. Seems to me Nick was making a point, which while trivially true (1% of the ocean is a lot of water), does not address the issue that Koonin raises: accurate determination of the response to GHG forcing is a very difficult proposition. Nick is smart enough to make a more subtle contribution, if he chooses to.

      • Steve, I generally agree with your comments. The only thing I’d add is the obvious statement that it gets less difficult to measure a 1% effect on a climate system as you make the measurement period longer, as long as you can characterize the natural fluctuations in the system.

        Long-period natural variability issue is, to me, the major question that needs to be addressed. But I don’t think this is something that can be meaningfully be framed in terms of percentiles. Not like this anyway.

      • Ignore the millennial at your perennial. Nice chalk session there, guys; thanks.
        ===============

    • Well Nick, yes and no. Another point about Koonin’s point is that numerical simulations become problematic if the change is this small in relative terms. Bear in mind that numerical truncation error analysis is always in terms of a fraction of the total value of the variables. It is not impossible, its just very difficult.

    • Is your statement true? Doesn’t tide vary sea water levels on any shore far more than 1%? What about seasonal ice? Seasonal and daily temperature variations are also far greater than 1%.
      Why then would a 1% change in the overall water level in the ocean necessarily mean a huge change?

      • Well, I thought that tides weren’t overall water level. A static amount of water just sloshes from one shore to the opposite shore according to the moon’s pull right? That’s not the same as an overall increase in the water level. Not that I’m saying that such a 1% water level increase would be a disaster, or even a problem.

  14. John Vonderlin

    Dr. Koonin,
    Thanks for your expansion. Are either of these appropriate analogies for your serious/small assertion?
    I am considering taking out a 4.0% 20 year balloon payment mortgage on a house I buy, but then have an opportunity to get one that is a 1% better offer (That is 3.96%.) I’d be hard pressed to find any indication of the “small” interest difference by looking at the money in my pocket or checking account during those twenty years, but would definitely notice the “serious” difference of what I owed when the balloon payment came due.
    Or is your small/serious assertion closer to: Knowing my monthly payment is $2 less a month with the 3.96% loan, I faithfully stick that “small” amount into a piggy bank, saving it until I have enough to splurge. Unfortunately, I drink the whole bottle of fine wine I treat myself to and receive a “serious” beating when I crash my car into a bunch of motorcycles outside a biker bar.

    • I think you may have the proper analogy that explains all the alarm about global warming –e.g., using a 1 or 2% hotter pepper to spice a burrito may not be noticed at first, especially if the pepper to burrito ration is measured in ppm but if that extra hotness causes you to drink more beer and that causes you to carelessly veer into oncoming traffic on the way home from the taco shop, you might accidently run over the last polar bear.

    • stevefitzpatrick

      Driving drunk is unwise, and a poor parallel to the point Koonin is making.

      • I think Koonin would agree that crashing into a bunch of motorcycles outside a biker bar is obviously unwise but running down the last polar bear may be a sign of too much warming.

  15. Steve –

    You opened your WSJ op-ed with the following:

    The idea that “Climate science is settled” runs through today’s popular and policy discussions. Unfortunately, that claim is misguided.

    Do you have some examples of scientists saying that “the science is settled?”

    If you do, can you explain what those scientists meant by that? Did they mean that there are no uncertainties w/r/t the impact of ACO2 on the climate? Or did they mean what you say in your article – that there is generally acceptance among scientists that the GHE is real and that it has the potential to significantly affect the climate?

    if you don’t have any examples, I would have thought that while it might be legitimate to describe the ways in which the science isn’t “settled,” you might have taken the time to explain that scientists are not actually making the claim there are no uncertainties. It seems likely that many readers might have read your WSJ article and been left with a mistaken impression about the arguments that scientists are making.

    Although “skeptics” often claim that scientists have said that the “science is settled,” when pressed they seem unable to find many examples, if any at all.

    Please remember:

    The political and diplomatic spheres are best suited to debating and resolving such questions, and misrepresenting the current state of climate science does nothing to advance that effort.

    Surely you would agree that misrepresenting scientists as “skeptics” often do, by asserting that the scientists have made claims of certainty that they haven’t made, does nothing to advance the effort to debate and resolve the complexities of AGW?

    The public is done a disservice if they think that scientists are more certain than the science supports. But by the same token, don’t you think you would be doing the public a disservice if you contributed to a misunderstanding among some members of the public w/r/t the certainty climate scientists actually state in their empirical analysis?

    • Just to acknowledge, I’m not suggesting that you claimed that scientists have said that “the science is settled.” You didn’t do that.

      But it does seem ito me to be mportant to clarify the differences that exist between what scientists actually say and the claims made about what they say/the claims made by non-scientists.

      Conflating what scientists say with what non-scientists say (or to repeat, risking giving the impression that scientists have said things that they haven’t actually said), does not seem to me, at least, to be likely to advance the public understanding of the state of the science.

      • It’s not the scientists joshie, it’s odumbo, kerry, gore, cook, and other agenda driven activists trying to influence the popular and policy discussion. As is stated here often, read harder

    • Little joshie is lucky that Judith doesn’t count the number of words, instead of the number of comments. He would be in moderation, again. Still, he makes the mistake of having to add dopey afterthoughts to almost every comment he spams at us. I hope Springer is lurking around and keeping track of the yammerings of our loquacious nonentity.

      • Just as I’m sure some sat in front of their Victrola lovingly engaged in the beauty of their own voice, Joshua may just sit and marvel at the brilliance of his prose. Collections of his works could be destined for the Antiques Road Show.

    • Oh it seems you caught that possible misrepresentation in your comment and corrected before I posted. Nicely done, I approve.

      Although the couple quick quotes I found still shows that people speaking officially for and as climate scientists are saying things very close to that. Not all of them, to be sure but it is enough of a problem that it does need to be addressed.

      • Brandon C –

        Thanks for your 2nd comment. Yes, I realized after I wrote my first comment that it could easily have looked like a misrepresentation.

        I think that it’s important to clarify what scientists are saying, and that if scientists do say things that don’t accurately represent the state of the science, it’s important to make that clear.

        On the other hand, I think it is important to make it clear when people are mischaracterizing what scientists say about the state of the scientists. IMO, it is important to stress that most climate scientists are careful to speak in terms of probabilities, error ranges, confidence intervals,etc.

        It is perfectly valid, IMO, to make scientific arguments about quantification of ranges or probabilities – but I often see it said that climate scientists (as a group) claim total certainty, and to do so is just as of a problem as it is when someone portrays the science as being more certain than it really is.

    • > Do you have some examples of scientists saying that “the science is settled?”

      A small example:

      Virtually all of the references are from IPCC AR5 WGI, so they are surely settled science.

      http://judithcurry.com/2015/04/08/are-human-influences-on-the-climate-really-small/

      That’s not science, but it’s important.

      • What’s interesting about that, willard, is that I wasn’t sure if Steve was being sarcastic.

      • Danny Thomas

        Hmmm. Seems that others besides Dr. Koonin have picked up on that “meme”: “The science on climate change has long been settled, and scientists agree that we must take urgent, aggressive action.” and since it’s in an article about “giving facts a fighting chance” and all………..
        (6th para from top). http://www.huffingtonpost.com/tom-steyer/giving-the-facts-a-fighting-chance_b_7025730.html

      • > Seems that others besides Dr. Koonin have picked up on that “meme” […]

        Dr. Koonin might be the only synthetic expert who’s swinging that meme from both sides. Perhaps because he’s the only expert so far identified as a “synthetic” expert. The box is only one day old after all.

        I would rather put our Beloved Bishop in the True Scot expertise box. A box for Matt King Coal, David Rose, and James Delingpole might also be needed. Either synthetic synthetic expertise or interpretative expertise would suit me fine.

        That’s not science, but it may be important.

    • Joshua,

      On a BBC Radio4 programme a couple or more years ago, I heard a lady from the UK Met Office (I think, Julia Slingo) say “the science is settled”. I thought it obvious that she meant the Greenhouse Effect does exist. But it could easily have been heard other ways.

      • anng –

        Yes, if a scientist says that, it could easily (and understandably) be misinterpreted to mean that that there are no uncertainties. Because it is ambiguous, the term shouldn’t be used. Good science should not be ambiguous in such away. And I don’t doubt that sometimes the term is used in a rhetorical fashion by “realists” – because they are exploiting ambiguity.

        On the other hand, some “skeptics” have also repeatedly exploited that ambiguity, and it falls in line with many “skeptics” not recognizing the extent to which “mainstream” climate scientists do quantify uncertainty.

    • Once again Joshua occupies himself with pedantic and off topic concerns while ignoring the science in a (simple, concise, and easy to follow) technical article, against forum guidelines on guest contributions, if I’m to understand correctly. Makes one wonder: does Joshua actually care about the topic at hand? Or is he primarily concerned with endless nitpicking and debate for debate’s sake?

      If you’re genuinely concerned about removing the “settled science” meme from climate discussions you’re barking up the wrong tree, Joshua. Now, why don’t I believe you’re off to spend your day admonishing the proper culprits? Make a liar out of me Joshua. Go do some good instead of endlessly stirring pots.

  16. richardswarthout

    Joshua

    Are you suggesting that the president made up his claim that “the science is settled”? He did not consult his chief scientist?

    Richard

  17. richardswarthout

    Dr Koonin

    Regarding the requisite precise measurements; how distant is the capability?

    Richard

  18. One could be forgiven for misinterpreting statements like:

    “There is, even today, a Flat Earth Society that meets every year to say the Earth is flat. The science about climate change is very clear. There really is no room for doubt at this point” ….from IPCC head (ex) Pachuri, as claiming the science is settled. But it wasn’t exact, and he isn’t really a scientists, so bonus points to you, but on the other hand he was leading the IPCC….. But of course nobody ever corrects the thousands of articles and interview where they call him a “top climate scientist”. Can you point to a comment you ever made outraged at the endless misrepresentation of non-climate scientists, as climate scientists in the media?

    ” “…we have as much confidence in the basic climate science as we have in the fundamental science of plate tectonics or biological evolution.” Ken Caldeira

    “Poorly understood aspects of climate change do not change the fact that a great deal of climate science is well understood. To argue that the 5% that is poorly understood disproves the 95% that is well understood betrays an incorrect understanding of the nature of science.” John Cook (added because he has been a force to try and perpetuate “consensus” at all costs, even integrity.)

    How about “The debate is over about whether or not climate change is real. Irrefutable evidence from around the world—including extreme weather events, record temperatures, retreating glaciers and rising sea levels—all point to the fact that climate change is happening now and at rates much faster than previously thought”…. from David Suzuki.

    These are just a few quick examples I found in a couple minutes. But it makes the point that many are arguing a version very close to that, but not exactly. Is it possible to use a catch phrase, like Al Gores “the science is settled”, to encompass the endless barrage of alarmists (scientists and not) saying variations on that theme? May not be an exact quote but isn’t exactly inaccurate. Saying “science is never 100% settled but pretty close on climate change”, is not significantly different in spirit from the “settled science” line.

    Do you often comment in annoyance when people mis-represent the most common sceptical position as “denies man is having an effect” or “denies that it has warmed” or “believes climate isn’t changing”? When everyone who can read clearly understands that the most common sceptical position is “there has been some warming and we are having an effect but we don’t know the natural processes and feedbacks well enough to say what that effect will be” and ” that they don’t accept it has been shown that the net effect of increasing CO2 will be bad”, my paraphrase, but I bet most of the sceptics on here would largely support that with few caveats. So your attention to details on attribution of an exact phrase is really wonderful, but seems selective in reality.

    When you complain of someone saying, “The idea that “Climate science is settled” runs through today’s popular and policy discussions.”, is inaccurate, since no scientists ever said that exact quote, is……….well it is horribly inaccurate. Since he said “the idea” and then qualified that this idea “runs through today’s popular and policy discussions” is a very accurate description of what is actually being discussed in those areas. That is exactly what people are often discussing in “popular and policy discussions”, Which is in reality what you actually are complaining about while taking exception to him pointing it out in the first place. Although….. did he say scientists said exactly that? Did you just misrepresent what he actually was saying? Hmmmm, something about glass houses and bricks seems to spring to mind.

    • +95% ;)

      • The alarmists are in the bargaining stage. In a few years you’ll not find a climate scientist willing to speak publicly about alarm.

        It will always be ‘others’ who were alarmists. The scientists will blame the politicians and the politicians will blame the scientists, and both can blame the financiers. There, everybody happy, and electricity rates will have skyrocketed.
        =================

      • They used Kuhn (and the Plate Tectonics Revolution) as a blueprint for building their manufactured paradigm. Why not use Kuhn’s discussions about how Science rewrites history as a blueprint for their CYA?

      • “The alarmists are in the bargaining stage. In a few years you’ll not find a climate scientist willing to speak publicly about alarm.

        It will always be ‘others’ who were alarmists.”

        ****

        Well, it certainly was this way after the housing boom. Now everybody “knew” it was a bubble – when those same people at the time were saying that real estate always goes up.

    • Brandon C –

      ==> “Do you often comment in annoyance when people mis-represent the most common sceptical position as “denies man is having an effect” or “denies that it has warmed” or “believes climate isn’t changing”? When everyone who can read clearly understands that the most common sceptical position is “there has been some warming and we are having an effect but we don’t know the natural processes and feedbacks well enough to say what that effect will be” and ” that they don’t accept it has been shown that the net effect of increasing CO2 will be bad”,”

      First, it is my impression that some “skeptics” underestimate the % of “skeptics” who are active in the “skept-o-sphere” and who reject the GHE of ACO2 outright, or who say that it could not in any way have a significant impact on the climate, or who say that they know that the impact will absolutely be miniimal.

      2nd,iIt would be a mistake to look at the comments in the climate-o-sphere and extrapolate from that to determine what the “most common skeptical position” is.

      http://www.culturalcognition.net/storage/no_warming.png

      Now I think those polling data might represent the “state of scientific belief” among “skeptics” in an entirely accurate manner. It could be, for example, that there were methodological flaws, or it could be that when some “skeptics” chose “no warming” they mean not anthropogenically-caused warming (even though they had the choice to pick a-cause warming and “natural” warming separately). But I do think that there are a significant # of “skeptics” who doubt that any warming is taking place, and who also do not have an accurate understanding of the state of the science. That wouldn’t be surprising, just as it wouldn’t be surprising that most “realists” likewise form their opinion without knowing the state of the science.

      But in line with Steve Koonin’s goals, the important thing is to accurately investigate the state of public knowledge about climate change and to try to have them be accurately informed – rather than to perpetuate misinformation.

      • Sorry – that should have said:

        ==> “Now I think those polling data not might represent the “state of scientific belief” among “skeptics” in an entirely accurate manner”

      • Can you identify the paper your figure is from?

        It is fine to quote a paper and if you can supply it i will look at it, but on this site for example, the vast majority of the sceptics do not deny warming or some human influence. The same holds true of many other blogs i frequently read. In fact on many “denier” blogs people who deny ghe are regularly attacked for their position by the majority.

        The problem in the climate trenches, is that strawmen and misrepresenting the opponents position is the norm. But trying to paint all sceptics with a unified brush, is no different than the “settled science” brush. Yes settled science scientists exist and so do people who deny any human influence. But there is alot of nuanced ground between those extremes.

      • Brandon C –

        From the same source:

        http://www.culturalcognition.net/storage/thumbnails/4177295-25350444-thumbnail.jpg

        The source:

        http://www.culturalcognition.net/blog/2014/8/22/for-what-its-worth-breaking-down-belief-in-gw-vs-belief-in-a.html

        ——————

        ==> “It is fine to quote a paper and if you can supply it i will look at it, but on this site for example, the vast majority of the sceptics do not deny warming or some human influence.”

        Not sure how you determine what comprises a “vast majority.” I have seen quite a few “skeptics” that post arguments here that say that there is not GHE from ACO2, or that it is known that ACO2 cannot significantly affect the climate or that we know that any effect will be absolutely minimal. I don’t think that it is a majority, but I think that it is a significant minority. I have commented many times in direct response to comments here from “skeptics” who hold such views that they are being thrown “under the bus” because they are inconvenient for other “skeptics” who wish to characterizes “skeptics” as a group as not presenting such arguments (when they aren’t explaining that “skeptics” aren’t monolithic).

        ==> “The same holds true of many other blogs i frequently read. In fact on many “denier” blogs people who deny ghe are regularly attacked for their position by the majority.”

        “Skeptics” who doubt the basic physics of a GHE from ACO2 are often met with opposition on “skeptic” blogs. And often they aren’t. And consider blogs like Tallbloke’s.

        I don’t think that I’ve often seen “skeptics” – who argue that there is no theoretical basis for believing there there could be a significant effect (as opposed to any effect at all), or who argue that it is certain that the effect would absolutely be minimal – met with strong opposition on “skeptic” sites from “skeptics.” Consider the late Jim Cripwell as an example: search for his arguments here and examine the amount of resistance he received from “skeptics.”

        ==> “The problem in the climate trenches, is that strawmen and misrepresenting the opponents position is the norm.”

        I agree, completely. And that problem rests upon a more fundamental problem, IMO – which that people are engaging in these discussions w/o a clear and objective definition of terms. For example, we see threads filled with arguments about whether or not Muller is a “skeptic” – when the entire discussion is a joke because people are operating from different starting assumptions about what comprises “skepticism.”

        ==> “But trying to paint all sceptics with a unified brush, is no different than the “settled science” brush.”

        Agree. And under-representing the # of “skeptics”‘ who believe that no warming is taking place is no better or no worse than under-representing the number of “realists” who are certain that unless we stop emitting ACO2 immediately, it is an absolute certainty that “catastrophic” climate change will result.

        ==> “Yes settled science scientists exist and so do people who deny any human influence. But there is alot of nuanced ground between those extremes.”

        I am all in favor of increasing the clarity and nuance of these discussions.

      • We will take that BS as a “no”, joshie.

      • Hi Don –

        How are ya’, buddy?

        ==> “We will take that BS as a “no”, joshie.”

        I saw one question from Brandon in his most recent comment to me. I answered that question.

        There were two questions in his previous comment directed towards me. I believe that I addressed both.

        Was there something that I missed?

      • Thanks for the link Josh

        A few notes:
        The questions asked, “From what you’ve read and heard, is there solid evidence that the average temperature on earth has been getting warmer over the past few decades?” and ” do you believe that the earth is getting warmer (a)mostly because of human activity such as burning fossil fuels or (b) mostly because of natural patterns in the earths environment?” are not precise or specific enough to refute my position on their own.

        The timeline of “past few decades” is too non precise and could be interpreted to mean about 20 or even more years probably up to 40 or more. I would argue that I am convinced there has been significant warming in the last 40 but not strongly convinced for 20. Too wishy-washy, could have been precise but missed the opportunity to get a clear answer. Instead we have to try and guess the actual timeline that was interpreted. Could be talking about 20 years and the pause, or could be 40 years and talking about pre-pause warming.

        It is fully possible to believe man is having a real effect, but not “mostly” the cause of warming. I don’t really see how the exact wording actually conflicts with my comments: “there has been some warming and we are having an effect but we don’t know the natural processes and feedbacks well enough to say what that effect will be”. It is very easy to interpret the question about asking if there has been warming over the last 20 years, and many would say that there has been little to none, hence the whole “pause” subsection of climate debate. But it is clearly possible to believe there has been warming over 30-50 years and man has been at least partly responsible, without believing there has been warming in the last 20.

        Frankly those questions are not well suited to refute my position at all, a poor choice, but maybe all that was available. You must realize that it is possible to believe “that man is having an effect, that there has been some warming but little to none over the last 15-20 years, and we don’t understand the feedbacks and cannot say man is responsible for all the warming” and not reject GHE and not refute the idea that there has been any warming. The ground between the alarmists and deniers is mostly made up of feedbacks, not GHE. It is a misrepresentation of the most common denier position to claim it is about GHE, rather than feedbacks.

      • Brandon C –

        ==> “Frankly those questions are not well suited to refute my position at all, a poor choice, but maybe all that was available.

        Well, you make some good points, but to paraphrase Rumsfeld, you examine public opinions based on the data you have, not the data you want or wish to have…

        I acknowledge the problems that you outline with the polling data (and indeed, I see some support for your speculation in that as the “conservative” respondents increased in “climate science intelligence” and perhaps were thus more familiar with the phenomenon of the “pause,” so did the likelihood of them selecting “no warming” among the choices), But look at that first chart – where more than 50% of the “conservative” respondents selected “no warming”

        So let’s skip over my first main point (w/r/t whether your speculation about the prevalence of various views in the “skept-o-sphere” is accurate) to my 2nd main point – which is whether it is a good idea to extrapolate from your experiences in the “skept-o-sphere,” among a cohort that are inherently outliers (by virtue of their high level of involvement), to characterize the broader public who poll as “skeptical” about climate change (the majority of whom are mostly uninvolved or only peripherally involved at best). .While I think it is certainly possible that some % of the “conservative” respondents sampled from the larger group (including the subset of “involved” “skeptics) would have selected “no warming” because they were thinking of the “pause,” I think that more than likely, a high % of Joe Public “conservatives” are not at all familiar with the short-term slowing down over the past decade + in the longer term trend of mean GSATs. Thus, I would say that “the pause” is an unlikely explanation for the answers of such a high % of the poll respondents.

        There are other, related data that an speak to some of this. For example:

        https://tamino.files.wordpress.com/2011/10/q74.jpg

        https://tamino.wordpress.com/2011/10/09/survey-says-2/

        There we can see that in association with degree of being “conservative” the respondents selected “don’t need more information to make up [their] mind.” Two points strike me about that. The first is that I think that it is unlikely that reality, there is such a strong association between being more “conservative’ and being more “informed.” The 2nd is that I think that it is unlikely that, for example, more than 50% of the more “conservative” respondents are really in a situation as to be able to decide about the scientific evidence related to climate change without getting any more information (as the data show they self-evaluate their own knowledge level). That suggests to me that there is not a general pattern that “conservatives” form their views on climate change as the result of greater understanding about phenomena such as “the pause” Not to say that for any particular individuals that pattern might not exist – or even that in general, there isn’t a relatively weak association between greater knowledge about climate change and greater “skepticism” among “conservatives (indeed, I think that the available data show that there is).

        With reference to that 2nd main point:

        ==> “The ground between the alarmists and deniers is mostly made up of feedbacks, not GHE. It is a misrepresentation of the most common denier position to claim it is about GHE, rather than feedbacks.”

        I’m not sure what you are referring to with “deniers” there (whether you’re meaning to distinguish them from “skeptics’) – but I would say that the vast majority of “skeptics” have no idea what “feedbacks” are outside of perhaps a microphone screeching when someone’s talking at a wedding.

        So again, I think that irrespective of whether you are accurately assessing the prevalence of views of “skeptics” in the “skept-o-sphere,” you are leaning too heavily on assumptions about extrapolating from what you read in the “skept-o-sphere” (or perhaps your own views) to characterize the views of the broader “skeptical” public.

      • Well, Josh, it could be that. Or it could be that your are simply full of it. On further consideration, I’m pretty sure it’s the latter.

      • jim2 –

        ==> “Well, Josh, it could be that. Or it could be that your are simply full of it. On further consideration, I’m pretty sure it’s the latter.”

        You know, it’s kind of funny because like Don, and the Chief, you seem to get some kind of a kick out of ad homs, and like Don, and the Chief, you have stated more than once that you were going to stop reading and responding to my comments only to fail to live up to your stated intention.

        Not that I would really want you to stop reading and responding, and not that I ever believed it when you said that you would, of course. :-)

        Anyway, as always, I thank you for reading and responding. I can’t tell you how much it means to me.

      • No problem, Josh.

    • stevefitzpatrick

      Brandon C,
      Clear and thoughtful comment. But you will have no impact on those who really do think ‘the science is almost settled…. time to impose our political views’.

    • Semantics often get in the way of any effective dialogue on the subject of climate science and the policy options being suggested. The concepts of global averages and sea level changes do not sit with what I understand is really happening out there.

      Regional variations in climate are never uniformly in one direction at the same time and the mean sea level changes are never uniformly experienced across all the ocean basins. Natural variability of climate seems to have kept it within remarkably stable bounds over millions of years.

      It is understood that the poles have been iceless for more than 70% of the Earth’s evolution from a firey ball to what it is today. The long term trend must surely be cooler, as the centre of the globe gradually cools and the Sun gradually wanes.

      There have been large areas of woodlands on the antarctic continent which have evolved long after the last major tectonic shifts have moved the continents around the globe to where they are situated today and thereby affecting where ocean currents prevail.

      I would prefer more geology and meteorology input into climate science and less emphasis on current recorded data and models in estimating trends in regions around the world. The domain of climate has never been adequately determined and as its trajectory spans billions of years PDF to be drawn from this population must therefore be heavily qualified.

  19. harrytwinotter

    The article still needs to explain why the global average temperature and ocean heat content continue to rise. I don’t buy the ” current stasis in global mean surface temperature” argument, the average surface temperature seems to be increasing in line with it’s long term trend as far as I can see.

    • Not a solid arguement.
      Observed Ocean heat content very uncertain prior to ARGO, and post ARGO well below what models predicted, which is what led to the laughable Balsama and Trenberth paper claiming it was somehow hiding below 2000 meters where ARGO cannot see it.
      As for temperature trends, yes we are still emerging from the LIA and have yet to reach the levels of the MWP, as Greenland today evidences. The historical rate of rise from 1900 means we have nothing to worry about by 2100. Now, if you mean only the supposed rise from about 1975 to 1998 that climate models project out to 2100, Koonin’s post simply points out that the subsequence temperature stasis (surely you know about that) calls into question the anthropogenic attribution to that rise. A ‘percentish’ anthropogenic change (maybe) in a large natural background known to vary substantially more than ‘percentish’ on multidecadal to multicentennial scales requires more precision over longer time periods (e.g. Argo) to sort out.

      • harrytwinotter

        If you want to be taken seriously, lay off the ad hominems.
        “still emerging from the LIA” citations please. I am not even sure what this means.
        “yet to reach the levels of the MWP” citations please.
        “Greenland today evidences” Seriously? Are you claiming Greenland is a proxy for the global average temperature?

      • Well, I take Rud very seriously Harry, and I don’t think you scored any points, because:

        “If you want to be taken seriously, lay off the ad hominems. ”

        Sorry Harry – I didn’t hear any ad hominems. He called a paper “laughable,” that’s all.

        “still emerging from the LIA” citations please. I am not even sure what this means. “yet to reach the levels of the MWP” citations please.
        “Greenland today evidences” Seriously? Are you claiming Greenland is a proxy for the global average temperature?”

        No Harry, we don’t need any citations for any of that, and No Harry, he didn’t claim Greenland is a proxy for GAT, because the existence and timing of the LIA is established already, and we know we’re coming out of it and haven’t reached the levels of the MWP yet, because of treelines still being lower around the world than they were (I’ve seen it myself, and don’t need a cite) and Greenland isn’t green yet – although it used to be during the MWP, and I don’t think we need a cite for that either, cause of the name and the archeological evidence and all that. We can’t grow grape vines in London yet either, though we know we could and did during the MWP

        You should know all this Harry.

    • “I don’t buy the ” current stasis in global mean surface temperature” argument, ”

      No, of course not Why buy facts when they don’t support your beliefs. I’d suggest asking yourself how it is that we’ve now seen so many explanations from establishment climate scientists for the pause you say doesn’t exist.

      In even simpler terms, if the heat isn’t missing what are they looking for it?

      • harrytwinotter

        “for the pause you say doesn’t exist” Building a straw man is not good for a discussion.
        “In even simpler terms, if the heat isn’t missing what are they looking for it?” What does that mean?

  20. Well, interestingly, none of the arguments here seem to be the same as the arguments that were being made in support in the last post. The actual statement was

    shift the atmosphere’s natural greenhouse effect by only 1% to 2%.

    There are probably two ways in which the natural greenhouse effect would typically be quantified: an effective surface temperature 33K higher than in the absence of the greenhouse effect, or a radiative forcing of 150W/m^2. The post seems to be doing a comparison with total fluxes (503W/m^2), or total temperature (288K).

    If one were to make the comparison with what most would regard as the natural greenhouse effect, then by the mid 21st century, temperatures will be expected to have shifted – depending on our emission pathway – by between 1.5K and 2.5K (4.5% to 7.5% of 33K), anthropogenic forcings will be between 2.5W/m^2 and 5W/m^2 (1.5% and 3.3% of 150W/m^2) and the total radiative influence of anthropogenic effects – forcings plus feedbacks – will probably be between 5W/m^2 and 10W/m^2 (3% and 7% of 150W/m^2). So, it seems that you can get small percentages as long as you define the natural greenhouse effect in a way that – IMO – is non-standard.

    So, again, I’m confused as to how the phrase could be atmosphere’s natural greenhouse effect when all the comparisons are made with quantitiesthat most would regard as not the standard descriptors of the natural greenhouse effect.

    On the other hand, if we want to do this, then every 1K in increase in average temperature produces a roughly 0.7K increase in wet-bulb temperature. Maximum wet-bulb temperatures today are 30-31 degree C. A 10K increase in average temperature would then likely increase maximum wet bulb temperatures well above the threshold for mammalian life. By the argument here, that’s only just a 3% shift, so quite small. No problem then.

    • by between 1.5K and 2.5K (4.5% to 7.5% of 33K),

      This doesn’t seem correct to me.

      I suggest a change of 1.5K to 2.5K is a 0.5% to 0.9% change (i.e. 1.5 / 273.16)

      • Correction:
        I suggest a change of 1.5K to 2.5K is a 0.5% to 0.9% change (i.e. 1.5 / 288)

      • Peter,

        I suggest a change of 1.5K to 2.5K is a 0.5% to 0.9% change (i.e. 1.5 / 273.16)

        Where does 273.16 come from? The greenhouse effect is typically taken to produce an increase in effective surface temperature of 33K relative to what it would be in the absence of an atmosphere and with an albedo of 0.3. Hence 1.5K to 2.5K is 4.5% to 7.5% of 33K.

      • I may be wrong but I don’t understand why you are using 33K as the denominator. I think a 2K change in the average surface temp – e.g. from 288K to 290K – is a 0.7% change.

      • Okay, yes, that’s what Koonin is suggesting above too. However, most people regard the greenhouse effect as producing a 33K increase in effective surface temperature.

      • But, that seems incorrect to me. If you did that calculation in F instead you’d get a different % change. I think your method is wrong.

      • If you did that calculation in F instead you’d get a different % change. I think your method is wrong.

        Of course you think it’s wrong. It’s not a method. I’m trying to point out how most would quantify the natural greenhouse effect. It’s either an increase in surface temperature of 33K, or a change in radiative forcing of 150W/m^2. So, by the mid 2050s we’d expect a further increase in temperature of 1.5K – 2.5K (4.5% to 7.5% of 33K) or an increase in radiative influence of 7W/m^2 to 11W/m^2 (again 4.5% to 7.5% of 150W/m^2 – I was bit lax when I first described those numbers).

        You can argue that comparing to other numbers is better (I disagree) but I think suggesting that I’m wrong is not quite correct.

        We can take this a bit further. If you want to argue that the correct comparison is with a baseline temperature of 288K, fine. Then put it into context. A 10% shift either way (about 30K) would be utterly catastrophic. We’d either be a snowball, or we’d have wet-bulb temperatures well above those where mammals can survive and would potentially be heading for a Venus-like runaway. So, 1% to 2% of 288K may sound small, but it’s 10 – 20% of the way to utter catastrophe (I know I’ve used the word catastrophe, which is normally forbidden, so – to be clear – I’m not suggesting we’re actually heading for that, just trying to illustrate why small numbers still need context).

      • Peter,

        I may be wrong but I don’t understand why you are using 33K as the denominator.

        Because in the absence of the greenhouse effect, we’d have an effective surface temperature of 255K. With the greenhouse effect, it is 288K. The difference is 33K. The greenhouse effect does not increase temperatures from 0K to 288K.

      • Then there’s physics

        Are you a physicist?

        I am just asking so I know if I am being advised by someone who knows what they are talking about.

      • Peter,

        Are you a physicist?

        Yes.

        I am just asking so I know if I am being advised by someone who knows what they are talking about.

        Well, I’m not sure that me being a physicist means that I know what I’m talking about. In fact, I fully expect a bunch of people to pop along and tell you in no uncertain terms that I don’t :-)

      • I think I agree with them. I think you haven’t a clue about physics or how to calculate percentages.

      • Peter,
        Wow, that’s quite a response. I think 1.5/33 is indeed about 4.5%, but feel free to differ and you can do so safe in the knowledge that I have no interest in correcting you at all. Revel in your ignorance, for all I care.

      • Sorry for the response. I just don’t understand why the denominator is 33K. I haven’t seen it done this way before. I may be wrong, so can you please point me to an authoritative references that shows the calculation method is correct and commonly used?

        If you want to argue that the correct comparison is with a baseline temperature of 288K, fine. Then put it into context. A 10% shift either way (about 30K) would be utterly catastrophic.

        Where does the 10% shift (i.e. 29K temperature change) come from? I’ve never seen anyone – not even the most estreme catastrohinsts, not even the Australian Climate Change Austhority) -suggest anything like that

      • Put another way, a 2K warming is a 0.7% increase in energy in the system. This seems trivial to me – certainly not catastrophic. the climate may actually be more stable once the polar ice caps have gone – which you’d know is the normal state for the planet. In case you didn’t know there’s been no ice caps at either pole for about 75% of the time that multi-cell life has thrived on Earth

      • Sorry, Peter, I’m done. Please read my 10% point again. I wasn’t suggesting it was possible, I was simply providing some context for the 1% to 2% numers. Just because someone can make something seem small, doesn’t mean it has no significance.

      • I’m used to providing context and using ‘bookend analyses’ etc. but if you make massive exaggerations like you are doing it becomes totally meaningless and you lose any credibility you have.

      • John F Pittman

        Peter the use of 33K is common among some climate scientists. However, it use is predicated on accepting that general circulation models without CO2 and with CO2 accurately reflect the response of the earth’c lcimate system. There are those who would disagree. Both of you are in a way correct, talking past one another. What you should question in more detail is the support for 33K, since chemical reactions and physics use absolute K. In this respect you are correct. “Then there’s Physics” did state that this use of 33K was accepted. That is correct that is generally accepted by some.

      • John F Pittman,

        Thank you. Much appreciated. That’s clear.

        My apologies to ATTP.

      • John F Pittman,

        I’ve just reread your comment. I don’t think you’ve addressed the point I was questioning. I am not disputing the 33K
        due to greenhouse effect. I was asking whether it is correct to say that a 2K change in temperature due to increased greenhouse effectiveness is a 7% change. I still think it is a 0.7% change (in temperature and energy in the system). I am not yer convinced that it is correct to use the 33K as the denominator.

        Can you or anyone else please explain if the 7% changes stated by ATTP is an accepted way of stating this change. Is it stated like that in AR5?

      • Any help here? I’m trying to understand one implication of this discussion that did not seem to get addressed and I’d like to know if I am missing something. Peter pointed out that the measure of 33k would be different if you were speaking in terms of Fahrenheit rather than Centigrade and this would change the percentages. ATTP did not address that point but did say the number/approach it was commonly used. Can both those things be true?

      • John,

        Peter the use of 33K is common among some climate scientists.
        However, it use is predicated on accepting that general circulation models without CO2 and with CO2 accurately reflect the response of the earth’c lcimate system.

        This isn’t strictly correct. You can get it from the figure in the post. If you look at the thermal outgoing TOA, it is 239W/m^2. You can equate this to a blackbody temperature using 239 = sigma T^4 and you will get T = 255K. In other words, the Earth emits the same amount of energy per square metre per second as a 255K black body.

        Now consider the surface. The outgoing flux from the surface is 398W/m^2. Do the same calculation (398 = sigma T^4) and you get T = 289K. I think this is not giving 288K because we’ve had almost 1K of warming this century already, but it shows that the surface radiates as much energy per square metre per second as a 288/289K blackbody. So, this is where the 33K (or 34K now if you like) comes from. It is the difference between the effective blackbody temperature of the surface (based on the outgoing surface flux) and the effective blackbody temperature of the planet (based on the outgoing TOA flux).

        planning,
        I’m not used to working in fahrenheit, but I seem to get the same percentage in fahrenheit as I do in celsius or Kelvin.

      • ATTP said:

        planning,
        I’m not used to working in fahrenheit, but I seem to get the same percentage in fahrenheit as I do in celsius or Kelvin.

        I might be wrong on that. I didn’t check it. My mistake and misunderstanding.

      • After all this, it still seems to me that a 2K change is a 0.7% change in K and 0.7% change in energy in the system. That seems to me to be small / negligible. I am far from persuaded that warming is potentially catastrophic:

        1. It seems to me, if the planet warms so the ice at the poles disappears (over a time period of many centuries to millenia), the climate will be more stable (less temperature gradient from tropics to poles).

        2. And we know that planet’s ‘normal’ temperature (for most of the past half billion years) is about 8 K warmer than present coldhouse temperatures.

        3. And we know that life thrived when the planets was warmer and stuggled when colder.

        Consequently, I’d suggest, we should stop scaremongering – talking about catastrophe and dangerous – and focus on net costs and benefits and probabilities of: time to next abrupt climate change, direction of change, duration of change, rate of change, maximum change and most important of all the damage function.

      • Peter and ATTP – Thank you both for clearing that up.

    • A 10K increase in average temperature would then likely increase maximum wet bulb temperatures well above the threshold for mammalian life.

      Statements like this remind me of why I shouldn’t take the climate debate too seriously at times.

      • Spence and Peter,
        Well, I have to say that you’re both illustrating the tactic of “how to misrepresent what someone has said in one easy step” beautifully. Well done. You should be very proud of yourselves. I guess someone new to debate who doesn’t know how to do these things will learn quickly from your examples. It is indeed rather frustrating when someone new doesn’t realise that engaging honestly and actually reading what someone else has said is not the way to interact. So, it’s good to see you both helping to reduce that likelihood.

      • aTTP: all I did was quote, exactly, copy/pasted, a statement that you wrote. I added nothing but stunned disbelief. How on earth can that possibly be “misrepresenting” you?

        The irony, given on the previous thread you linked to a fisking of Dr Koonin’s op-ed which misrepresented his opinions wholesale, and which you happily engaged in, is not lost on me.

      • ATTP,

        Well, I have to say that you’re both illustrating the tactic of “how to misrepresent what someone has said in one easy step” beautifully.

        I didn’t not mean to represent you. If I’ve misunderstood what you were saying, I apologise. However, your comment is unhelpful. You should have at lease explained what I said and explained how I misrepresented what you’d said. If I misunderstood you (as distinct from misrepresenting you), you should give me the benefit of the doubt and try to explain another way.

      • Oh, and for what it is worth, I would agree that the manner of expression of the impact of temperature change as a percentage of kelvin is somewhat peculiar and not helpful or meaningful. If you’d have just written that and not added the nonsense about 10K rise and mammalian extinctions I would not have had a problem with what you wrote.

        On the other hand, Dr Koonin is absolutely correct to note that trying to understand the dynamics of a complex system when variation within the system is of a similar order of magnitude to the change you are trying to identify is absolutely spot on, and nobody has effectively countered that point yet.

      • Spence,
        For some reason, you left out this bit

        By the argument here, that’s only just a 3% shift, so quite small. No problem then.

        In other words, something that we probably both agree would be ridiculously large, would – by Koonin’s argument – be quite small (3% to 4% shift to the natural greenhouse effect). So, yes, you managed to somehow quote one bit and leave out the bit where I was associating what I was saying with what was being claimed in the post. I call that misrepresenting what I was saying. You can call it something else, if you wish. I stand by my point. This discussion has been a classic example of bad faith and both you and Peter seem to excel. Well done. You should be very proud. You’re both first-class Climateballers.

        I’ll add, that I actually thought that someone would do what you’ve done. Take what was intended to be an extreme example to illustrate how a small percentage of something can still be significant, and use it to suggest that somehow I was arguing that that extreme was somehow possible, or likely. I thought I’d explained it sufficiently clearly (and added sufficient caveasts) that noone would actually do so. I should have realised my naivety and remembered that the standard mantra of the online climate debate : if you can misrepresent what someone is saying to your advantage, do so.

        Anyway, I’m done. I’ve got better things to do than deal with people who can see little fault in an article like Koonins, but manage to easily misrepresent what I had thought was a fairly obvious point about how small percentages can be misleading if you don’t provide suitable context. As I’ve already said, well done. You’ve succeeded in reminding me of the many pitfalls of the online climate debate. You’d think I’d have learned by now, but I do sometimes forget. So, thanks for putting the effort in and reminding me why it’s generally pointless to engage.

      • ATTP,

        Yes your done. You make accusation that I’ve misrepresented you then run away and hide. You haven’t explained what I said that misrepresented you. You haven’t to sort it out as an honest person would do. You’ve simply make an accusation, an unsupported assertion, and then run away. You didn’t provide a link to show that your method is valid. Since you haven’t I think it is not valid and is just BS. But I am definitely keen for someone who knows what they are talking about to explain to me that it is valid. Then, if I am persuaded, I’ll acknowledge I was incorrect on that point. But you still haven’t explained how I mistreperesented you.

      • Attribution rears its ugly head. The IPCC’s pretense of attribution is a mirage, a tragic joke. What are you going to do about that, Ken?
        =================

      • aTTP: I didn’t misrepresent you. When someone tells me they adhere to scientific evidence, that gravity is an attractive force between objects with mass, and that unicorns are real, I’m not going to question the gravity thing.

        Mammals flourished under the PETM which is the best evidence we have on what happens at 8 to 10K above present. The idea that a few more degrees would be “well above” the extinction threshold is… unicorns. No matter what you believe about gravity.

        A scientist should welcome such a correction, not declare it “bad faith”.

      • Spence,
        Oh, I see. You’re arguing that 10K warmer than now would be fine for mammals. Okay. I stand corrected. I didn’t cross my mind that this is what you meant. I think you’re completely wrong, but at least I understand your point now.

      • It’s good for the whole biome, Ken, and we, poor pitiful mammals that we are, are part of the biome.

        There is more to this than physics.
        =====================

      • It may be fine for the small mammals that radiated during and after the PETM, but the cows in Texas will have problems.

        And the cows in Wisconsin will have it worse, that being the location of the continental US record wet bulb temperature.

        31 C is the highest recorded wet bulb temperature and 35 C for a few hours would be fatal for humans. Hominids evolved long after the conditions of the PETM subsided into a much cooler state.

        “”These temperatures haven’t been seen during the existence of hominids, but they did occur about 50 million years ago, and it is a legitimate possibility that the Earth could see such temperatures again,” Huber said. “If we consider these worst-case scenarios early enough, perhaps we can do something to address the risk through mitigation or new technological advancements that will allow us to adapt.”

        from here

        http://www.purdue.edu/newsroom/research/2010/100504HuberLimits.html

        NIce map here

        http://www.city-data.com/forum/weather/1391165-highest-wet-bulb-temperatures-around-world.html

      • bob droege, let me put your mind at ease, since you (and aTTP, amongst others) make a number of errors.

        Firstly, let me assure you that the Homo genus of the Hominidae family has a greater adaptability, as evidenced by a far greater geographic range of any land-based mammal, either now or during the PETM. They are also far more mobile, and can offer mobility to other mammals as well. Also remember that mammals in the PETM needed to cool, and would have been up against the same heat stress issues.

        So how do we square the circle that mammals flourished in the PETM when temperatures were 8 deg C warmer than today? The answer, of course, is relatively simple. Just because the globe is, on average, 8 deg C warmer, does not mean all temperatures are uniformly 8 deg C higher. This appears to be a basic climatology error that you and others above+below make.

        In fact, colder temperatures increase more than warmer temperatures under typical climate warming. This means:
        – min temperatures go up by more than max temperatures
        – winter temperatures go up by more than summer temperatures
        – polar temperatures go up by more than equatorial temperatures (polar amplification)

        This is exactly how mammals flourished under PETM conditions; in fact, the record maxima that you quote don’t actually go up by much. They will go up a bit, and that small increase would make (say) Texas a slightly less hospitable place to live. But on the other hand, vast parts of Canada, Alaska, Siberia, etc., etc. will become considerably more hospitable. This is seen in the PETM where polar temperature change is estimated to be ~50% higher than the global mean temperature change – which in turn, of course, means that temperatures at the lower latitudes would not have gone up by as much; and in particular, maximum temperatures even less so.

        I’m not suggesting all mammals will fare equally well, but the belief that there would be a mammalian extinction really is not well supported by the evidence we have to hand, and reasonable expectations of what might happen. Of course, this is all hypotheticals anyway – we’re not going to see 8 deg C warming any time soon.

      • And while I’m at it, I’ll point out why aTTP has failed to actually understand the point being made in the original article (as have others, such as Eli etc)

        The reason for the 1% is not to suggest 1% is a small number in terms of risk. The reason for mentioning the 1% is that in order to accurately model the global climate, your model needs to accurately capture all processes that may modify the climate at that level.

        And that is an awful lot of processes, in an awful lot of detail, all with very accurate forecasting capability.

        Of course, it is entirely reasonable to question this aspect. But aTTP didn’t really understand my point about mammalian extinctions, and I don’t think he really understood Dr Koonin’s point about the difficulty in reliably modelling the earth’s climate either.

      • Spence,

        You are making a basic error in that you conflate evolve with flourish.

        This is not what people generally associate with flourishing.

        ” Fossils of the now-extinct ground-dwelling herbivores Ectocion and Copecion from the PETM interval are reconstructed as approximately half the weight of those before and after it, and several other mammal groups that survived the PETM show the same pattern. The earliest members of the artiodactyls, perissodactyls and primates were also much smaller than their immediate descendants.”

        from here

        http://www.palaeontologyonline.com/articles/2011/the-paleocene-eocene-thermal-maximum/

        From the same source

        ” The higher concentrations of atmospheric carbon dioxide throughout the PETM decreased the nutritional value of plant material, however, leading to a temporary decrease in the size of some herbivorous insects and mammals.”

        Warm good, hot better

        h/t to George Orwell

        And look at the map I posted a link to

        Where the wet bulb temperature extremes are expected. India, Pakistan and Brazil are what you would expect, but like I said for the US it’s Wisconsin or the upper midwest and high wet bulb temperature project well into Canada.

        The PETM was 6 C in 20,000 years against 1 C in 150 years, so it all depends on your definition of anytime soon.

        Temperatures are going up faster now, at 0.1 C per decade, then they did at the beginning of the PETM.

      • …and you appear to be conflating weight with flourish.
        If that’s the case then why do you ignore the plight of lichens and mosses which cling on to survival during the short periods of relative warmth in cold climates?

      • Phatboy,

        What about the plight of lichens, they grow almost anywhere, even inside solid rock.

        Mosses could help, by absorbing the excess CO2 in the atmosphere.

        Seems a win-win to me.

        Smaller mammals means smaller pigs, which means smaller bacon, which is a lose-lose.

      • Spence,

        But aTTP didn’t really understand my point about mammalian extinctions

        Kind of true. However, this is mainly because It did not cross my mind that anyone could be suggesting that mammals would flourish if we had a 10K increase in average surface temperature. I’m still amazed. I should thank you really. You’re going to save me some time in future.

      • …not for the pigs.
        And as for the humans, it’s small bacon, really

      • ATTP, are you sure it’s going to be 10 degrees warmer?
        Or even a degree or two warmer, as opposed to a bit warmer for a few days longer of the year than at present?
        In short, are you sure that the mammals will even notice the difference?

      • bob droege –

        ==> “Smaller mammals means smaller pigs, which means smaller bacon, which is a lose-lose.”

        Now there’s a man after my own heart. Climate change may be important, but it pales in comparison next to bacon.

      • Hmm. Something has gone very badly wrong here. ATTP, for someone who complains so vehemently about misrepresenting arguments, I’m astonished that you would write this:

        It did not cross my mind that anyone could be suggesting that mammals would flourish if we had a 10K increase in average surface temperature. I’m still amazed.

        I’d love to know who said this. I certainly haven’t said this, and both you and bob seem to be very confused about what I’ve said. Perhaps I haven’t communicated it well enough, but the logic really should not have been hard to follow.

        Lets start with what I was responding to: the very specific claim that a 10K temperature hike would be well above the threshold for mammalian extinction. Note, not just above, but “well above” – whatever that means. I took it to mean that the threshold would be quite a bit below 10K, i.e. not 9K or similar.

        I called this out because I don’t think this was based on any rational claim or scientific evidence. And, indeed, I think the PETM shows this is almost certainly not the case. The evidence we have about the PETM is quite limited; we have proxy data to indicate temperatures, and we have indications of mammal populations and types from the fossil record. These data show the PETM was quite warm (circa 8 deg C above today) and that mammalian populations flourished, by which I mean populations increased, geographical range increased, and population diversity increased. These are all good indicators of evolutionary success.

        But both of you seem to have then made a basic error which possibly stems from a poor understanding of biology. I am not arguing that the warm temperatures *caused* the success of the mammals during this period. I am saying that the success shows that the mammals were not beyond any kind of extinction point, and were almost certainly not close to any kind of extinction point (since temperature would not have been constant but would have fluctuated to some degree; so we can be pretty sure they were not on a cusp).

        What I am arguing is that this shows pretty conclusively that there is good evidence to doubt that a 10K increase would breach some threshold of mammalian extinction. That does not mean I would expect mammals to flourish if the temperatures went up that high today. Unlike you, I don’t have sufficient hubris to think I can make such predictions. But what I can say, from evidence and logic, is that the idea that some specific temperature between 0 and well below 10K, that mammals will become extinct, is contradicted by the evidence we have from the fossil and proxy record.

        As for bob’s commentary, bob I am not mistaking evolution for flourishing. Evolution happens all the time, even when populations are shrinking and reducing geographic range. Also, you present much speculation as fact. It is really not correct to link one specific selective pressure to one specific phenotypic trait as you do in your post. Just like I would never argue that a single variable (temperature) was the *cause* of the success of the mammals in this period. And perhaps this is where our misunderstanding comes from – I assumed you guys would understand this automatically. Evolution of species occurs from trade-offs between multiple selective pressures and it is really an error to attempt to isolate these, and we struggle to manage that even today in the wild, where we can count and monitor populations, let alone millions of years back where we have far more limited data.

        Speculation is fun, for sure, and we all engage in it (it is a necessary part of the scientific process), but it is important to separate speculation as to causes of the mammalian success during the PETM and what we actually know – and what we know is that the success during this period eliminates the possibility that an 8K increase crosses some extinction threshold.

        Also, bob, the reason why Canada (which, from your graph has wet bulb highs of 15-25 deg C) would remain hospitable was as I described in my previous post – the colder temperatures increase by more than the warm temperatures; latitudinal variation is one part of that, but the other two examples I gave (min more than max, winter more than summer) absolutely also apply as well, which is why the corner of Canada with high summer peak WBT would not be a problem even under quite large global average increases.

      • Ah bob, I forgot about this:

        The PETM was 6 C in 20,000 years against 1 C in 150 years, so it all depends on your definition of anytime soon.

        Temperatures are going up faster now, at 0.1 C per decade, then they did at the beginning of the PETM.

        Once again, you appear to be presenting speculation as fact. I am not aware of any proxy or dating method that can pin down what happened in the PETM at the century scale or below. Most proxies of that time are of the order of hundreds of kyears, and dating methods have similar uncertainty. As such, we do not know what temperatures did at that detail at any time of the PETM; they may well have fluctuated quickly, they may not. We just don’t know. See “Nyquist” for more info.

        One of the most important things for a scientist to recognise is the limits of the knowledge we have. I do not think the claims you make here can be supported by evidence, but would be happy if you can point me to some evidence of decadal-scale temperature variation during the PETM, as I would certainly learn something new if such a thing exists.

      • Spence,
        I didn’t say this either,

        the very specific claim that a 10K temperature hike would be well above the threshold for mammalian extinction.

        I said

        A 10K increase in average temperature would then likely increase maximum wet bulb temperatures well above the threshold for mammalian life.

        Sort out your own strawmen before complaining of what others are saying. The word extinction is not present and nowhere did I imply that all mammals would go extinct, and neither did Bob. If you’d spent more than a nanosecond thinking about what I’d been saying in the first place, we wouldn’t have been having this discussion and we’d all be better off as a result. This is not a complicated concept.

      • Spence_UK http://judithcurry.com/2015/04/08/are-human-influences-on-the-climate-really-small/#comment-692147

        Excellent comment. Thank you. I find it astonishing that alarmists continually divert discussions to ridiculous extremes – like ATTP’s 10K warming – without having to provide any support for their beliefs.

        I also note your opening comment:

        ATTP, for someone who complains so vehemently about misrepresenting arguments, I’m astonished that you would write …

        ATTP also asserted that I had misrepresented him but despite me asking what I said that misrepresented him he didn’t answer – and then repeated the assertion it in a later comment in a reply to you. I’ve concluded I couldn’t trust anything he says.

      • ATTP: saying “unicorns exist” and “unicorns likely exist” will provoke equal criticism from me. I don’t care much for using “likely” as a justification for peddling junk science.

      • Peter,
        Okay, I’ll explain it. I’ll try to do it slowly and carefully because it appears that you have real trouble understanding basic concepts. With me so far?

        The premise of this post is that the anthropogenic influence is small and this is done by comparing the possible change of a few K to the current surface temperature of 288K. It’s – according to the author of the post – 1% to 2%. Following me so far? Stop me if I’m going too fast.

        My point – which I thought was obvious – is that something like a 10K increase in temperature would have a substantial impact on life on this planet. It wouldn’t make everything extinct but – as Spence points out – there would be regions where the wet-bulb temperatures exceeds that where mammals could survive (well, without technology and without burrowing underground). So, 10K is – I would argue – an extreme change. Still with me? I’ll try and do the next part slowly.

        However, by the metric used by the author of this post, a 10K change is only 3% to 4%, so quite small. However, we probably all agree that 10K is extreme and yet it’s only a few percent according to Koonin’s method. So, therefore, simply because something appears small, does not mean it would not have a measureable and significant impact.

        I’ll repeat something I said when I first made this point and which you appear to have missed or ignored (why would you do that). I’M NOT SUGGESTING IT IS NECESSARILY POSSIBLE, I’M TRYING TO SHOW HOW EVEN SOMETHING EXTREME CAN BE MADE TO SEEM SMALL IF YOU COMPARE IT WITH SOMETHING THAT HAPPENS TO BE BIG BY COMPARISON.

        Okay, have you got it now? Apologies for being a bit condescending, but sometimes it’s hard not to be, especially with people who appear to suggest that they have some knowledge of this subject when they clearly do not and also appear to suggest that they can understand basic and simple concepts, when they clearly do not. Oh, and it might help if you behaved less like an unpleasant prat, but maybe that’s just me.

      • Spence_UK

        Thank you for your excellent, interesting and informative comments.

        Re: your reply to Bob,

        I am not aware of any proxy or dating method that can pin down what happened in the PETM at the century scale or below. Most proxies of that time are of the order of hundreds of kyears, and dating methods have similar uncertainty. As such, we do not know what temperatures did at that detail at any time of the PETM; they may well have fluctuated quickly, they may not. We just don’t know. See “Nyquist” for more info.

        I agree 100%. I’ve frequently seen people stating that the rate of temperature increase over the past decades or past century is unsurpassed. However, they don’t have any evidence for that. It’s a baseless statement. In fact there seems to be good paleo evidence for rapid, very large temperature rises. And guess what – life thrived during the rapid warmings. Here’s an excellent example from Ireland: http://eprints.maynoothuniversity.ie/1983/1/McCarron.pdf
        See Figure 15.21 and text.

        Points to note:
        • Abrupt changes with rapid warming occurred frequently
        • Two such warmings occurred 14,500 and 11,500 years ago
        • These warmed from near glacial conditions to near current conditions in7 years and 9 years respectively
        • text and other figures indicate life loved warming

      • Peter, thanks for the kind words. Unfortunately on such a polemic and emotive subject, online debates often end with considerable misunderstandings and disagreements. No one party is guilty of this, but it is frustrating that ATTP seems to spend more time pointing fingers than actually trying to resolve the misunderstandings.

        Although my commentary tends to be quite sharp which probably doesn’t help either. I get frustrated easily, and I get the feeling ATTP is probably similar; I suspect these discussion would be far more cordial and constructive over a coffee in person than over the internet.

      • Spence,

        will provoke equal criticism from me. I don’t care much for using “likely” as a justification for peddling junk science.

        And you think that a 10K increase isn’t likely to increase the wet-bulb temperature above the threshold for mammallian life? That’s absurd, especially as you appear to have agreed that it would.

        Okay, I’ll explain the sentence construction to you. The likely referred to what would happen if the temperature rose by 10K. It didn’t referred to the likelihood of this temperature change actually happening. If I’d wanted to say that, the words would have been in a different order. This is not complicated.

        Here’s a couple of points. Firstly, I only have to defend what I actually say. I don’t need to defend what you think I said, or what you think I meant. I can’t help it if you can’t understand what a basic sentence in English is intended to mean.

        A second point. When you write some kind of rebuttal to what someone else has said. Read what you’ve written. Then go back and read what the other person has said. Then consider the possibility that what you’ve written as a rebuttal isn’t actually inconsistent with what the first person has said. Having a discussion with someone who responds to what you’ve said with something that is essentially consistent with what you said in the first place is both tedious and irritating. A complete and utter waste of time, but then that’s no great surprise.

      • ATTP,

        I stopped at this:

        Okay, I’ll explain it. I’ll try to do it slowly and carefully because it appears that you have real trouble understanding basic concepts. With me so far?

        That’s sufficient to demonstrate, on top of your previous comments, you are arrogant, abusive and not to be taken seriously. Your continually refrain that others are misrepresenting you when, in fact it is you who is doing the misrepresentation, demonstrates you are intellectually dishonest: http://judithcurry.com/2013/04/20/10-signs-of-intellectual-honesty/

      • By the way, ATTP. I scanned your comment for a quote of what I said that you claimed misrepresented you. I didn’t see any quote of what I’d said, so clearly you have not addressed my question, which was: what did I say that misrepresented what you said? It’s a pretty simple question. you dodged it – further confirming you lack intellectual integrity.

      • Peter,

        further confirming you lack intellectual integrity.

        Okay, why don’t you confirm your intellectual integrity by illustrating why what I said was an exaggeration. You made the claim not me. I don’t have to defend your characterisations of me, you should be willing defend your own claims. Of course, I don’t specifically care as all this exchange is illustrating is why engaging in such discussions is both pointless and a waste of time and energy.

      • Peter,
        I missed your earlier comment. You should consider the possibility that me trying to annoy you isn’t intellectually dishonest. If anything, it’s the reverse, in that I’m telling you precisely what I think. It’s odd that people call me dishonest when I pretty much say exactly what I think. It’s almost as if they would rather I lied about my views.

      • ATTP,

        You said I had misrepresented you. I asked you several times to say what I’d said that had misrepresented you. You continually dodged the question (even said “I am out of here” or words to that effect). Clearly you made an assertion and couldn’t defend it. You have repeatedly demonstrated intellectual dishonesty. I wouldn’t trust you on any statement you make. It’s too late now for you to attempt to recover. Your just digging a deeper.

      • Peter,
        Okay, this has got out of hand, so I’ll try and tone it down. Here’s the issue. I know 10K is extreme; an exaggeration. That was the point. I was trying to show how something extreme can seem small if you compare it to something that happens to be large by comparison. Therefore, it is extremely irritating to then have that interpeted as a typical alarmist exaggeration. I even added that I knew I had said something extreme and that I wasn’t arguing that it was possible.

        This whole exchange just seems typical. Someone like me can’t even say something like that, with all the caveats attached, without someone immediately interpreting it as an alarmist exaggeration. If I seem annoyed that’s why. Apologies for the tone of my lengthier response to you.

      • You have repeatedly demonstrated intellectual dishonesty.

        Blast, now you’re making me regret my most recent comment.

      • Spence_UK,

        I meant to include the caption for Figure 15.21 I referred to in my previous comment:

        Figure 15.21 The stable isotope record (∂18O) from the GRIP ice core (histogram) compared to the record of N.pachyderma a
        planktonic foraminiferan whose presence indicates cold sea temperatures) from ocean sediments (dotted line). High concentrations
        of IRD from the Troll 8903 core are marked with arrows. After Haflidason et al. (1995). The transition times for critical
        lengths of the core were calculated from the sediment accumulation rates by the authors and these gave the following results:
        Transition A: 9 years; Transition B: 25 years; and Transition C: 7 years. Such rapid transitions have been corroborated from the
        recent NGRIP ice core data.

        My previous comment said:

        “… there seems to be good paleo evidence for rapid, very large temperature rises. And guess what – life thrived during the rapid warmings. Here’s an excellent example from Ireland: http://eprints.maynoothuniversity.ie/1983/1/McCarron.pdf
        See Figure 15.21 and text.

        Points to note:
        • Abrupt changes with rapid warming occurred frequently
        • Two such warmings occurred 14,500 and 11,500 years ago
        • These warmed from near glacial conditions to near current conditions in 7 years and 9 years respectively
        • text and other figures indicate life loved warming”

      • Wait, what opinion are you ascribing to me that I do not hold now?

        And you think that a 10K increase isn’t likely to increase the wet-bulb temperature above the threshold for mammallian life? That’s absurd, especially as you appear to have agreed that it would.

        You can’t help yourself, can you? I said I did not know what impact such a thing would have, because I do not have your strange hubris to think you would be able to predict such a thing. There is no reason to believe an extinction would be “likely” though.

        Well, let me see. Sure, a uniform 10K increase to the max WBT globally, no matter how daft such a suggestion is, would probably render Singapore uninhabitable. But Siberia would probably be still a bit cold in winter, but quite pleasant in the summer. I checked Scott Base at Ross Island in Antarctica, I was too lazy to try and find the max wet bulb temp, but the max dry bulb temp each summer is around -1.2 deg C. The wet bulb temp is, by definition, less than this. I would suggest that an increase of 10K in max wet bulb temperature would still mean the Antarctic was too cold to be comfortable to live.

        So no, I see no reason why a uniform increase of 10K in WBT would result in mammalian extinction, as there would still be plenty of land mass to live on. But no, I don’t know what would actually happen; and anyone who claims they would unambiguously know (extinction or otherwise) is deluding themselves.

        Also: you misrepresent Koonin’s position. He didn’t say 1% meant it was small in the context of not dangerous. He said it was 1% of absolute temperature which makes it hard to model, since you would probably need to accurately model all natural processes that are of this order of magnitude, which is difficult. I actually think he underestimates this problem. Your straw man is not only wrong but is a misrepresentation of Koonin’s original claim. For someone who gets so upset about misrepresenting arguments, you sure do it a lot.

      • Spence,
        Let’s see if we can do something really simple. Why did you say this?

        There is no reason to believe an extinction would be “likely” though.

        I haven’t said it was likely. I haven’t even mentioned extinction other than to point out that you used it. So, why do you keep referring to the possibility of an extinction? My impression is that you think I said an extinction was likely, but I very clearly did not. Can you at least admit this and can we see if we can at least resolve this in an attempt to get something non-negative out of this exchange?

        Let’s also remind ourselves that this entire exchange appears to relate to a single sentence I wrote yesterday.

        A 10K increase in average temperature would then likely increase maximum wet bulb temperatures well above the threshold for mammalian life.

        (note: The word extinction does not appear and the word likely refers to the possibility of the maximum wet bulb temperature exceeding the threshold for mammallian life, not to the possibility of a 10K rise in temperature) Let’s also bear in mind that I don’t specifically disagree with much of what you’ve said about WB temperatures (well, apart from the bits where you vere making claims about what I may, or may not, have done, and my own take might be somewhat more negative than yours). As far as I can tell, this entire exchange is based on you reading a single sentence of mine, and interpreting it in a manner that is entirely inconsistent with the words I used, in the order that they were used. Any interest in resolving this? My guess is no, but feel free to prove me wrong.

      • Wait a second, Spence was the one that brought up the 8-10 C warmer during the PETM, not us warmists.

        Yes you can get rapid temperature changes like the 7, 9 and 25 year events Lang has cited, but those happened at the same time that some major ice sheets were collapsing, might those events have had something to do with the rapid climate change?

        And I am aware that the increase in temperature during the PETM was not uniform, but still the tropics increased by 3 C and the mid latitudes by 5 and the poles a little more, but that was compared to temps previous to the PETM, not to temperatures now.

        ” I mean populations increased”

        You can’t get population density from the fossil record, as far as I know, maybe you have a cite to support this.

        ” very specific claim that a 10K temperature hike would be well above the threshold for mammalian extinction.”

        I certainly never mentioned extinction, only that cows in Texas would have problems. They would probably move, or be moved north.

        I did note that there is evidence that mammals evolved to smaller sizes in response to the PETM, whether that can be associated with any extinctions is not supported by any evidence.

        But there is evidence that increased CO2 leads to less nutritious plants.

        And the weekend that the wet bulb temperature hit 31 in Wisconsin was associated with a large number of heat related deaths.

    • The actual statement was

      shift the atmosphere’s natural greenhouse effect by only 1% to 2%.

      Actually, you quoted out of context. The actual statement was:

      human additions to carbon dioxide in the atmosphere by the middle of the 21st century are expected to directly shift the atmosphere’s natural greenhouse effect by only 1% to 2%.

      IOW, the ΔCO2 makes a “1% to 2%” difference to the total effect of all GHG’s, including water vapor.

      The whole argument strikes me as a sort of “angels on the head of a pin” thing, given the hyper-complex non-linear nature of the system, but if you must debate that point, consider that he’s excluding water vapor “feedback” in his “directly shift”, but including it in “the atmosphere’s natural greenhouse effect”.

      Is this warranted? I’d have to say so, intuitively. There’s certainly no reason to assume a linear relationship between the “feedback” from water vapor to a small incremental change in CO2, and the total integral of “feedback” from water vapor over the difference between no CO2 and present (or pre-industrial) values.

      And, should it really be about temperature, rather than actual amount of radiative “forcing”?

      Especially since the whole effect of “feedback” from water vapor, and especially clouds, is at the core of the debate.

      OTOH, any op-ed in the WSJ would be mostly pointed at people who don’t understand those issues. IMO.

      • AK,
        I’m happy to accept your context because I don’t see the difference. I wasn’t trying to misquote. From the post written here by Steve Koonin he appears to be determining the percentages relative to either 288K or relative to a downward flux of 503W/m^2. So, I don’t think we disagree about the amount of change. What I think we disagree about is whether or not it is appropriate to measure this change relative to a 0K Earth, rather than relative to an Earth without a greenhouse effect.

        IOW, the ΔCO2 makes a “1% to 2%” difference to the total effect of all GHG’s, including water vapor.

        I know some have tried to make this argument, but it’s not clear that this is what this post is suggesting. Furthermore, why compare only Delta CO2 with the greenhouse effect, when the greenhouse effect is a combination of CO2, plus cloulds, plus water vapour, etc? It would not seem to be a like-for-like comparison then. There should be similar feedbacks to Delta CO2 as there are in the greenhouse effect itself.

        Plus, in the very same paragraph, there is a comparison with natural variability. Why compare the effect of Delta CO2 with natural variability, when what is of interest is the total anthropogenic response relative to natural variability, not just the response to Delta CO2 alone?

        And, should it really be about temperature, rather than actual amount of radiative “forcing”?

        It should not really make a difference. The greenhouse effect can either be regarded as increasing the effective surface temperature by 33K, or producing a net change in radiative forcing of 120 W/m^2 (or sometimes taken to be 150W/m^2 depending on how you measure it).

      • All that junk is linear. Well, yeah, in the models.
        ==========

      • There should be similar feedbacks to Delta CO2 as there are in the greenhouse effect itself.

        Did you read my whole comment?

        There’s certainly no reason to assume a linear relationship between the “feedback” from water vapor to a small incremental change in CO2, and the total integral of “feedback” from water vapor over the difference between no CO2 and present (or pre-industrial) values.

        Especially since the whole effect of “feedback” from water vapor, and especially clouds, is at the core of the debate.

      • Did you read my whole comment?

        Yes, but am not sure what that has to do with my point. Comparing the natural greenhouse effect with Delta CO2 only, doesn’t really make much sense to me. Whatever your views on feedbacks, we have expected changes by 2050 that include forcings and feedbacks. You may disagree with them, but they exist. So, if Koonin really did just mean CO2 only, then I think that was unclear and – IMO – doesn’t make much sense (why ignore feedbacks in a claim about the anthropogenic influence by 2050). On the other hand if – as this post – suggests, the comparison was made with a 0K Earth, rather than a 255K Earth, again this an obvious way to quantify the effect. TBH, it all seems rather ambiguous to me and what seems clear is that the argument is that the effect is quantitatively small without any attempt to put “small” into any kind of actual context. You are free to disagree, of course.

      • Comparing the natural greenhouse effect with Delta CO2 only, doesn’t really make much sense to me.

        Well, given that the actual magnitude of “feedbacks” is unknown, it makes an excellent starting point. It’s also, AFAICT, the exact meaning of his words, no matter what he meant to say.

        Whatever your views on feedbacks, we have expected changes by 2050 that include forcings and feedbacks.

        Do we? Well, nobody knows about cloud feedbacks, and especialy aerosols via clouds (both natural due to climate change, and anthropogenic).

        Let’s take a look at straight water vapor: According to this NASA website

        Specifically, the team found that if Earth warms 1.8 degrees Fahrenheit, the associated increase in water vapor will trap an extra 2 Watts of energy per square meter (about 11 square feet).

        So, assume about 1°C from “bare” CO2. (1.8°F.) Add the extra “2 Watts of energy per square meter” to the “bare” 3.7, and divide by 342 (total thermal down from chart in main post). Answer: 1.666666666666667% (rounded). That’s still in the range. Even if you allow a little for positive cloud feedback, it’s still within “only 1% to 2%.

      • AK,
        If you want to argue that there is a way to construct an argument that makes what Koonin said correct, fine; I don’t actually dispute that. I just think it’s not very clear and minimises the issue and that the comparison is being done with quantities that are not normally used to quantify the natural greenhouse effect. We can choose to disagree.

        I’ll comment on this, though.

        Specifically, the team found that if Earth warms 1.8 degrees Fahrenheit, the associated increase in water vapor will trap an extra 2 Watts of energy per square meter (about 11 square feet).

        Feedbacks, feedback on themselves. I.8 degrees Fahrenheit is – I think – 1K. So what the above is saying is that the feedback response is 2W/m^2/K. A 1K increase in temperature is associated with a radiative forcing of about 3.1W/m^2 (1.2K is associated with about 3.7W/m^2). So the feedback fraction is 2/3.1 = 0.65. If you want to etimate the total effect of the feedbacks, you need to complete the infinite sum

        1 + f + f^2 + f^3 + ….. = 1/(1-f)

        which for f = 0.65 is 2.8. So, feedbacks of 2W/m^2/K would amplify a 1K warming by a factor of about 2.8. Of course, that would be the equilibrium response, not the transient response, which would be smaller.

      • If you want to argue that there is a way to construct an argument that makes what Koonin said correct, fine; I don’t actually dispute that.

        He was writing rhetoric (actually the written equivalent). All he needs to justify an argument is for it to be scientifically correct. Since scientists themselves disagree on how warranted any particular scientific analogy is, his argument is just as good as those of people who think AGW is a big issue.

    • David Springer

      503W/m2 is a radiant temperature of 307K not 288K.

      You and Koonin are both wrong. Earth’s average surface temperature in that chart is implied by thermal radiance of 398W/m2 which is blackbody temperature of 288K.

      The so called greenhouse effect is the difference between thermal illumination of 342W/m2 and thermal emission of 398W/m2 or 56W/m2.

      Absent greenhouse gases (predominantly H2O not CO2 by the way) there would be no thermal illumination and thermal emission would be the sum of atmospheric and surface absorption of solar shortwave (161 + 79) 240W/m2 which equates to a blackbody temperature of 255K.

      See how that all works out to give a 33K greenhouse effect from 56W/m2 net thermal illumination from greenhouse gases?

      The additional thermal illumination caused by CO2 doubling is given to be 3.7W/m2 or about 6.6% increase in greenhouse effect (3.7 divided by 56).

      If Koonin was the MOST atmospheric physics savvy person working on APA climate statement that’s a real problem. I really big problem given the incompetent explanation offered for the 1% – 2% increase in greenhouse effect.

      I’m stunned.

      • Actually, if you divide 3.7W/m^2 (GH effect of added CO2 without “feedbacks”) by 342W/m^2 (total thermal down from chart) you get 1.08187134502924% (roughly). So he’s got his numbers right, although how warranted he is in using that ratio is up for debate.

      • AK,
        that’s all i understood as “percentish”. no more, no less

      • Solar absorbed by the atmosphere 79 and the surface 161 roughly equal TOA outgoing 239. Fair enough.
        Outgoing 239 equals then equals down-welling and sets the base line for the surface, which is then warmed by an additional 161 (or 159) solar, making the 398W/m2.

      • correction.. Outgoing 239 then equals down-welling 239 and sets the base line for the surface… etc

      • David Springer

        AK the greenhouse effect is from NET thermal illumination not total thermal illumination. NET thermal is 56W/m2. That is totality of greenhouse effect and raises the radiant temperature from 255K to 288K.

        It’s easy enough to plug in 255K and 288K into blackbody calculator and see the difference in radiant emission between the two temperatures is 56W/m2.

        CO2 doubling ostensibly raises the NET thermal illumination by 3.7W/m2. These are simple calculations done with online blackbody calculator I referenced previously.

        The numbers work out exactly every which way but loose. Blackbody theory is not in question nor is greenhouse effect. What’s in question is feedbacks which can either amplify or negate the 3.7W forcing from CO2 doubling and the feedbacks appear to be a moving target that may change under the influence or confluence of many other factors such ocean overturning rate, size and duration of north polar sea ice, soot and particulates both manmade, volcanic, and kicked up by winds in long term drought areas, influenced by cosmic ray strength modulate through changing solar magnetic field, and God only knows how much else.

        Anyhow, with regard to basic blackbody calculations for solar energy in and thermal energy out, I can lead a horse to water but I can’t make him drink. If you accept what I wrote you’ll be correct and if you reject it you’re wrong. It’s not debatable except by physics illiterates who don’t have the first smidgen of understanding of how electromagnetic radiation and matter interact. Given ATTP is ostensibly versed in astrophysics if he disagrees with anything I wrote in regard to blackbody radiation above he’s either lying or purchased his degree without passing the classes. I think his degree is from someplace in Africa so I’m not ruling out the second option that it was bought instead of earned.

      • Leaving other factors aside, if the surface is absorbing 161 (or 159) solar, it would be absorbing another 239 thermal to make 398W/m2?

      • I think his degree is from someplace in Africa so I’m not ruling out the second option that it was bought instead of earned.

        Just thought I would highlight this as it probably deserves due diligence.

      • Which calculator do you use?
        I’ve tried one or two online calculators as well as my own, and I get 390.11 – 239.76 = 150.35W/m2.

      • David Springer

        @phatboy

        Mibad. Good catch. Net thermal is 398W/m2 upwelling from surface surface minus 239W/m2 leaving at top of atmosphere or 159W/m2 greenhouse effect. Percentage change from doubling CO2 is then 3.7/159 or 2.3%. Not much above Koonin’s upper bound of 2%.

        What screws this up some is Planck feedback i.e. radiant emission goes up as the 4th power of temperature. Check this by calculating radiant emission increase from 255K to 265K (40W) then compare with 275K to 285K (50W).

        The increasing power required to raise temperature by another degree is called Planck Feedback, which is negative, and is ostensibly what prevents a runaway greenhouse.

        Not also that at 288K it takes 5.4W/m2 to raise the temperature to 289K so a CO2 doubling at 3.7W absent positive feedbacks can only raise radiant temperature by 0.7C.

      • Given ATTP is ostensibly versed in astrophysics if he disagrees with anything I wrote in regard to blackbody radiation above he’s either lying or purchased his degree without passing the classes.

        Care to walk this one back. I would have pointed out the same issue that phatboy did, but chose not to since – according to you – disagreeing with anything you wrote would imply that I was lying or had purchased my degree.

      • David Springer

        This is interesting. I was checking to see how Planck Feedback is handled in models and found this:

        http://judithcurry.com/2011/10/16/feedback-in-climate/

        In models, this is actually calculated with a full radiative transfer code for each gridpoint on the Earth; this so-called “Planck feedback” amounts to about an extra 3.2 Watts per square meter (W/m2) power emitted for every 1 K temperature increase (Soden and Held, 2006). Climate sensitivity is often taken as the ratio of the temperature change to the radiative forcing; so for example, a doubling of CO2 which constitutes a forcing of ~3.7 W/m2 would produce a temperature response of 3.7/3.2 = 1.2 K.

        Does not compute. The average temperature of the earth is given at 288K which is a radiant emission of 390.1W/m2 according to blackbody calculator here: http://spectralcalc.com/blackbody_calculator/blackbody.php

        At 289K the radiant emission goes up to 395.6W/m2. Thus if CO2 doubling produces 3.7W/m2 then the blackbody temperature response is 3.7/5.4 = 0.7K not 1.2K.

        What’s up with that?

        This alone would explain why models are running hotter than observations but it’s hard to conceive of such a basic error in climate model as due to Planck Feedback being too low.

        A temperature increase of 240K to 241K takes 3.2W/m2. I have no idea how that 3.2W/m2 per 1K temperature increase is justified given earth average temperature of 288K.

      • This number probably comes from using 255 K as the base temperature. The top-of-atmosphere emission temperature has to increase by 1.2 C to counter the reduction caused by CO2. This translates to the surface because nothing fundamental happens to the lapse rate, but they account for the lapse rate change when considering water vapor feedbacks.

      • David,
        Come on! The forcings are computed at the TOA. So, yes an increase of 1K at the surface produces a change in surface flux of 5.4W/m^2, but a change in TOA flux of 3.2W/m^2. If you didn’t spend all your time claiming that everyone else had made some kind of stupid mistake, and instead considered that they hadn’t and that maybe you don’t quite understand this as well as you claim, you may even be worth discussing this with.

        Any chance of walking back your earlier confident claim? Okay, I’m being silly – of course not, but worth asking, I guess.

      • 255 K gives you 3.7 W/m2/K, so I don’t know where 3.2 comes from. Anyway 1 C per doubling is an often used number for this, and it seems about right.

      • JimD,
        I think it comes from the GHE. The GHE can be thought of as the atmosphere having an emissivity of 255^4/288^4 = 0.61. So, a 1K increase in surface temperature produces an increase in surface flux of 5.4W/m^2. Multiply that by 0.61 and you get 3.3W/m^2. Not quite 3.2W/m^2, but I think that is roughly the issue. Essentially, a 1K increase in surface temperature doesn’t produce a 1K increase in the temperature at TOA.

      • ATTP, ah, yes, thanks. The formula is 4*epsilon*Ts^3, and epsilon, as you calculate it, is used for the whole atmosphere to translate from T at TOA to Ts.

      • David Springer

        @ATTP

        TOA thermal emission is the sum of solar energy absorbed by atmosphere and absorbed by surface. At equilibrium energy-in must equal energy-out. The only place energy enters the system is from the sun. Greenhouse gases don’t create energy. Write that down.

        The greenhouse effect increases temperature in the troposphere and decreases it in the stratosphere. TOA energy in/out at equilibrium is unaffected by greenhouse gases. A slight imbalance of 0.6W/m2 believed to exist today between TOA energy in and out is due to the difference between transient and equilibrium response. In other words 0.6W/m2 is “in the pipeline”.

        You really don’t know very much about this subject ATTP and seem to have no comprehension at all of the law of conservation of energy which prohibits TOA emission at equilibrium from exceeding the energy supplied by the sun. The greenhouse effect doesn’t add energy it rearranges its distribution in the atmospheric column.

        You do not seem to understand that greenhouse gases, being better absorbers of infrared are also better emitters of infrared. Kirchoff’s law ring any bells to you? Surely you heard of that law in your PhD physics program, right? This results in greater radiative resistance through the troposphere and lesser radiative resistance through the stratosphere i.e. warming of troposphere and cooling of stratosphere.

      • David Springer

        @ATTP

        TOA thermal emission is the sum of solar energy absorbed by atmosphere and absorbed by surface. At equilibrium energy-in must equal energy-out. The only place energy enters the system is from the sun. Greenhouse gases don’t create energy. Write that down.

        The greenhouse effect increases temperature in the troposphere and decreases it in the stratosphere. TOA energy in/out at equilibrium is unaffected by greenhouse gases. A slight imbalance of 0.6W/m2 believed to exist today between TOA energy in and out is due to the difference between transient and equilibrium response. In other words 0.6W/m2 is “in the pipeline”.

        You really don’t know very much about this subject ATTP and seem to have no comprehension at all of the law of conservation of energy which prohibits TOA emission at equilibrium from exceeding the energy supplied by the sun. The greenhouse effect doesn’t add energy it rearranges its distribution in the atmospheric column.

        You do not seem to understand that greenhouse gases, being better absorbers of infrared are also better emitters of infrared. Kirchoff’s law ring any bells to you? Surely you studied Kirchoff’s Law in your PhD physics program, right? This results in greater radiative resistance through the troposphere and lesser radiative resistance through the stratosphere i.e. warming of troposphere and cooling of stratosphere.

      • David Springer said:
        “Net thermal is 398W/m2 upwelling from surface surface minus 239W/m2 leaving at top of atmosphere or 159W/m2 greenhouse effect.”

        159 solar.

      • David,
        Energy conservation, Kirchoff’s Laws…hmm, maybe you could explain those to me. Sounds fascinating. At the same time, maybe you could explain why your most recent comment has anything to do with what I said. I’m somewhat failing to get it. It seems like you’re playing that “I’ll say a bunch of things that are right so as to make it seem that what the other person said is wrong”. Tedious and boring, but no great surprise. Ohh, and the Planck response is 3.2W/m^2/K.

      • ulriclyons – in the article cartoon, SW from the sun (340) minus SW reflected (100) minus SW absorbed by atmosphere (79) equals SW absorbed by the surface. It goes out as 56 LW, 84 LH and 20 SH, which is an implied surface imbalance of 1 (.6).

        Which, according to the profoundly landmark skeptic darling, is drilling .6 into the southern ocean, which is being found by the ARGOnuts though it may not be happening.

        Nobody ever mentions this, but the Stephens et al diagram has a different result at the stop of the big blue yonder.

      • However it gets out, 159 solar requires an extra 239 thermal to reach 398.

      • 161 absorbed by surface plus 79 absorbed by atmosphere, 240 SW absorbed, is what has to get out.

        239 is what getting out.

        That why they call it an energy imbalance.

      • David Springer said:
        “At 289K the radiant emission goes up to 395.6W/m2. Thus if CO2 doubling produces 3.7W/m2 then the blackbody temperature response is 3.7/5.4 = 0.7K not 1.2K.”

        That looks good though, 0.68K.

      • Yes, I’ve met cabdrivers who can dissimulate even better than he does.
        ==============

      • Judith,
        You have some really classy commenters here. You must be proud?

      • Off with their heads.
        ==============

      • David Springer

        …and Then There’s Physics | April 11, 2015 at 10:19 am |

        “If you didn’t spend all your time claiming that everyone else had made some kind of stupid mistake”

        You’re in denial. The models are running hot. Rate of CO2 increase in atmosphere accelerated for all of past 20 years and global average temperature remained flat.

        The stupid mistake made by global warming prognosticators is a fact demonstrated by reality. We’re simply trying to figure out where the rubes went wrong.

      • David Springer

        …and Then There’s Physics | April 12, 2015 at 3:02 pm |
        Judith,
        You have some really classy commenters here. You must be proud?
        =================================================

        Yeah but even Judith has standards. She kicked out B B D and W e b H u b T e l e s c o p e who are now two of your most prolific contributors. You must be proud.

        ROFL

  21. Over longer timeframes 1% changes can actually have very big effects (in systems with storage effects especially).

    The more pertinent things seem to me:

    -“First, it means precision observations are required to see the climate response.” and
    -“And finally, because life at the 1% level is rich, the models have to get many small phenomena right to confidently isolate and project the response to anthropogenic effects.”

    -you have to have very precise measurements of independent variables (your data input) even if you have valid models.

    -structural model quality is very important of course because of complex nonlinear feedbacks (the idea seems to be that errors cancel themselves out, but thats valid for linear systems only).

    Hard to see how the current state of science can yield any robust predictions.

    Which leaves us with the precautionary principle.

    • well life through evolution will be adapted to changes that occurred in the recent past.

      But I wonder where in the recent past global temperature has even risen 2 degrees C in a matter of centuries.

  22. “I tend to think we are reducing the risk of the next abrupt cooling event and, since this would be far more damaging that an abrupt warming event, we are actually, inadvertently, reducing rather than increasing the risk of climate change.”

    Unless of course the next abrupt cooling event happens after CO2 drops back down again. In which case it will fall from much higher with more damage.

  23. Thanks Dr Koonin for expansion of your comment. Too bad you couldn’t continue to contribute to moderate the activists. They control the run away train
    Scott

  24. Pingback: Steve Koonin and the small percentage fallacy | …and Then There's Physics

  25. David Springer

    “the total downward flux on the Earth’s surface estimated as 503 ± 7 W/m2 (161 W/m2 solar + 342 W/m2 thermal aka “Greenhouse effect”)

    Total hogwash.

    A black body illuminated by 503W/m2 is warmed to 307 Kelvin (34C). The hottest mean annual temperature anywhere on the planet is only 34C. Death Valley, California is only 25C.

    You can check the math here:

    http://www.spectralcalc.com/blackbody_calculator/blackbody.php

    Plug in 307K for temperature on the left, press the calculate button, and note the resulting radiant emittance is 503W/m2.

    • David,
      Just a hint. Consider how much energy leaves the surface via convection and evaporation. It’s not zero.

      • Just consider how energy is tranferred by photons. They react with electrons, in general. Convection, evaporation – so 20th century, isn’t it?

        Just a hint – get with it. Physics has moved on – maybe you haven’t.

        Try it – you might like it!

      • David,
        Is that you again?

      • Nope. You do realise how energy is transferred, do you, being a physicist and all?

        Maybe you would care to educate us all, and describe how convection and evaporation actually work, using up to date physical terms.

        More actual physics, and less snark, might help.

      • More actual physics, and less snark, might help.

        I seriously doubt it.

      • Heh, and Then There’s Only Radiative Physics.
        ===================

      • Heh, and Then There’s Only Radiative Physics.

        Actually, my point was that there isn’t only radiative physics. The surface energy balance is downwelling solar + downwelling long wavelength, balanced by outgoing longwavelength + convection (or sensible heat) + evaporation.

      • and Then There’s Physics: Consider how much energy leaves the surface via convection and evaporation. It’s not zero.

        That is what I have been writing about. Changes in the rates of those processes impose a thermodynamic “constraint” on how much surface warming can result from an increase in atmospheric CO2 concentration. O’Gorman et al wrote of energetic constraints on precipitation, but I reversed the approach and looked at how estimates of precipitation change impose energetic constraints on surface warming.

      • How can it do that?

      • David Springer

        convection and evaporation results in condensation and precipitation – the atmosphere is heated by rain – the energy stays in the troposphere – what’s you’re point?

      • David Springer: the energy stays in the troposphere

        From the troposphere, the energy is radiated to space.

      • David Springer

        Matthew, the only way energy leaves the earth system is via radiation. That’s a given. Evaporation and convection carries energy aloft but it is returned to the troposphere via condensation and precipitation. It must of course eventually leave the troposphere through radiation as that’s the only way it can happen but the evaporation/condensation cycle in and of itself does not cool the atmosphere it cools the earth’s surface. This changes the lapse rate in the troposphere, a negative feedback, which reduces surface temperature while raising temperature aloft where adiabatic cooling reaches the dewpoint. This is known as lapse rate feedback and, like many other details in global climate, is not well known through observation, and gives rise to the notorious “GHG fingerprint” of greater warming aloft than at the surface.

      • David Springer: It must of course eventually leave the troposphere through radiation as that’s the only way it can happen but the evaporation/condensation cycle in and of itself does not cool the atmosphere it cools the earth’s surface.

        That is what I have written elsewhere. I can’t see where you think I wrote something that contradicts that in this thread.

      • It is funny that everybody seems to think there is some basic thing Matthew is missing, which may be, but when I read him there are no basic things he is missing. He seems to be asking about a calculation in the future – after some amount of warming – to do will constraining LH/SH, and the effect of that on surface warming/cooling.

      • When CO2 is doubled, the earth has to figure out how to radiate an extra 3.7 W/m2 to space. The most obvious way is by warming from the surface through the atmosphere. If you can think of a way adding evaporation helps it to emit this extra heat you need to say it. Generally adding water vapor to the atmosphere is a hindrance to restoring the balance, not a help, because it is also a GHG. The more you add, the warmer it needs to get to restore the balance.

      • When CO2 is doubled, the earth has to figure out how to radiate an extra 3.7 W/m2 to space. The most obvious way is by warming from the surface through the atmosphere. If you can think of a way adding evaporation helps it to emit this extra heat you need to say it.

        Straw man. Another way of radiating an extra 3.7 W/m^2 is by reflecting more from more clouds. Which more evaporation might do.

      • matthewmarler, “That is what I have been writing about. Changes in the rates of those processes impose a thermodynamic “constraint” on how much surface warming can result from an increase in atmospheric CO2 concentration.”

        Probably the easiest way to estimate the “constraints” is by considering DWLR estimates that are already constrained. The estimated 340 Wm-2 of DWLR at the “average” surface is the result of solar absorbed, latent and sensible transfer and current radiative forcing. 3.7 Wm-2 of additional forcing is roughly a 1 percent increase. That DWLR forcing is “constrained” by the average energy of the oceans of roughly 4 C degrees (334.5 Wm-2 via S-B). Should ocean heat uptake stop, the average ocean energy should mirror average DWLR less some small “atmospheric window” loss.

        The atmospheric window as seen from the ocean surface by the way is much smaller than the published 40Wm-2. “Globally” from the surface the window is roughly 20 Wm-2 and most of that is from land areas. If you remember the Kimoto paper link I gave you, he used Kiehl and Trenberth’s energy budget which had a number of difference from the Stephen’s et al budget. Using the Stephens et al budget you get a different number for the Planck feedback parameter of about 0.8 C per 3.7 Wm-2. That isn’t a perfect estimate by any means, but since DWLR already includes amplification at our current somewhat steady state, it is more accurate than some of the other guesses.

      • AK,
        I don’t think you understand what a strawman is.

        Yes, you’re right that we could retain energy balance simply by changing our albedo through changes to clouds. Any evidence that that is likely or has happened in the past? I think the answer is no, but feel free to prove me wrong. The current best evidence is that clouds are a small, positive feedback (i.e., the add a bit of extra warming).

        Additionally, the evaporation rate depends on temperature, so it sounds like even your method requires some warming.

      • attp, “The current best evidence is that clouds are a small, positive feedback (i.e., the add a bit of extra warming).”

        If you allow for scientific inertia, the time it takes for past bad wild assed guesses to be replaced with more accurate scientific wild assed guesses, cloud feed back would be a regulating response and not a feedback. I believe this is part of the new “adjustments” and aerosol indirect effect revisions.

        I suggest you visit science of doom and look up the initial versus boundary value issues of climate models.

      • JCH: He seems to be asking about a calculation in the future – after some amount of warming

        And not just the future, but the past as well. If the Earth surface has warmed 0.9C since 1880 or so, how much have the rates of advective/convective, evapotranspirative, and radiative transfer of energy from the surface to the upper troposphere increased?

      • Capt,
        In what way is that a response to my point? The current best evidence is that clouds are a small, positive forcing. I don’t know if clouds are actually a small positive forcing, or if it’s going to change as we gain understanding of the role of clouds, but it happens to be the case now. Pointing out that it might change, doesn’t make it not true.

      • CaptDallas2 0.8 +/- 0.2: Probably the easiest way to estimate the “constraints” is by considering DWLR estimates that are already constrained.

        What would be your estimate of the increase in the surface cooling rate (sum of rates of three processes) induced by the 0.9C increase in global mean surface temp? Use another figure if you don’t like 0.9C.

      • I don’t think you understand what a strawman is.

        Oh yes I do. The argument was (in effect, including tacit assumptions) that the surface had to get warmer because that was the only way “this extra heat” could be radiated. Extra radiation isn’t necessarily required to be IR from extra heat. It could be extra incoming SW radiated before it had a chance to heat the surface.

        Any evidence that that is likely or has happened in the past?

        Well, the albedo symmetry between hemispheres strongly suggests that we don’t know enough about the subject to say.

        Additionally, the evaporation rate depends on temperature, so it sounds like even your method requires some warming.

        Of at least the skin layer. However, more low clouds means more downwelling IR (by a couple of orders of magnitude more than CO2), so a positive feedback between evaporation and IR could take place (in certain locations) even while increasing the albedo to the point that it lowered the overall “global average”.

      • attp, “In what way is that a response to my point?”

        THE, current best estimate isn’t “that clouds are a small, positive forcing.” If you insist on assuming that is the best estimate you should expand your reading a bit.

      • There was a hope by some that clouds would increase, but during the most rapid warming of the 90’s, they reduced, so those hopes were dashed. Some think, oh well, maybe next time they will do the opposite.

      • Capt,

        THE, current best estimate isn’t “that clouds are a small, positive forcing.” If you insist on assuming that is the best estimate you should expand your reading a bit.

        Well, prove me wrong. Happy for you to do so. My understanding is that the current position is that clouds are probably a small positive forcing. Soden & Held (2006) is one reference that shows this. I know it’s a little old, but I think you can find similar more recent estimates. Even Bjorns Steven’s presentation at the Ringberg meeting is arguing for a small effect from clouds (i.e., difficult to justify and ECS 3.5).

      • matthewmarler, “What would be your estimate of the increase in the surface cooling rate (sum of rates of three processes) induced by the 0.9C increase in global mean surface temp? Use another figure if you don’t like 0.9C.”

        I don’t believe you can get a reliable estimate of surface cooling rate changes since that depends on weather and land response which is the chaotic part of the problem. For example Arctic Winter Warming is a cooling that produces an increase in temperature while reducing energy. There really isn’t enough polar data available to do anything realistic.

        Until more data is available I believe the Kimoto style ratio based on effective temperature and energy is as good as it gets. Then if you have accurate numbers for latent and sensible you would just ratio the ocean contribution and consider some land average with a higher uncertainty range. Basic estimates are simple it is the details that are the bitch.

      • HTML messed up the end of my last comment. Was meant to be difficult to justify/motivate/explain an ECS smaller than 1.5K or larger than 3.5K.

      • Matthew,

        What would be your estimate of the increase in the surface cooling rate (sum of rates of three processes) induced by the 0.9C increase in global mean surface temp? Use another figure if you don’t like 0.9C.

        I don’t know if this answers your question, but there was a recent paper (link to paper in the post) that shows a measured increase in downwelling longwavelength flux over land of 2.2W/m^2/decade between 1973 and 2008 (a total of 7.7W/m^2). Over the same time interval, the increase in outgoing longwavelength flux is about 5.5W/m^2. If the land is in equilibrium, then that implies that about 28% of the net flux leaving the surface is sensible/latent heat (2.2W/m^2 of the 7.7W/m^2). Looking at the figure in the above post, it’s about 20%. So, it seems higher, but I suspect that the period is too short and the uncertainties too large, to really be sure.

      • ATTP, “Well, prove me wrong. Happy for you to do so.”

        The easy way is just to look at tropical SST and deep convection. The “hiatus” that doesn’t exist is mainly due to variation is tropical deep convection which changes stratospheric water vapor and ozone. The radiant part of clouds is just part of cloud response. Stephens et al 2015 did a good job of showing how quickly clouds respond to stabilize albedo. That would be a regulating response not a feedback.

        http://www.met.reading.ac.uk/~py904867/home/thesis.pdf

        This these by Andrew Barrett .details some of the cloud parameterization issues, specifically mid level liquid layer top stratiform clouds which by Barrett’s estimate correct parameterization would reduce their model sensitivity to about 1.6 C per doubling. Barrett considers MLLLT clouds to be a “negative” feedback, but he is young. Regulating responses or “adjustments” will become more popular with time.

      • Capt,
        I don’t see how that proves me wrong. I’m well aware that clouds are the big uncertainty. I’m well aware that they might not be a positive feedback. That, however, does not change that currently the general view (best estimate – choose your own term) is that the cloud feedback is probably small and positive.

      • attp, “That, however, does not change that currently the general view (best estimate – choose your own term) is that the cloud feedback is probably small and positive.”

        Then let’s just agree to disagree and wait for time to prove me right :) With a regulating response though it can appear to be either a positive of negative “feedback” depending on your assumed “normal” condition (SOD initial versus boundary). Since clouds would respond mainly to absolute SST and the models generally screw the pooch on absolute SST, you would need to refer to authorities that don’t have models that cannot get SST correct.

        I don’t have a ready link, but someone at the GFDL “tweaked” ESNO region SST and surprise surprise, modeled the pause. Woods hole even managed to model a centennial scale Pacific “oscillation”. So as long as you include losers in your “authority” go to list, our conversation is pretty much a waste of pixels.

      • Capt,
        Sure, happy to agree to disagree. I don’t know if cloud feedbacks are positive, but I do think that that is the current general position.

      • Attp

        Here is the met office take on cloud feedback

        http://www.euclipse.eu/downloads/Presentations_Hamburg2013/markwebb_CFMIP_Hamburg_final.pdf

        Page 6 shows all the various papers expected over the last year or two. Presumably when they are all in we shall have a better idea of whether clouds are a positive or negative feedback.

        Tonyb

      • captdallas2: Until more data is available I believe the Kimoto style ratio based on effective temperature and energy is as good as it gets.

        That has at least as many dubious simplifications as what I wrote, plus the assumption that climate sensitivity is constant. If one of these calculations proves to get the first significant figure with no more than 25% error I shall be surprised. I do agree with you that variations in regional and seasonal responses to temperature increase and forcing increase may make the calculation of the overall effects impossible from aggregates like global surface mean temperature and global total evapotranspirative surface cooling. However, lots of writers treat the global statistical summaries as though their use gets results that are accurate “to the first order.”

      • matthew marler, Stephens et al had +/- 17 Wm-2 surface uncertainty and if you look at the range of latent and sensible estimates you can see why. If you throw out some of the older “accurate to first order” estimates you could get to about +/- 8 Wm-2 but that is about it I believe. Bjorn Stevens and Steven Schwartz had a paper dealing with some of the uncertainty and issues with some model assumptions.

      • David Springer

        …and Then There’s Physics | April 11, 2015 at 4:56 pm |

        “Yes, you’re right that we could retain energy balance simply by changing our albedo through changes to clouds. Any evidence that that is likely or has happened in the past? I think the answer is no, but feel free to prove me wrong. The current best evidence is that clouds are a small, positive feedback (i.e., the add a bit of extra warming)”

        More uninformed nonsense. Clouds, depending on type, can be either positive or negative feedback. Get a clue. High altitude clouds have positive feedback, low altitude clouds negative feedback.

        http://www.climate.be/textbook/chapter4_node8.html

        Lindzen’s Cloud Iris Hypothesis posits that SST warming reduces cirrus cloud cover allowing heat to leave the system more efficiently.

        http://earthobservatory.nasa.gov/Features/Iris/

        “We wanted to see if the amount of cirrus associated with a given unit of cumulus varied systematically with changes in sea surface temperature,” he says. “The answer we found was, yes, the amount of cirrus associated with a given unit of cumulus goes down significantly with increases in sea surface temperature in a cloudy region.”

        This is the finding that led Lindzen’s team to propose that the Earth has an adaptive infrared iris—a built in “check-and-balance” mechanism that effectively counters global warming (Lindzen et al. 2001). Much like the iris in a human eye contracts to allow less light to pass through the pupil in a brightly lit environment, Lindzen suggests that the area covered by high cirrus clouds contracts to allow more heat to escape into outer space from a very warm environment.

        Ignorant prat.

      • matthewmarler, “That has at least as many dubious simplifications as what I wrote, plus the assumption that climate sensitivity is constant.”

        One of the things I liked about Kimoto’s approach was that it was flexible. Since energy is fungible you can expand his basic equation to make it as complex as you like. Assuming that climate sensitivity is constant isn’t really part of his equation, it just assumes a tendency toward equilibrium so you get a “current” sensitivity for each of the energy flux values you consider. The “greenhouse” effect based on his method is about 220 Wm-2 which with the real surface atmospheric window would give you approximately 240 Wm-2 at some radiant surface in the middle troposphere. So as long as albedo remains fairly constant his method would produce a fairly good estimate.

        https://curryja.files.wordpress.com/2015/04/slide1.png?w=500&h=375

        The problem with energy budgets is that they have to be “closed” at some point. The budget posted has latent uncertainty skewed which is misleading I think. Stephens et al had a larger latent uncertainty which was centered. That is more indicative of the assumptions required to close the surface energy budget. Pick a reference in the mid troposphere to TOA or a sub-surface reference and you simplify the problem. It really doesn’t much matter because the “surface” is chaotic which I believe is how the troposphere got its name. Those references give you about 0.6 Wm-2 +/- 0.4 but no perfect explanation of the cause of the imbalance. Some of it has to be CO2 of course, but since the imbalance isn’t symmetrical solar could be larger than most think due to the stage of the precessional cycle.

        I think it is an excellent problem that is being screwed up by too many “accurate to the first order” mistakes.

      • aptdallas2 0.8 +/- 0.2
        matthew marler, Stephens et al had +/- 17 Wm-2 surface uncertainty and if you look at the range of latent and sensible estimates you can see why.

        That’s a point I was meaning to bring up. In my write-up I merely allude to the “uncertainties” without propagating them through the calculations.

        It’s about 25% of the estimate for SH, 11% of the estimate for LH, almost 30% for the net radiative imbalance at the surface. The error bounds on the estimate of the net change in rate of cooling of the surface resulting from a 1C mean temperature increase are about 100% of the estimate (recall that Romps et al have 12% +/- 5% for the estimate of CAPE*PR). That’s assuming the approach is unbiased. If the approach is meaningful, then all the calculations have to be redone as more information is reported.

        I think it makes sense for someone to turn away from trying to estimate “the” climate sensitivity for an abstract Earth equilibrium that can’t exist, and look at the changes in the flux imbalances at the surface of the Earth as it is now (or has been recently), as the surface warms (or has recently warmed.)

        The changes in LH, SH, and radiative imbalance at the surface are all convex functions of the initial temperature. Assuming that the same functions apply in each region of the surface, then working with mean T instead of the distribution of T values somewhat underestimates the mean change in each flux. Working with a toy distribution (uniform between the min and max recorded regional means, i.e. poles vs Sahara) I showed that the bias is not very large. That is what everyone assumes anyway when working with mean T.

        .

      • matthewmarler, “then working with mean T instead of the distribution of T values somewhat underestimates the mean change in each flux. ”

        That is a bit of an understatement. “Surface” air temperature has a range of roughly -80C to 50C which is an energy equivalent range of about -70 Wm-2 to 700 Wm-2 if you allow for some latent. This is the reason I focus on the tropical oceans and not alpine tree rings. 30S-30N is roughly 50% of the surface and 80% of the energy.

        https://lh3.googleusercontent.com/-ukz4Fm2AXYM/VGtyZ7z6SqI/AAAAAAAALwg/L8jhnsps0rA/w682-h449-no/kobashi%2Boppo.png

        That is Kobashi’s NH hockey stick with the Oppo Indo-Pacific Warm Pool. Unlike Mann, Kobashi didn’t smooth out the variability. The imbalance tracks oceans not NH land.

    • Tonyb
      I always look for and enjoy your comments. Can’t wait till sea level 2,and any updates for your temperatures.

      Clouds are hard. Some are bright and reflect back out, some absorb and retransmit, some increase with winds and no models do well in simulating or predicting. Then we have the deep ocean and transport down at the poles to heat the abyss by o.1 *C to return in 1,000 years. Plus the Milankotich orbital cycles and maybe the sun variations or sunspots. Lots of work today but don’t bet too much on economic carbon trading impacting temperatures within our lifetimes. First we need good temperature observations, then ocean observations and then natural variation predictions before one can hope to measure the impact of carbon dioxide on tempeatures.

      The political slant of settled science and denier callouts have only delayed understanding of the real science. Pretend problems are easier to address than real problems like the middle east, poverty in africa, war in the East Russian border and fossil fuel depletion.

      I do enjoy your articles and perspective so thanks for that.
      Scott

    • David Springer

      Jim D | April 11, 2015 at 4:46 pm |

      “When CO2 is doubled, the earth has to figure out how to radiate an extra 3.7 W/m2 to space.”

      Absolutely not. Greenhouse gases do not create extra energy.

  26. I’ve got 2 really big issues with basic climate science :-
    (1) The Trenberth Energy Budget Diagram. – No one knows how much energy is in the atmosphere whizzing around, creating hurricanes, calming down for a still period, etc – not to mention the energy in the oceans as well. Yet there’s always an unspoken and unchallenged assumption that a balance is necessary. An instantaneous, microscopic balance, yes. But overall? Are we really able to measure the outgoing energy from top-of-atmosphere well enough? TOA is a variable, changeable position in space.
    (2) Human carbon-dioxide emissions are called an ‘External Forcing’ and then treated as if they’re as impervious to anything happening on Earth or in its’ atmosphere. And just because a co2 molecule will radiate IR energy, there’s an assumption that there must be extra sensible, measurable heat – which ca’t be found. But energy can be used for lots of things – movement, state-change etc.

    Are there any references that go into these things indetail?

    • They’ve taken a basic laboratory finding, plugged it into expensive and powerful machines with the best presently possible programming from a plethora of scientists, and haven’t replicated the real world. It seems they are not even very close. We need to take the meaning of ‘best presently possible programming’ into the future.

      Well, that’s a platitude, but it is the problem.
      ===========

      • In that case (because I take notice of real-world observations), I’ll says the extra atmospheric co2 does a bit of feeding microbes, a bit of helping create reflective clouds and a bit of melting ice-crystals in clouds to give more rain.

        Is any real-world evidence for that?

      • The Lab experiments do at least show that carbon-dioxide molecules can output sensible heat under certain circumstances. They would always prefer to get away from surrounding molecules and hare off into outer space but are prevented by the Coriolis force of the Earth’s spin and pressure of the upper atmosphere.

        Under high-pressure systems where carbon-dioxide molecules can’t escape, we’d expect some additional heat but not as much as seen when confined to a flask in a Lab.

        This would give a maximum temp just a little bit higher, but make an ‘mean surface temp’ calculated by averaging max and min values too high. I think HADCRUT & GISS are calculated like this.

      • “…best presently possible programming from a plethora of scientists…”

        I doubt that is true. Software engineering is a specialized and lucrative profession with many interesting projects to explore. I seriously doubt that the people that write the code for the climate models are up to industry standards. That code is likely written by students, post docs, and otherwise distracted academics. In the private sector, software engineers are focused, richly compensated, and easily fired.

        In addition, private sector software engineers face many daunting constraints. There is pressure to produce price and feature competitive products. Technical support, constantly harassed by outraged customers, puts enormous pressure on engineering to fix their bugs. Quality assurance is compensated by how well they surface bugs, which if missed will quickly emerge on the tech support battle front. Product marketing and management drive product features based on market competition. The whole thing is a dogfight and nobody gives a rats patoot about anyone’s CV. I spent 2 decades in that business and I doubt those academics, who I know well from my university experience, could handle it.

      • I agree. I should have said ‘best presently possible in climate science’. Much room for improvement, as you point out, in a very daunting process. The modelers were aggrandized by the power of their tools without having sufficient humility to sense the power of what they were studying.
        ================

      • They’ve been too busy carving the stone into the desired image instead of just removing the overburden from the beauty within.
        ======================

  27. Looking at the diagram of heat flows, how much do these vary by Time of Day, Season and Latitude? I’m assuming that these variations are an order of magnitude or so greater than the CO2 effect. Is that correct?

    • Doesn’t seem to be a whole hell of a lot of global warming at night, in the Libyan desert, does there?

      I suspect you are right, but 97% of of the completely clueless would disagree, no doubt! The fact that Summer differs from Winter seems to escape them completely.

    • This is why Jim Cripwell yammered on so about sensitivity being indistinguishable from zero. He was hammering on the same point Koonin does, that the signal is so far too small to detect.
      =============

    • I deal in nanogram quanties of drugs, so I find single digit percentages easily in the detectable range.

      Even as much as I detest percentages, the use of percentish brings that to a whole new level.

  28. The statement by Dr. Koonin

    An even simpler indication of the percentish influence is to note that a 3 C mean global surface temperature increase on a base of 288 K is also about a 1% effect.

    is beneath contempt. Without greenhouse gases the surface of the earth would be near 255 K. Eli would not be surprised to see such a lights out statement from Kim, indeed it would be a step up in the usual.

    • What would the temperature be of a “snowball Earth” with an albedo of 85%? No CO2 or water vapor (to speak of) in the atmosphere?

    • 70% of the surface is a thermal reservoir which because of its heat capacity has to keep the surface warmer than a black body, irrespective of greenhouse effects.

      • Arthur Smith has considered these effects and more in an arXiv article. If there is no CO2 in the atmosphere, there soon is no water vapor either as we enter a super snowball Earth phase. We is of course wrong, as it would be difficult for any life on the surface to survive.

      • No such mention of oceanic thermal reservoir effects there that I can see.

      • “We then end up with essentially the same equations as in the previous section, for example Eq. 16 is the same, except that effectively the solar input S and thermal inertia c in those equations are increased by the factor 1/(1−f /2).”

        Also Ref 4

        See Principles of Planetary Climate, by R. T. Pierrehumbert (retrieved February 2008 from
        http://geosci.uchicago.edu∼rtp1/ClimateBook/ClimateBook.html) – in particular section 8.3 on thermal inertia

      • A paywall, a dead link, and thermal inertia is not the same as absolute thermal capacity.

      • 70% of the surface is a thermal reservoir which because of its heat capacity has to keep the surface warmer than a black body, irrespective of greenhouse effects.

        This doesn’t make any sense to me. In the absence of the greenhouse effect and with an albedo of 0.3, the surface will absorb 240W/m^2. To be in energy balance it will have to radiate – on average – 240W/m^2 back into space. Since there will be no atmosphere, or it will be radiatively inefficient, convection and evaporation can’t transport energy from the surface. Therefore, the surface will have to have an effective blackbody temperature of 255K. The existence of an ocean doesn’t change this basic energy balance picture.

        All that having a large heat sink (ocean) means, is that if you are out of energy balance, it can take a long time to return to energy balance if you need to re-equilibrate the ocean (i.e., add or remove a large amount of energy from the ocean).

      • In the absence of the greenhouse effect and with an albedo of 0.3 […]

        Where did that number come from? Wouldn’t a frozen earth have an ice-like albedo?

      • ATTP said:
        “This doesn’t make any sense to me.”

        If they were solid it would not make sense, but the oceans get heated at depth, they do not convect during the day, but they do convect at night.

      • The heat comes up night rumbling
        From the solar through the day.
        ======================

      • aTTP:
        “In the absence of the greenhouse effect and with an albedo of 0.3,..”

        Just a nitpick but without water vapour and hence clouds there would be no cloud albedo.

      • Just a nitpick but without water vapour and hence clouds there would be no cloud albedo.

        Yes, I know. That’s why I specifically defined an albedo, to – unsuccessfully, obviously – avoid nitpicks.

        If they were solid it would not make sense, but the oceans get heated at depth, they do not convect during the day, but they do convect at night.

        So what? If the planet receives an average flux of 240W/m^2, then the average flux it needs to radiate has to also be 240W/m^2. If there is no greenhouse effect then either there is no atmosphere or it is radiatively inactive. This means that the only way in which the surface can lose energy is through radiation. Therefore the surface must radiate the same average amount of energy per square metre per second as a 255K blackbody (240W/m^2). The oceans may mediate this in some way, but they cannot act to change what the planet – on average – radiates into space in the absence of a greenhouse effect. The system has to – on average – be in energy balance. The existence of an ocean doesn’t change this basic fact.

      • “The existence of an ocean doesn’t change this basic fact.”

        Night time convection and the lack of convection in daytime does. What is heated at depth cannot be radiated away immediately.

      • Just a change of ratio of daytime/nightime clouds could make a huge difference; you wouldn’t even need to find change in amount, height, or location.
        =============

      • ulric,

        Night time convection and the lack of convection in daytime does. What is heated at depth cannot be radiated away immediately.

        That doesn’t change my point. Without a greenhouse effect, a planet that receives – on average – 240W/m^2 will have to settle to a state where it radiates – on average – 240W/m^2. The existence – or not – of an ocean cannot change this. If we were radiating – on average – more then we recieved we would cool. If we were radiating less, we would warm. In both cases we would tend to a state where we radiated – on average – the same amount of energy per square metre per second as we receive. This is basic energy conservation.

    • Without greenhouse gases the surface of the earth would be near 255 K.

      What would be the global average albedo in the absence of all greenhouse gases? All meaning to include those that are in the vaporous and liquid state of water.

      Where does all that cooling come from?

      • Dan, you are basically walking into the issue of how to escape from snowball Earth and you really can’t do it without greenhouse gases. One of the theories about how that happened was large volcanic eruptions.

      • Well, try this one, in the past few million years, the Earths global temperature has swung maybe 10 C. Let Eli see 3/10 = 30%

    • I’m sorry you have contempt, but your follow up doesn’t appear to have any relevance to the statement.

      Koonin’s statement, I guess, is that 3K is about 1% of 288K which is close to the global average for April and October.

      Your contempt seems to be that he didn’t consider an imaginary case without greenhouse gasses ( not particularly relevant to the real world of today ).

      But,
      3/288 = 1.04%
      3/255 = 1.18%

      Both are “about a 1% effect”

      But wait – observations are that temperature rise associated with the nominal ( not effective, but nominal ) forcing of 3.7W/m^2 is closer to 1.6K, not 3K:
      http://climatewatcher.webs.com/TCR.png

      So even for a doubling ( which we are less and less on the path to ever reaching ), it’s about a 0.5% effect.

      We know that doesn’t cause the end of the world because global average surface temperature varies by about 3.5K every year:
      http://climatewatcher.webs.com/TCR.png

    • David Springer

      Eli Rabbett writes: The statement by Dr. Koonin

      An even simpler indication of the percentish influence is to note that a 3 C mean global surface temperature increase on a base of 288 K is also about a 1% effect.

      is beneath contempt.

      =====================================================

      Yup. I don’t often agree with Eli but in this case I do.

    • “Beneath contempt”? Some denizens need a refresher in Civil discourse…

  29. Even worse, Steve Koonin is comparing the world to one without CO2, something that has never existed. The forcing from 2X CO2 exceeds the difference between today and the depth of the ice ages.

    Indeed, beneath contempt

    • The comparisons are meaningless to considerations of current policy options.

      • Well, consider: The difference in global mean temperature btw today and the depths of the ice age is ~6 C. The difference in global mean temperature between preindustrial and today is ~1 C, so that is a ratio of about 20%. If we go up 3 C (which is not out of the question in this century and 6C in a couple is possible) due to pushing greenhouse gases into the atmosphere that is quite substantial.

        At 3C there would be heavily inhabited places on Earth where people would not be able to live because of heat stroke (37 C wet bulb temperature)

        That appears to this naive bunny as policy relevant.

      • Rob Starkey

        Eli

        It seems policy relavant to you because:
        1. you have concluded that temperatures will rise by more significant amounts than the current data supports, AND

        2. you have concluded that a rise in temperature WILL result in a worstening of conditions for humans overall over the long term.

        You do not have any reliable science to support either conclusion.

      • At 3C there would be heavily inhabited places on Earth where people would not be able to live because of heat stroke (37 C wet bulb temperature)

        Really? Which models show this? AFAIK the majority of that mythical average3C” would be at high latitudes.

      • Another way to put 3 C in perspective. Since a 1951-1980 baseline, the summer (June-August) mean temperature in much of the US and Europe has risen by a standard deviation of 0.7 C. This means the bell curves today and then still have a significant overlap, so people are not really noticing. However, adding about 2 C more is three more standard deviations on top of today’s shift. The bell curve of summer average temperature by then has shifted so far right that the extreme coldest summers in the future are the extreme hottest summers of the past, and the future warmest third of summers are all beyond anything we have seen yet. That could be as soon as 2100.

    • The forcing from 2X CO2 exceeds the difference between today and the depth of the ice ages.

      Glacials are not caused by global forcing!

      Indeed, the global forcing may even be positive when the glacials start!

      How can I say that? The glacials begin when orbital variation result in SUMMER sunshine reduction at the poles but summer sunshine increase at lower latitudes, meaning more sunshine fell on the low albedo oceans.

      Beyond the much greater uncertainty, the glacials are a very poor analogy because so much was different, including the higher temperatures of the tropical oceans, which is in part a reflection of the shape of the solar forcing:

      http://upload.wikimedia.org/wikipedia/commons/1/19/CLIMAP.jpg

    • David Springer

      No Eli. You’re assuming sensitivity is a constant. It isn’t. There’s a tipping point when glaciers advance below 65N where albedo increase from additional ice fosters lower temperature and even more ice.

      Repeat after me.

      ECS decreases as SST reaches the strong convection temperature of 28C. It then becomes capped at 30C. Almost nowhere never does open ocean surface ever exceed 30C. Strong convection just starts carrying the heat insensibly into the dry stratosphere where it is released as sensible heat and is free to radiate directly to space from there due being above the emission altitude.

      ECS rises as land surface temperature approaches 0C and high albedo snow & ice persist year round raising the earth’s albedo and causing even more cooling from albedo feedback. There’s a tipping point when permanent glaciers start advancing below 65N latitude. .

  30. Obviously, there’s a great deal of disagreement over the ultimate effect of small percentage changes in the mean-annual radiative balance. My question would be “what are the impacts of the actual (not averaged) radiative forces?”

    The actual incoming/outgoing radiation at any given point is what drives our climate. Global averages are convenient (and currently necessary) fictions that, IMO, can produce pretty good models to play around with.

    Yet every time we use an average we lose (or ignore) data. In contrast, Nature cannot ignore any data. Is that distinction sufficient to distort climate projections to such a degree that they may be misinforming policy makers?

    Does anyone have suggested references for studies on the impact of “averaging the numbers”? I would assume that weather forecasting models make greater use of close-to-actual data (though still a lot of averaged and snapshot data) when compared to GCMs so perhaps that would be the discipline for me to investigate?

    Or am I so far behind the curve of improvements in GCM inputs that my question on the impact of “averages” vis-a-vis model predictions is simply outdated?

    • Why yes, what you want is what you get from GCMs and Earth System Models. Indeed, IEHO, the major advance in Hansen 1988 was to show the distribution calculated for different areas. Now it is true that resolution even today is too low for what we want, but yes, the places to get the best idea of what will happen are GCMs.

      • onlinelibrary.wiley.com/store/10.1002/jgrd.50174/asset/image_n/jgrd50174-fig-0001.png?v=1&s=467a611f176b685bf132967763ecea88d60c0cc0&name=test.png

      • Not if you want <1.5K climate sensitivity. Can't get that out of a GCM. People have tried, which is one of the reasons not to trust Lewis and Curry, their estimate is too close to the lower limit of the physically possible.

      • Eli continues the fantasy of arguing that when a GCM does not match observed conditions, then it does not mean there is an issue with the model.

        What science argues for averaging the reults of a bunch of models of varing accuracy and then making policy decisions based on the results. Answer–no valid/reliable science follows that practice.

        Eli– what speific GCM do you rely upon to forecast temperature change for the next 20 years?

      • Steve Fitzpatrick

        Josh Halpern,
        “but yes, the places to get the best idea of what will happen are GCMs.”

        I expect the humor in that comment is lost on you.

        In any case, Lewis & Curry based their calculations on the IPCC best estimates of forcing and well known historical temperature changes. If you believe their PDF fails to account for longer term effects (eg an increase in climate sensitivity with rising temperature), then you should make that argument… and remember that is not a consistent behavior in GCMs, so it may not be an easy argument to make. It seems a bit of a stretch to suggest a bunch of models which consistently predicted much greater warming than is being actually observed are a reliable way to estimate what will happen in the distant future.

      • Eli Rabett: Why yes, what you want is what you get from GCMs and Earth System Models.

        Those have produced estimated rates of mean temperature increase that are substantially too high for any use. They illustrate one of the points of Steven Koonin’s essay: the errors in the estimates of climate change are larger than the quantity that are to be estimated. The misfit of models to data over the last 2 decades has many explanations as yet untested with respect to out of sample data; on explanation is that the climate sensitivity to a doubling of CO2 (given the climate as it is now) is close to 0 and of indeterminate sign.

    • What I was getting it is that regardless of how well any model reproduces a global average, it is insufficient for projecting local or regional effects. The average can be informative without being truly useful since, for example, none of us lives in a perpetually average state of climate. As the resolution of model inputs improves, perhaps model outputs will also improve. But if they are tuned to the averages then they may be missing ingredients necessary for higher resolution accuracy. But I don’t know where theoretical examinations of that (possible) problem might be found.

      So regardless of whether the errors in measured (and averaged) global radiative balance are reduced to zero, it may not be sufficient to produce the detailed information we need. As pointed out by Stephens, et al. (2012),

      Models are commonly tuned to the TOA, so direct comparison of TOA fluxes provides little insight into model performance.

      And, further:

      Although model and observations broadly agree in the global mean, important regional biases exist in the modelled energy budgets that are not conveyed in global mean statistics.

  31. The effect is indeed out of proportion to the cause and also the purported desolation of the effect is out of proportion to the effect itself.

    Some “scientists” would have me to believe in a nutshell:

    A 1% enhancement of the GHE (Downwelling IR) supposedly causes up to a 18% increase in temperature gain from the GHE (from 33°C to 39°C) which is only a 2% gain in GAST but will somehow be wholly responsible for such devastating effects as species diversity loss, civilization debilitating weather extremes, and unprecedented SLR; but not only that but also cause everyone to “suffer” through such horrors as warmer nights, earlier springs, and a greener planet.

  32. https://twitter.com/BigJoeBastardi/status/585186308710146049

    Here is the data, but it does not support AGW theory so it must be wrong.

  33. The Very Reverend Jebediah Hypotenuse

    Koonin:

    I write here to explain something that’s quite simple, but has profound implications.

    275 K is only about 1% more than 272 K
    Therefore, there is no important difference between water and ice.

    Humans and chimpanzees and bonobos share about 99% of their DNA.
    Therefore, they cannot be different species in any meaningful way.

    Good stuff.

    • A pity that your ‘percentish’ analogies are irrelevant to a discussion of climate uncertainty, signal to noise, and implications for the attribution problem between anthropogenic and natural causes of climate change. Those things are what Koonin’s post discusses. You might reread it as your comprehension is demonstrably lacking.

      • ristvan: A pity that your ‘percentish’ analogies are irrelevant to a discussion of climate uncertainty, signal to noise, and implications for the attribution problem between anthropogenic and natural causes of climate change.

        I wouldn’t put it that way. Steve Koonin does express concern for the implications of change for life on Earth. That the change is only 1% of baseline on the absolute scale may indeed be biologically important. But the small size makes a potentially important effect hard to identify clearly.

    • It’s astonishing how quickly the usual suspects are able to convene and attack an article with absolutely no understanding of the point made. It makes me wonder how much of the science they understand at all, if this simple article is so vexing to them…

      Koonin’s actual point was not that 1-2% difference can’t produce significant changes, and if you read and comprehended his post that should’ve been plain to see, since he points it out specifically. Instead, his point is that our ability to measure and model that level of change is wholly inadequate at the time scales we have been working with. Only with better measurements, a more complete understanding of the entire climate system, and/or more time can we ferret out an accurate picture of the effects.

      Now, assumingyou understand the topic now, do you have an argument?

  34. An article about asking the question are human influences on the climate really that small is laughable when all the available data says clearly NO.

    I would venture to say zero influence other then the UHI effect.

    AGW theory has predicted thus far every single basic atmospheric process wrong.

    In addition past historical climatic data shows the climate change that has taken place over the past 150 years is nothing special or unprecedented, and has been exceeded many times over in similar periods of time in the historical climatic record. I have yet to see data showing otherwise.

    Data has also shown CO2 has always been a lagging indicator not a leading indicator. It does not lead the temperature change. If it does I have yet to see data confirming this.

    SOME ATMOSPHERIC PROCESSES AND OTHER MAJOR WRONG CALLS.

    GREATER ZONAL ATMOSPHERIC CIRCULATION -WRONG

    TROPICAL HOT SPOT – WRONG

    EL NINO MORE OF -WRONG

    GLOBAL TEMPERATURE TREND TO RISE- WRONG

    LESSENING OF OLR EARTH VIA SPACE -WRONG? I have a study showing this to be so.

    LESS ANTARCTIC SEA ICE-WRONG

    GREATER /MORE DROUGHTS -WRONG

    MORE HURRICANES/SEVERE WX- WRONG

    STRATOSPHERIC COOLING- ?? because lack of major volcanic activity and less ozone due to low solar activity can account for this..

    AEROSOL IMPACT- WRONG- May be less then a cooling agent then expected, meaning CO2 is less then a warming agent then expected.

    OCEAN HEAT CONTENT TO RISE- WRONG – this has leveled off post 2005 or so. Levels now much below model projections.

    Those are the major ones but there are more. Yet AGW theory lives on.
    Maybe it is me , but I was taught when you can not back up a theory with data and through observation that it is time to move on and look into another theory. Apparently this does not resonate when it comes to AGW theory , and this theory keeps living on to see yet another day.

    Maybe once the global temperature trend shows a more definitive down trend which is right around the corner (according to my studies ) this nonsense will come to an end. Time will tell.

  35. All AGW is talk and more talk without any data to make their case. They have nada data to make their case.

    I dare them to show me the data.

    • I dare you to try and understand it if they did.

      • ATTP, you should really leave the snarky admonishment at home. After all, you admitted in an earlier thread it’s your inability to control yourself that leads you to ban anyone you disagree with on your own blog, ostensibly in an effort to hide that shortcoming from the world. So your ability to civilly comment to others is questioned by many… Including yourself.

      • After all, you admitted in an earlier thread it’s your inability to control yourself that leads you to ban anyone you disagree with on your own blog

        No I didn’t. I was talking about Twitter, not my blog.

      • Lookie, no point in revealing the data because yer
        wouldn’t understand it if they did. Argumentum Ad
        Ass-sum-tu-um.

    • Santoron,

      Thanks for pointing that out. It explains a lot.

  36. krmmtoday says (April 9, 2015 at 3:49 am) “Which leaves us with the precautionary principle.”

    Indeed; but the PP is “global”, which is to say, it must be applied to all possible dangers. When that is done, what you get is basically what we’ve got; people apportion most resources where they feel most endangered NOW and less resources to far-distant possibilities.

  37. The radiative forcing chart relative to 1750 is completely wrong. It has no hard data to back it up with just more pie in the sky from AGW theory.

    The blind leading the blind.

    • @SDP: The radiative forcing chart relative to 1750 is completely wrong.

      In what regard? That CO2 has not increased since 1750, or that a 43% increase in CO2 (400/280) cannot change the global mean surface temperature?

  38. Eli Rabett wrote:

    “… their estimate is too close to the lower limit of the physically possible.” Last I heard, the uncertainty in cloud albedo alone is of sufficient magnitude to make even zero sensitivity “physically possible.” Adaptive iris and all that stuff. What do you have that shows the lower limit of “physically possible?” If your response refers to GCMs, I will take that as a “nothing.”

  39. Scott
    They don’t need to. They are at the controls of the run away train and smear, defame and obsfucate to confuse the public. We need articulate and competent spokespeople to prevent the data and provide arguements to convince the public bypassing mainstream media. Very sad the Dr Koonin backed away from the POPA group and let the activists control the message.
    Scott

  40. present the data

  41. http://www.climatedepot.com/2013/08/09/paper-finds-lifetime-of-co2-in-atmosphere-is-only-5-4-years-paper-presented-at-the-seventeenth-symposium-on-thermophysical-properties/

    CO2 LIFE TIME
    TME 5.4 YEARS PLUS IT FOLLOW THE TEMPERATURE DOES NOT LEAD IT.

    but this is ignored because like everything else that does not conform to AGW theory which is just about all of the data ,all of the data is wrong.

    Just wrong because it does not support their theory.

  42. Dr. Koonin, thanks for this post and for the WSJ piece. As you can see from some of the comments above, level headed factual engagement has little or no impact on committed warmunists pushing political agendas. Unfortunately that also appears true for the APS despite your efforts, which is more damaging to general trust in science and scientific societies.

    • Rud makes such an excellent and important point.

      The problem with the discussion about climate change is that the “warmunists” are politicizing the science, you know, like by using political and polemical labels to characterize those that disagree with them about the scie….

      Oh.

      Wait.

      Nevermind.

      • Another snarky attack with no contribution to learning. Are you here for science or just to play Climateball?

        Again you miss the mark, Joshua. If derogatory name calling concerns you so, you’re Again speaking to the wrong side. Clean up your own house and watch what that does for civil discourse throughout. Until then, your faux concern is hypocritical nonsense clogging up an informative thread.

  43. The physical smallness of anthropogenic influences, which comes as a surprise to many non-climate-expert scientists, has profound implications for climate understanding and modeling. First, it means precision observations are required to see the climate response. Second, it means that natural variations can easily overwhelm human influences, at least on multidecadal scales (witness the current stasis in global mean surface temperature). And finally, because life at the 1% level is rich, the models have to get many small phenomena right to confidently isolate and project the response to anthropogenic effects.

    I think those points have been made before, maybe sporadically rather than consistently as a steady metaphorical drumbeat. They are worth repeating over and over again.

    Also, the diverse human impacts (UHI effect, deforestation, CO2 increase) are not the same size.

    Thanks to Steve Koonin for a good essay.

  44. “AR5 WG1 Figure 2.11 shows the global radiative balance, with the total downward flux on the Earth’s surface estimated as 503 ± 7 W/m2……”

    “AR5 Figure SPM.4 shows the total anthropogenic direct perturbation (i.e., absent feedbacks) of this balance (the “forcing”)to be some 2.3 ± 1 W/m2, less than 1% of the downward thermal flux or less than 0.5% of the total downward flux.”

    So the uncertainty in the energy flux is 7 w/m^2, the anthropogenic contribution is less one third of the overall uncertainty, and has 700% better precision than the basic flux measurement and I am supposed to conclude that the human contribution represents an existential threat to the biosphere?

  45. Figure 4 from Harde shows that a doubling from the pre-industrial 280 ppm to 560 ppm increases the absorptivity by about 1% on a base of 82%, or, again a percentish shift. An even simpler indication of the percentish influence is to note that a 3 C mean global surface temperature increase on a base of 288 K is also about a 1% effect.

    well actually that graph shows a doubling from 380 to 760 ppm. Careless but the result is the same.

    And an increase of 1% in absolute temperature causes an increase of 4% in the T^4 Planck feedback.

    That is why the whole idea of tipping points and further “catastrophic” warming is ill-informed. Nothing is going to get the better of a T^4 negative feedback.

  46. Steven,

    Physicists should take the time to understand their physics better.

    Only 1% to 2% . . . that may sound small and insignificant . . . but it isn’t.

    It is well known that the normal human body temperature is about 310 K. Furthermore, it is also well known that a seemingly small change (up or down) in absolute body temperature by only 1% (3.1 K, or 5.6 F) would make one sicker than a dog, and, that a 2% change in body temperature (up or down by 6.2 K, or 11.2 F) will virtually guarantee a dead body. From this, it should be sufficiently clear that, when viewed in absolute energy terms, the viable margin between life and death in the Earth’s biosphere is remarkably narrow – so much so that a seemingly insignificant 1% to 2% change in the total energy of the global environment will invariably result in serious disruption of the established infrastructure of life in the biosphere.

    There is no substitute for appealing directly to basic physics for physical insight and better understanding of the ongoing global warming problem. And I do recall one particular case in the 1970s (in which you might have participated) when the JASON group of physicists was tasked to weigh in on the then open question of radiative forcing due to doubled CO2. At that point in time, the JASONs had available the computational resources to calculate one of the earliest line-by-line radiative forcing determinations for doubled CO2. They found the downward flux change at the ground surface to be less than 1 W/m2, from which they erroneously concluded that the radiative forcing caused by the doubling of atmospheric CO2 was “not all that significant”.

    While the JASON group’s radiative calculations were numerically on target, the JASONs were clearly mistaken in their interpretation of the calculated results. Radiative forcing takes place over the entire atmosphere, and not just at the ground surface. If they had to select a single point on the vertical profile that best describes the radiative forcing by CO2, they should have selected the tropopause point, where the instantaneous flux change due to doubled CO2 is nearly 5 W/m2 for a clear-sky atmosphere. Moreover, the JASONs did not take into account the additional radiative magnification that is invariably contributed by the longwave opacity from water vapor and cloud feedbacks, which are several times larger than the radiative forcing due to CO2 alone, and therefore should have been included in their analysis.

    In simple terms, the basic essence of the global warming problem is best understood as a straightforward problem in global energy conservation, as was first noted by Joseph Fourier in 1824. Specifically, the global-mean surface temperature of the Earth is about 288 K, which implies that the Planck emission from the ground surface must be about 390 W/m2. Furthermore, the global-mean solar energy absorbed by the Earth is observed to be about 240 W/m2 (with about 100 W/m2 reflected directly back to space).

    Given that the Earth should be in near-global energy balance, this implies that the Earth must radiate about 240 W/m2 of longwave energy out to space (as has also been verified by satellite measurements). Absent the greenhouse effect, the 240 W/m2 of absorbed solar energy can only support a surface temperature of 255 K. This “missing energy” circumstance led Joseph Fourier to conclude that there must be thermal heat energy radiated downward from the atmosphere to supply the additional heating of the ground surface.

    The flux difference of 150 W/m2 between the 390 W/m2 emitted by the ground surface and the 240 W/m2 of LW flux going out to space at the top of the atmosphere is a direct measure of the strength of the terrestrial greenhouse effect. Greenhouse action builds up the surface-emitted flux to 390 W/m2 and creates the ensuing reduction by 150 W/m2 of the outgoing longwave flux to space – all accomplished by radiative energy transfer means (via sequential emission, absorption, and re-emission interactions).

    Physicists should also appreciate the nature of the Clausius-Clapeyron relation, and the fact that it is exponential in temperature. Undisturbed, with a source of liquid water, the atmosphere is always striving to reach an equilibrium 100% relative humidity. In simple terms this means that the holding capacity of the atmosphere for water vapor doubles for every 10 K increase in atmospheric temperature. And, there is no doubt that water vapor is a very potent greenhouse gas.

    Detailed radiative attribution calculations show explicitly that water vapor accounts for about 50% of the 150 W/m2 of greenhouse effect, and that longwave cloud opacity accounts for 25%. Both of these radiative effects are due to the climate system’s fast feedback processes. The remaining 25% of the greenhouse effect comes from the radiative forcings by the non-condensing greenhouse gases (which incidentally also act to sustain the terrestrial greenhouse effect at its present strength). Of the non-condensing greenhouse gas contributions, CO2 is by far the strongest contributor accounting for about 20% of the 150 W/m2 greenhouse effect, with the remaining 5% due to minor GHGs like CH4, N2O, O3, and CFCs.

    A key point to keep in mind is that it is these non-condensing greenhouse gases that act as the principal radiative forcing agents of the climate system. Because of their thermodynamic, chemical, and radiative properties, CO2 and the minor GHGs are chemically slow-reacting with atmospheric lifetimes ranging from decades to many centuries. Once they are injected into the atmosphere these gases effectively remain there indefinitely by not condensing or precipitating at prevailing atmospheric temperatures as they continue to exert their radiative forcing.

    Since CO2 is the strongest and most effective of these non-condensing radiative forcing gases, it then follows that CO2 can be identified as the principal LW control knob that governs the global climate of Earth. The fact is that the other forms of radiative climate forcing (e.g., changes in solar irradiance, surface albedo, and aerosol forcing) are small by comparison. This makes the case for recognizing CO2 as the principal climate control knob that much more compelling.

    Atmospheric water vapor, on the other hand, has the role of principal fast feedback process in the climate system by condensing and precipitating from the atmosphere in response to changes in local meteorological conditions (constrained by the exponential temperature dependence of the Clausius-Clapeyron relation), meaning that the atmospheric distribution of water vapor (and clouds) can change rapidly on a time scale of hours and days in response to changing weather conditions.

    Applied radiative forcings that heat (or cool) the atmosphere cause more (or less) water vapor to evaporate, which generates more (or less) longwave opacity, which then contributes more (or less) radiative greenhouse effect. Such changes in water vapor cause big changes in radiative heating or cooling, but the changes are limited in magnitude by how much change the water vapor undergoes in reaching its new equilibrium distribution.

    Because of this, water vapor and clouds act to magnify the initial radiative perturbation, but cannot on their own initiative manufacture or impose a warming or cooling trend on global climate, even though they contribute more strongly to the atmospheric radiative structure than the radiative forcing gases that actually drive and control the global temperature trend.

    The physics cause-and-effect nature of the global warming problem is not all that complicated. The basic “cause” component of global warming has been clearly identified and understood for many decades, and has been accurately quantified by precise measurements of atmospheric CO2 (e.g., the Keeling curve).

    This is fully corroborated by the latest annual data report of fossil fuel extraction that now approaches 10 gigatons of carbon/yr (roughly equivalent to 10 cubic km of coal/yr, which when burned, adds about 5 ppm CO2 to the atmosphere, half of which remains there for many centuries). The radiative effects of CO2 are fully known from well-established understanding of greenhouse gas radiative properties and radiative transfer modeling of the atmospheric structure.

    How can a physicist not comprehend that it is atmospheric CO2 that is the principal radiative forcing agent for the ongoing global warming? . . . and not be concerned that water vapor, as the climate system’s principal feedback agent, has an exponential dependence on temperature?

    To be sure, there are other factors that contribute to climate change. But decades of measurements and analysis have shown that variations in solar irradiance, land use, aerosols, ozone, and other minor greenhouse gases, while making a contribution, are small by comparison to CO2.

    Of greater interest is the “unforced” variability of the climate system on decadal time scales that arises from changes in ocean circulation patterns that are effectively un-influenced by changes in atmospheric radiative forcing. The deep ocean is a very large cold storage reservoir. An upwelling blob of cold ocean water can put a “pause” in the ongoing global warming, temporarily diverting the greenhouse “heat” to warming the ocean. But once that cold blob of ocean water has been warmed up to its equilibrium temperature, it is back to the business of continued global warming. And also note that the ocean cannot cause a decadal warming spurt – the deep ocean is colder than the surface biosphere, so it cannot be a source of heat.

    Significantly, the key climate system components (water vapor, clouds, ocean) are not configured to respond to radiative and/or temperature perturbations on a sufficiently small enough incremental scale that would permit a monotonic approach to global energy balance equilibrium. Instead, there is always over-reaction such as when water vapor condenses en mass to produce storms, coupled with the similarly over-reactive responses by atmospheric and ocean dynamics to pressure-temperature and salinity differences, to produce the quasi-chaotic weather and the longer-term climate noise that characterizes the climate system.

    Physicists should not be confused by these random-looking quasi-chaotic fluctuations about the local climate equilibrium point, and should instead focus more on the changing energy balance equilibrium point of the climate system. They should also pay attention to the geological record that points to an atmospheric CO2 level of 450 ppm as being incompatible with polar ice caps, a level that is expected to be reached by the end of this century. While it may take a thousand years for the polar ice to melt, the future course is being prepared for a 70 meter rise in sea level.

    • The biochemical processes within the human body operate only under a much closer range than do the climate processes for the Earth. Excellent analogy, esp. for in how it fails.
      ==================

      • I don’t understand this “control knob” claim, although it’s been made many times. From everything I’ve read, co2 has always lagged temperature according to ice cores, so how can it be a control knob? A volume knob is a control knob on a stereo – it controls volume. I’ve never seen a stereo get louder by itself first, and then see the volume knob move afterwards.

        What am I missing here?

      • What you’re missing is feedback. If there were a feedback from the volume of your stereo (suitably defined) to the volume control then you would see the volume control move when the volume changed.

        But while there is no such feedback in that situation, there is between CO2 and temperature. More CO2 traps more heat, which raises the temperature, which (a) liberates more CO2 from the ocean and (b) evaporates more water from the ocean. The latter in particular is a very powerful feedback.

      • You make an interesting point. How does this then explain that temperatures fall in advance of the CO2 which continues to rise for some time before falling. I understand the idea that cooling oceans remove CO2 from the air but what brings about the start of the cooling phase?

      • “…More CO2 traps more heat, which raises the temperature, which (a) liberates more CO2 from the ocean and (b) evaporates more water from the ocean. The latter in particular is a very powerful feedback.”

        Beautiful. Thank you, doc. So, there must be a lot of evidence for that “very powerful” water vapor feedback. You will get a very nice cookie, if you can find it.

      • How does this then explain that temperatures fall in advance of the CO2 which continues to rise for some time before falling.

        CO2 and temperature are tied together. Milankovitch theory says that the cycles are triggered by changes in insolation, which are then amplified by the positive feedbacks involving both CO2 and water vapor, whose increase caused by rising temperature insulates the planet better, resulting in further warming. You would therefore expect changes in temperature to lead changes in CO2 (and water vapor but that’s impossible to measure in air bubbles in ice cores because ice is H2O), but only to the extent that there is any slack in that coupling.

      • Yes, the CO2 feedback to warming a degree could be as much as 10-15 ppm. An order of magnitude less than emissions, but there nevertheless. This was the dominant effect as the last Ice Age ended, and 90 ppm was added that way to get from 190 ppm to 280 ppm as the earth warmed 6-10 C.

      • So, there must be a lot of evidence for that “very powerful” water vapor feedback.

        Great question, Don. The evidence is in two parts.

        1. We know that higher temperatures increase water vapor in the atmosphere. Two mechanisms are involved.

        (a) By increasing the rate of evaporation at the surface.

        (b) By decreasing the rate of condensation at any given altitude.

        For both these mechanisms, theory and observation were in excellent agreement long before steam punk was invented. The basic principles were developed largely in the 19th century, e.g. Clapeyron’s 1834 paper, while steam tables have been intensively studied since their formulation in 1934, exactly a century later.

        2. We know that water vapor traps heat. This was measured in the laboratory by Tyndall, and can be observed in the atmosphere from space by comparing the amount of radiation leaving Earth at wavelengths known to be blocked by water vapor with that at other wavelengths. The former wavelengths seriously notch the spectrum at the former wavelengths. (Similar observations at ozone- and CO2-trapping wavelengths show the same thing for those greenhouse gases.)

        Putting 1 and 2 together, we infer that higher temperatures result in the atmosphere trapping more heat.

        By how much is an excellent question that is currently above my pay grade (but it’s on my list to learn about). In the meantime, settle for the points that (a) without those greenhouse gases the Earth’s surface would be 33 °C colder and (b) the effect of increasing GHGs is to widen the portions of the spectrum that they block, leaving less of a window for heat to escape through, so wider blocked regions can be expected to have a nontrivial effect.

        If that warrants at least half a cookie I’ll be in Sydney in May. (But you’re in QLD?)

      • That’s pathetic, doc. We know about the inferences. Where is the evidence that water vapor has increased in the atmosphere as a result of increasing CO2? I am pretty sure you understand the question. Rapid powerful water vapor feedback should be detectable. You ain’t detecting. No cookie. Not even a crumb. In fact, there is evidence that there has been no powerful water vapor feedback, which you surely know about:

        http://www.nws.noaa.gov/ost/climate/STIP/37CDPW/37cdpw-jbytheway.pdf

        This climate alarmism hysteria has corrupted a lot of otherwise decent people.

      • The Held item on Week in Review is worth looking at, but rather long. Bottom line: faster-than-expected polar warming can reduce or delay the water vapor feedback and tropical hot spot. A single global temperature change doesn’t tell you about the distribution of the warming which influences the water vapor feedback in the transient climate change phase.

      • “transient climate change phase”

        Phrase of the day – good job.

      • Don Don’s nearer than you presume, Vaughan. Perhaps he’ll offer to meet you in Silicon Valley, with the same arrangements he made with Moshpit?

      • Thank you, jimmy dee:

        ‘Bottom line: faster-than-expected polar warming can reduce or delay the water vapor feedback and tropical hot spot. A single global temperature change doesn’t tell you about the distribution of the warming which influences the water vapor feedback in the transient climate change phase.”

        We will take that as a painful admission that there is no evidence for a POWERFUL and RAPID water vapor feedback, along with some of your lame obligatory armwaving excuse making BS. When is the rapid and strong positive water vapor feedback going to kick in, yimmy? How long do some evaporation take these days?

        “faster-than-expected polar warming”

        Why is everything faster-than-expected with these alarmist characters, with the embarrassing exception of the RAPID and STRONG water vapor feedback response? OMG! Who do they think they are fooling?

      • Mind your own business, wee willy. I have plenty of friends in real life. I don’t need to socialize with any blog characters.

      • Don, you keep quoting a “rapid” water vapor feedback. Who are you quoting? Are you sure rapid means it is happening tomorrow? The ocean takes a while to catch up to the forcing, but it is warming at its own pace and the water vapor feedback follows that.

      • Everybody can see how dishonest you are, jimmy. You yammer on about hot it has gotten. And OMG! it’s getting hotter all the time. Hottest evah! How freaking long does it take for water to notice that it’s getting hotter and it’s supposed to evaporate, jimmy?

        In the NOAA chart of TPW, you can clearly see that evaporation rapidly reacts to react to temperature changes. You can also see that there is no freaking trend for 22 years, when you jokers have been screaming that it’s getting hotter all the time and we are in for imminent disaster. How does that heat get down to the hiding places in the deep ocean abysses without evaporating something off the freaking surface, jimmy dee? Have you no shame?

        http://www.esrl.noaa.gov/gmd/publications/annual_meetings/2014/slides/22-140327-C.pdf

        Strong annual cycle found in all latitude zones. No robust trend. That’s an understatement. They wanted to find a trend, but it ain’t there.

      • Don M, so if I understand, you say it is reacting rapidly, but not rapidly enough?

      • > I don’t need to socialize with any blog characters.

        Does it mean you consider Vaughan a blog character, Don Don?

        Let me remind you what you told Moshpit:

        And you give me a childish, dumb, insulting, cowardly reply without even familiarizing yourself with the criticism of John S., who towers over you intellectually the way that I am sure that I dwarf you physically.

        http://judithcurry.com/2012/12/04/multidecadal-climate-to-within-a-millikelvin/#comment-283022

        Don’t you recall when you invited him to meet on friendly terms?

      • Stop the foolishness, jimmy disingenuous. You can read the NOAA TPW charts. All latitude zones show strong annual cycles, but no trends. You can see the reaction to the 97-98 El Nino clearly in 0-30S. It don’t take 22 years to react to increasing temperature. You are not doing the cause any good with your dishonest crap, jimmy dee.

      • You really need to get a life, willy. You are taking this blog nonsense way too seriously. I suggest you show a mental health professional what you have been doing on various climate blogs for the last several years. You must have a file.

      • Don M, you are the only one expecting a rapid water vapor feedback. If it was rapid, we could forget TCR and just talk about ECS because they would be equal.

      • We get it, jimmy. The ever increasing heat is causing the glaciers and ice caps to melt, it’s warming up the deep ocean abysses, but it ain’t evaporating any water. The evaporation is in the pipeline. The water will evaporate sometime in the future. We can be sure of that. And when it does, it will be worse than we thought. Your making me sick, jimmy.

      • Don, from your link :
        …we can neither prove nor disprove a robust trend in the global water vapor data.

        But that slide is out of date. It was first published in 2012. Check out more recent work:

        Mieruch et al. doi:10.1002/2014JD021588
        “…the trends [in global total column water vapor] range from 1.2% to 2.2% per decade.”

        Schr¨oder et al. doi:10.5194/amt-6-765-2013
        “The global mean decadal trend derived from both data sets is…1.67%decade_1 for the CM SAF data set, and …1.62%decade_1 for the RSS data set.”

        The feedback is there and has been measured.

      • You are kidding, patty. The first paper is a convoluted analysis of data from 1996-2005, and the other basically same-same from 1987-2006. How is that more up to date than the NOAA analysis, which actually is decipherable and makes sense? You jokers are really struggling.

      • I don’t believe that patty even read those papers. From the first one:

        “The common observation time period is between 1996 and 2005. Due to the relatively short length of the period, the strong interannual variability with strong contributions from El Nino and La Nina events and the strong anomaly at the start of the common period, caused by the 1997/1998 El Nino, the observed trends should not be interpreted as long-term climate trends.”

        Shame on you, patty. You are as bad as jimmy dee.

      • Don, you keep quoting a “rapid” water vapor feedback. Who are you quoting? Are you sure rapid means it is happening tomorrow? The ocean takes a while to catch up to the forcing

        Well, you should re-read what Lacis wrote ( and Isaac Held ).
        Water vapor is (presumed) to be a rapid feedback.

        Indeed, in the seasonal cycle, here’s the response:
        https://curryja.files.wordpress.com/2013/12/pw_sfct.png

        Less than a month.

        Now, it is an interesting point whether Arctic warming means less global warming, probably so. But you’d have to throw out all the GCMs and start over again, because they seem to have both Arctic Warming and a Hot Spot for 1979 through present:
        http://climatewatcher.webs.com/HotSpot.png

      • TE, the Held article also shows a range of GCM responses with different water vapor feedbacks. Some are better than others. The GCMs have generally missed the decline rate of Arctic sea-ice as much as they have missed the hot spot, which supports the idea that the warming distribution is not what was predicted, but no better in consequence.

      • TE, also obviously the precipitable water is not delayed relative to the surface ocean temperature, because convection is a fast response, but the surface temperature is delayed relative to the forcing. The oceans are lagging the land in their warming rate, especially since 1980 as is consistent with a sharp rise in external forcing. As a result, the global relative humidity, especially over land, should be decreasing in this transient phase. Not necessarily a good thing.

      • JimD, “The GCMs have generally missed the decline rate of Arctic sea-ice as much as they have missed the hot spot, which supports the idea that the warming distribution is not what was predicted, but no better in consequence.”

        but no better? So if you don’t know it has to be as bad or worse? funny how the people that have been pointing out where the models miss and how badly have been looking at what the implications are, “sensitive” in the lower half of the initial estimated range and potentially below the lower estimated limit, more natural variability impact etc. have been catching grief from the minions but now that the reality is unavoidable y’all cling to your talking points and step up the “deniers should be jailed” rhetoric.

        . The reader’s digest version of Helds post is that “global” mean temperature sucks as an energy proxy. That is a zeroth law issue. Has been always will be. Pretty much all of the high end extreme cases are based on a sucky metric aka the worst possible thermodynamic frame of reference one could possibly pick. That is one of those “skeptical” talking points that gets people banned from “scientific” blogs.

      • captd, certainly there are issues with global mean sensitivities. The land is currently warming at nearly 4 C per doubling, and the Arctic even faster. It is very misleading to talk about the 2 C per doubling effective global value without this extra information to supplement it, isn’t it. More skeptics need to pay attention to this variability. It is not a flat warming at all.

      • JimD, “captd, certainly there are issues with global mean sensitivities. The land is currently warming at nearly 4 C per doubling, and the Arctic even faster”

        Zeroth Law, write that down. The “land” from 30 north to 60 north is warming at a higher rate than modeled and can cool at a faster rate than expected. Lower thermal mass and knock on effects related to land abuse. Region where trees were planted, wet lands restored an even water permeable paving install have temperature reductions. The CO2 cause all harm mentality tends to miss those subtle things.

        In the Arctic, most of the warming is in early winter. That would be due to increased wall energy transport aka pole ward advection. Since we don’t have satellites to krige the pre-1950 period we have a false “normal” of no change due to no data. Dr. Curry mentions that the 1930s could have been as warm as today in the Arctic and the minions crawl out of the dark recesses of the internet.

        Dr. Curry mentioned from the get go that better ocean models would be needed and that just so happens to have been dead nuts on. You really need to brush up your basic thermo so you can stop having the tail wag the dog.

      • So, according to little jimmy all the dangerous AGW that alarmists of his ilk have been hollering about for decades, ain’t causing increasing evaporation. The RAPID and STRONG positive water vapor feedback ain’t so RAPID and STRONG, after all. But it’s coming. Maybe another 30-60 years. We can wait.

        As soon as the ever increasing (since about 1950) forcing from the well-mixed CO2 get’s done heating up the 30% of earth’s surface that is land, it will get started on heating up the top layer of molecules on the water. Then we will be sorry.

        In the meantime, the hysterical alarmist ilk will have to pretend they haven’t been hollering about the oceans gaining gazillions of Hiroshima bombs worth of heat that has apparently not affected at all the layer of molecules on the surface. There is something Biblical about all this, rev. jimmy. Old Testament stuff. Miraculous. Do you hear a very deep authoritative voice revealing these fantastic rationalizations to, jimmy?

      • captd, do you notice that the skeptics usually have to point to periods of no data to back up their various hypotheses? Unfortunately what data there is does support the AGW hypothesis. Certainly transport from a warmed up Atlantic is helping to speed up the loss of sea ice. If you want to separate a warmed up Atlantic from a warmed up rest of the world, go for it. The northern continents around the Arctic have warmed up especially fast, as evidenced by the (not-so-)perma-frost.

      • Don M, read about transient versus equilibrium responses. Perhaps captd can explain it to you if you ask him. The ocean has a large thermal inertia. It warms up slower. The water vapor depends on how quickly the ocean warms. Here are the warming rates of the land and ocean.
        http://www.woodfortrees.org/plot/crutem4vgl/mean:240/mean:120/plot/hadsst3gl/mean:240/mean:120
        Notice that the land has been warming twice as fast since about 1980. This fact must be very mysterious to you.

      • Danny Thomas

        Can’t have anything to do with UHI or land use as that’s historically what’s been the leading cause of change but it’s not a fossil fuel.

      • I thought UHI was put to rest. Does permafrost respond to UHI?

      • Danny Thomas

        Short memory?
        One example: http://www.sciencedirect.com/science/article/pii/S0169809515000988

        https://criticalangleblog.files.wordpress.com/2015/03/emm5.jpg?w=660&h=517

        The raw Melbourne temperature records of the Bureau of Meteorology are compared to the ACORN-Sat values. The ACORN-Sat adjustments are evaluated. This analysis shows evidence for a strong urban heat island effect.http://judithcurry.com/2015/03/22/taking-melbournes-temperature/

      • OMG! Has the Atlantic warmed up? Well, certainly not the top layer of molecules. That would cause increased evaporation. And we ain’t having none of that.

      • JimD,which AGW hypothesis,, the doubling CO2 and abusing land will cause some warming, `1C per doubling of CO2 with possibly some water vapor amplification or the OMG we are doomed one? The known is that there is about 1.6C of warming per doubling expected from a 1900 to 1930 baseline, that would be Callandar, Arrhenius take two and Manabe. Both Calladar and Arrhenius thought it would be beneficial.

        “captd, do you notice that the skeptics usually have to point to periods of no data to back up their various hypotheses?”

        https://lh5.googleusercontent.com/-ClZbSfc1vYU/VSnIversAlI/AAAAAAAANXs/zW7SCRqtjz8/w683-h455-no/arctic%2Bdjf.png

        When you reference Arctic warming you are picking the the second worst region for data coverage. Skeptics noting that have an extremely valid point. When you reference Arctic warming and just wait ’til land water catches up you are using your last hail Mary. You need to prepare your graceful, “I guess it isn’t as bad as we thought.” exit so folks can get down to business.

      • Your cheap trick won’t work, jimmy. You are trying to say the oceans aren’t warming, because the land is warming faster? Nobody has claimed that the land has warmed faster? The continents are also not devoid of water, jimmy. Tree got water. Bees got water. People got water. Have you ever heard of sweat and urination, jimmee? That’s all I can take of you mendacity for now, jimmy. You are not doing any good here.

      • Don M, are you looking at the graph I posted and suggesting that the ocean is not warming? I don’t understand where you are arguing from. The ocean is the green line. See it goes up at the end? Not as fast as the red line.

      • JimD, “I thought UHI was put to rest. Does permafrost respond to UHI?”

        UHI is generally put to rest, there is an urban heat island effect but as far as the “global” mean surface temperature anomaly goes, it is insignificant. But then Global Mean Surface Temperature isn’t all that useful is it?

        Land abuse though has similar warming issues related to changes in soil permeability, ground cover height which impacts soil temperature and water availability which impact soil and air temperature. UHI doesn’t impact permafrost but albedo darkening due to black carbon and erosion does.

        http://upload.wikimedia.org/wikipedia/commons/thumb/7/75/AralSea1989_2014.jpg/800px-AralSea1989_2014.jpg

        In fact there is a huge surface temperature difference between close cropped pasture and well managed pasture with a corresponding hydrological cycle impact. btw, close cropped pasture was found to be the cause of one of those butterfly “extinctions” until the land use changed and the butterfly resurrected itself. Eli Rabbet called it the “suburban” heat island. One of the issues with long range interpolation and kriging is that those local, “suburban” effects are smeared when building a “global” temperature product so only irrigation impact “cooling” is noted while everything else is assumed to be AGW i.e. CO2 related. That is what happens when the consensus tells you where to look.

      • You’re skimming abstracts, Don (April 11, 2015 at 7:13 pm), and it shows. We’ll let serious readers decide, eh?

      • @DM: Where is the evidence that water vapor has increased in the atmosphere as a result of increasing CO2?

        Perhaps my explanation of the process was not sufficiently clear, Don. What I pointed out was that water vapor increases as a result of increasing temperature.

        1. Do you accept that temperature has both human and natural influences?

        If not then we’re stuck right there.

        If you do accept this then we can proceed.

        2. Do you accept that temperature has remained relatively steady between 2004 and 2010?

        If not then we’re again stuck.

        3. Combining 1 and 2, would you expect water vapor to increase with increasing CO2, or stay steady because the temperature remained steady?

        If the former then again we’re stuck.

        If the latter then I don’t understand your complaint.

      • @DM: Maybe another 30-60 years. We can wait.

        Speak for yourself, Don. I very much doubt I’ll be alive then (how many people live to 101, let alone 131?). How about we wait until December 31, 2010 to look at the trend over the preceding 120 months?

      • You are playing the fool, doc. Anybody who has read one or two of my comments here can see that I know what makes water evaporate. A man of your distinction shouldn’t stoop, doc.

        Why don’t you just admit that you can’t explain why there is no evidence to support the strongly positive water vapor feedback assumption?

        More foolishness: “Do you accept that temperature has remained relatively steady between 2004 and 2010?”

        I don’t have any trouble accepting any part of the pause that is killing the cause, doc. That period is covered in the NOAA NVAP water vapor project analysis of data from 1987-2009, that was discussed at their last annual meeting. Why don’t you have a lookie see, doc:

        http://www.esrl.noaa.gov/gmd/publications/annual_meetings/2014/slides/22-140327-C.pdf

        Do you see any evidence for a strongly positive water vapor feedback response to the recent multi-decadal warming trend that we will stipulate was caused by ACO2 anywhere in that data, doc? You do understand that the AGW story is that each decade is warmer than the last. Ooops, by my 20/20 eyeballing it looks like a drop in TPW from 2004-2009. What’s up with that?

        And that ties in with the Solomon paper:

        http://www.sciencemag.org/content/327/5970/1219.abstract

        “Stratospheric water vapor concentrations decreased by about 10% after the year 2000. Here we show that this acted to slow the rate of increase in global surface temperature over 2000–2009 by about 25% compared to that which would have occurred due only to carbon dioxide and other greenhouse gases. More limited data suggest that stratospheric water vapor probably increased between 1980 and 2000, which would have enhanced the decadal rate of surface warming during the 1990s by about 30% as compared to estimates neglecting this change. These findings show that stratospheric water vapor is an important driver of decadal global surface climate change.”

        So let’s review our AGW story. Man adding plenty CO2 to atmosphere, inexorably increasing CO2 causing increasing warming, each decade warmer than the last, it’s worse than we thought, CO2 is definitely the control knob yatta…yatta…yatta. And as if that isn’t bad enough, there’s a rapid, strongly positive water vapor feedback bringing da heat that will make the warming caused by the CO2 look lukewarmish. Except NOAA says water vapor no trendy uppy. Solomon says stratospheric water vapor actually decreased by 10% after 2000, during the hottest decade EVAH! OMG! WTFreak is going on here?

        I don’t think you people are really serious about saving the world. You need to find a new hobby.

    • Rob Starkey

      “Physicists should not be confused by these random-looking quasi-chaotic fluctuations about the local climate equilibrium point, and should instead focus more on the changing energy balance equilibrium point of the climate system. They should also pay attention to the geological record that points to an atmospheric CO2 level of 450 ppm as being incompatible with polar ice caps, a level that is expected to be reached by the end of this century.”

      It is a complex system and your hubris shows in your conclusion that the climate equiliburim point is something easily determined during timespans important to humans. Since the residence time of CO2 is much shorter than once estimated, what is the rush to remove CO2 now?

      • Rob,
        Sorry, but if you want to gain some insight into this topic, you really should read Andy Lacis’s comment again. It pretty much has it all. You might also want to look up the meaning of hubris (or, potentially, try to understand what is meant by the term “ironic”). Bear in mind, this is coming from one of those pesky physicists who should aim to understand their physics better :-)

      • You don’t believe in statistical mechanics either. How quaint.

        There are parameters of complex systems with are easily determined, such as the temperature of the system.

        Injecting CO2 into the atmosphere will cause a rise in the concentration that will persist for hundreds of years and not fully decay for thousands. In that period, the CO2 in the atmosphere will be rapidly cycled between the three reservoirs: the atmosphere, the biosphere and the upper oceans, before vanishing into the deep oceans. Individual molecules of CO2 will rapidly cycle between the three fast reservoirs.

      • Rob Starkey: [Climate] is a complex system and your hubris shows in your conclusion that the climate equiliburim point is something easily determined during timespans important to humans

        “Timespans important to humans” can range from a fortnight (should I bring an umbrella to the barbecue in two weeks time?) to a century (will global mean surface temperature smoothed to a running mean of 20 years have risen by 1 °C or 6 by 2094?). That’s a range of 11 octaves (2^11 is a factor of 2048, about the number of fortnights between now and 2094).

        Are you claiming that climate a century hence is just as hard to predict as weather a fortnight hence? And if so, on what basis?

        Were you right about that, casinos would be unable to predict their long term profitability because they cannot predict whether the house or the customers will be ahead during the next hour, and therefore would be even less able to predict this during the next 24 hours, 24*365 hours, etc.

    • Certainly a thoughtful response.

      Radiative forcing from carbon dioxide has a long history before the few decades we have of actually measuring the atmosphere regularly and is well documented. So why is there any question? Beyond the fact that single point satellite observations cannot precisely enough sample short and long wave radiative fluxes, radiative forcing remains hypothetical. It is hypothetical because it pertains, by definition, to an atmosphere at rest. Such an atmosphere will never occur. The 1 dimensional radiative models that Manabe constructed, when applying radiance only, exhibited a lapse rate that was super-adiabatic. With apologies to Voltaire, if convection did not exist, the atmosphere would invent it. Manabe went on to construct a radiative-convective 1-D model with assumptions about what lapse rates might induce convection. However, convection is multi-factoral and a chaotic part of the general circulation, so general circulation models have become the tool, burying with them the parameterizations of convective exchange that may or may not represent reality.
      That is why doubt remains.

      When one examines the radiance change imposed by a doubling of CO2 on an observed atmosphere ( March 21, 2010, held ‘at rest’ ):
      https://turbulenteddies.files.wordpress.com/2015/03/rf_adjusted.png
      one finds, much like with ‘Daisyworld’, a net surplus of net radiance at the tropopause ( RF ).

      Examining this, one notices that most of the radiance difference is imposed by the lower half of the troposphere. What’s not displayed here is the change heating rate which is actually negative for the top 10% of the troposphere. Lesser net heat added to upper half ( and even cooling in the top 10% ) over greater heating in the lower half is a good recipe for restoring balance by convection.

      Further, using the same atmosphere with a CO2 doubling, with temperatures warmed uniformly by 2.4K and the humidity increased to maintain constant relative humidity results in this distribution:
      https://turbulenteddies.files.wordpress.com/2015/03/rf_warmedandhumidified.png

      For the doubled,warmed,and humidified condition, the average difference radiance at the tropopause is, as believed, closer to zero. But notice the spatial variation in this balance. The poles, given the same shape soundings, do not tolerate being warmed or humidified without cooling at a greater rate. More importantly, the radiative forcing is much lower at the tropopause than it is near the surface. The upper half of the troposphere under such conditions actually loses more energy than for the base case!
      https://turbulenteddies.files.wordpress.com/2015/03/retained_warmedandhumidified.png

      Cooling over warming is quite likely to result in increased convective exchange of thermal energy – even with the same amount of mass exchange between upper and lower tropopshere. Because convection is chaotic and sub-gridscale, no one can be sure of a precise prediction. This would not preclude warming. But additional radiative forcing appears to create its own negative feedback through convection in the same way radiative only models create an auto-convective lapse rate.

      • Convection to space does not work very well.

      • Convection to space does not work very well.

        True, but it doesn’t have to in order to provide negative feedback to water vapour.

      • Further, it’s an indication that warming of 2.4K is perhaps twice too high an estimate, because the lower half of the troposphere will have to share about half of its 2.4K worth of energy with the upper half of the troposphere just to keep balance.

        Further still, the excessive cooling of the poles remains – they just won’t sit still and be warmed and humidified.

      • David Young

        Does the missing hot spot have something to do with this?

        I have often wondered about convection modeling as a source of errer

      • Does the missing hot spot have something to do with this?
        I have often wondered about convection modeling as a source of error

        Clearly something’s not right.

        If cooling aloft leads to convection, convection transfers heat and humidity aloft, which leads to cooling. A deadly embrace.

        It’s not surprising there would be errors, though considering the narrow scale with which deep convection takes place – we can’t even measure the energy to within 17W/m^2 !!!

        How could we predict what we can’t even verify?

      • Turbulent Eddie: That is why doubt remains.

        Good post.

      • ER: Convection to space does not work very well.

        TE: True, but it doesn’t have to in order to provide negative feedback to water vapour.

        TE, would you please quantify this remarkable statement?

        If “it doesn’t work very well” by ten orders of magnitude less, then wouldn’t you say that your argument for any such impact is pretty weak?

      • ER: Convection to space does not work very well.

        TE: True, but it doesn’t have to in order to provide negative feedback to water vapour.

        TE, would you please quantify this remarkable statement?

        If “it doesn’t work very well” by ten orders of magnitude less, then wouldn’t you say that your argument for any such impact is pretty weak?

        Convection to space is very close to zero ( save for the very few molecules of atmosphere lost or gained to/from space ).

        But, what I wish I had noted while folks were still reading this thread is this:
        The Hot Spot is ( in the models ) a dynamical feature – how the atmosphere moves creates the Hot Spot ( in the models ).

        The HotSpot represents the so called Lapse Rate Feedback.
        Soden and Held found that the Lapse Rate Feedback is indeed significant. So this is an example of how convection changes the amount of energy earth radiates to space. The same thing is true of pole to equator exchange. Therefore, the efficiency of radiance to space is in part determined by the motion of the atmosphere. The Hot Spot doesn’t appear to have verified in most analyses, but that doesn’t change the fact that dynamics effect radiance.

        Soden and Held Feedbacks:
        https://www.ipcc.ch/publications_and_data/ar4/wg1/en/fig/figure-8-14-l.png

    • Don Monfort

      “It is well known that the normal human body temperature is about 310 K. Furthermore, it is also well known that a seemingly small change (up or down) in absolute body temperature by only 1% (3.1 K, or 5.6 F) would make one sicker than a dog, and, that a 2% change in body temperature (up or down by 6.2 K, or 11.2 F) will virtually guarantee a dead body.”

      That’s a cheap trick that I don’t think will impress many physicists. We are talking about climate and human beings. Right, Andy? Humans live in blazing hot deserts and on ice. Let’s call that a temperature spread of over 100F.

      • It might be a cheap trick, but it does it’s job well.

        It’s not a priori that a change of a percent or two does not have consequences of the scale we might be interested to care about. It’s now up to you to show that an average increase of two or three or four degrees C global mean surface temperature is something to not worry about.

        While you ponder your answer, note that Lacis follows his cheap trick with the less cheap example of the geological record, which strongly suggests such conditions are incompatible with polar ice.

      • Don Monfort

        “It’s not a priori that a change of a percent or two does not have consequences of the scale we might be interested to care about.”

        I didn’t say that I wasn’t interested. Neither did the author of this post.

        “It’s now up to you to show that an average increase of two or three or four degrees C global mean surface temperature is something to not worry about.”

        No it isn’t. It’s up to the alarmists. And they are doing a poor job at convincing billions of people who are not losing any sleep over climate change that something has to be done about it. Making dumb analogies isn’t going to get the job done. People dying from a 2% change in body temperature and polar bears falling from the sky, is just BS preaching to the choir. We are not impressed. You people better up your game.

      • It’s now up to you to show that an average increase of two or three or four degrees C global mean surface temperature is something to not worry about.

        Really? Global mean surface temperature – your term – rises around 3.5K every year from January to July and then falls 3.5K again.
        Are you worried because northern summer is coming?
        Why not?

        Maybe because global mean surface temperature isn’t a term in the equations of motion.

        The hemispheric gradient of pole to equator temperature DIFFERENCE does reflect an important term – temperature gradient – which does impact the jet stream and climate, though life certainly doesn’t cease in summer.

        But the temperature gradient is mostly due to the fact that earth is a rotating, revolving oblate spheroid, a fact which will not change, regardless of CO2.

      • > We are not impressed.

        Do you speak as a representative for those who can claim dirty trick expertise, Don Don?

        Speaking of cheap tricks, I don’t think the “many physicists” you alluded to in your earlier comment would be impressed with your “blazing hot deserts and on ice.” since it displaces the analogy from the effect of 1-2% to some kind of techno-pop “we humans are so great we can do anything.”

        Andy’s analogy shows that Dr. Koonin’s argument as a variation on the “CO2 is a trace gas” cheap trick.

        ***

        If you prefer an argument, it directly follows Andy’s analogy:

        From this, it should be sufficiently clear that, when viewed in absolute energy terms, the viable margin between life and death in the Earth’s biosphere is remarkably narrow – so much so that a seemingly insignificant 1% to 2% change in the total energy of the global environment will invariably result in serious disruption of the established infrastructure of life in the biosphere.

        Some physicists may argue that focusing on the analogy instead of Andy’s argument is a cheap trick.

      • Thought fer Terday:

        “Strong claims require strong evidence from said claimant.
        Up ter a researcher ter reject or nullify the null hypothesis.”

        H/t Her Excellency bts. .

      • Don Monfort

        willy, willy

        I am among the group of billions of folks who are not losing sleep over “climate change”. We are not impressed. Should I say, “I are not impressed.” Is this really something you want to make a big freaking issue out of, wee willy? I hope your former colleagues don’t find out how you are spending your precious time.

        I am very impressed with Andy’s regurgitation of the CAGW dogma. I am with him most of the way. If Andy or some other genius can find evidence that the theorized strongly positive water vapor feedback is operating, or a bout to kick in, I will join the choir. Show me where water vapor has increased along with temperature and CO2 and explain why the alleged water vapor/cloud fast positive feedback ain’t working. The pause is killing the cause.

      • Don Monfort,

        willy, willy

        I am among the group of billions of folks who are not losing sleep over “climate change”. We are not impressed.

        I often ask people: “if all the people of the world could vote YES or NO on whether they would support policies that raise the cost of energy for some intangible benefit of a possible reduction in climate damages 10 years from now, how many would vote YES?”

        I suspect less than 1% of the world’s population (70 million) would vote YES.

        The majority of the YES vote would be the loony left, rich elites who live in the concrete jungles of the developed countries – e.g. people like Willard, Joshua, etc.

      • Don Monfort

        I agree almost entirely, Peter. Except that wee willy and little joshie are very unlikely to be among the rich elites. Navel gazing and nitpicking are not lucrative preoccupations.

      • The majority of the YES vote would be the loony left, rich elites who live in the concrete jungles of the developed countries – e.g. people like Willard, Joshua, etc.

        Spoken by an impoverished swagman tramping the vast Australian outback for nigh on half a century. We elites of the dusty dirty concrete jungles, rendered loony by our great wealth, salute the clarity of their primitive pristine insights.

      • I’m just a fully funded lord
        Living in an underfunded world.
        ================

      • @DM: Humans live in blazing hot deserts and on ice.

        Don raises an excellent point. Assuming they can maintain their food supplies, humans are unique in being able to survive over a range of 100 °F. Climate-appropriate clothing and shelter are just two of their many tricks.

        Other animals are not as well equipped to survive extreme changes in latitude, though they can at least adapt to the difference between summer and winter, by adjusting “clothing” (more or less fur), by either hibernating or estivating as appropriate, by migrating to other latitudes and altitudes (depths in the case of marine life), etc.

        But hotter summers and winters require modifying these adaptation mechanisms outside their normal envelope. There is a limit to how much fur can be shed. Animals locked to a given length of either hibernation or estivation may overheat while asleep. In summer, mountain dwellers that move uphill (higher altitudes) may find themselves at a peak with nowhere to retreat to from increased heat, while those that migrate polewards (higher latitudes) in summer may encounter obstacles they’re not adapted for, e.g. coastlines, escarpments, etc.

        Littoral (shallow-water) marine life in both oceans and lakes is at constant risk of hypoxia (reduced oxygen in the water), which has markedly increased in recent decades. Here the link with warming (however caused) is less clear, though core samples, micropaleontology, etc. would appear to suggest one. California waters are currently experiencing huge die-offs in marine life due to major interruptions in food chains caused directly by hypoxia which in turn (along with California’s drought) is likely to have been caused by a “gargantuan blob of warm water” (google it), though whether global warming is implicated in this particular blob is currently fair game for scientific debate.

        Those species that feature prominently in food chains put their predators at risk when the former’s numbers are depleted, for whatever reason. This effect can propagate up any given food chain, all the way up to humans, who may find themselves challenged by food shortages. This is already happening to some extent today. By how much these shortages will increase over the next fifty years remains to be seen, but with the populations of humans and their food supplies moving in opposite directions it is not a risk to be blithely ignored.

      • Buzz Fledderjohn

        “Humans live in blazing hot deserts and on ice.”

        Not without a huge amount of assistance to moderate their body temperature!

    • A. Lacis, “Furthermore, it is also well known that a seemingly small change (up or down) in absolute body temperature by only 1% (3.1 K, or 5.6 F) would make one sicker than a dog, and, that a 2% change in body temperature (up or down by 6.2 K, or 11.2 F) will virtually guarantee a dead body. From this, it should be sufficiently clear that, when viewed in absolute energy terms, the viable margin between life and death in the Earth’s biosphere is remarkably narrow –”

      Interesting comparison. For the human body we have about 98.6 +/-0.2 F normal range with +/- 3 F being abnormal but everyone has experienced that abnormal range, so there is a “normal” range of abnormal. The Earth’s “surface” normal temperature is in the ballpark of 15 C +/- 1 C with regional variations of up to 30 C in a day and 70 C annually. The average diurnal range is about 11.5 C. So based on your analogy the Earth died some time ago because it cannot stand a 1 to 3 C variation in temperature over some hundreds of years.

    • CO2 doesn’t “precipitate” out of the atmosphere, but it certainly cycles in and out of the atmosphere.

      At least 20% each year is absorbed out of the atmosphere by vegetation, soils and oceans and converted to other Carbon molecules.

      So it does not have the stability that your “nice tidy story” implies.

      Nor has it been proven that water vapor operates at the rates that are prescribed in the Clausius Clapeyron relation. Actual water vapor compared to temperature calculations show that it is only about half of what C-C predicts.

      What happens to your global warming predictions when you take the water vapor feedback down to by 50%. Warming falls to 1.7C per doubling. Now take cloud feedback down to net Zero as well and overall warming is down to 1.35C per doubling.

      You need to prove that the feedbacks are actually operating as the theory assumes. Because they make the crucial difference between benign and potentially negative.

    • A Lacis: It is well known that the normal human body temperature is about 310 K. Furthermore, it is also well known that a seemingly small change (up or down) in absolute body temperature by only 1% (3.1 K, or 5.6 F) would make one sicker than a dog, and, that a 2% change in body temperature (up or down by 6.2 K, or 11.2 F) will virtually guarantee a dead body.

      On that we agree.

    • A Lacis: Physicists should not be confused by these random-looking quasi-chaotic fluctuations about the local climate equilibrium point, and should instead focus more on the changing energy balance equilibrium point of the climate system.

      What makes you think there is an “equilibrium” point? The climate is a high-dimensional non-linear dissipative system, and such systems lack an “equilibrium point”. It’s possible that the climate trajectory is in an attractor, and that the location of the attractor may shift, but that is very hard to calculate.

    • A Lacis: How can a physicist not comprehend that it is atmospheric CO2 that is the principal radiative forcing agent for the ongoing global warming? . . . and not be concerned that water vapor, as the climate system’s principal feedback agent, has an exponential dependence on temperature?

      How can a physicist be concerned with water vapor and not rainfall? A survey by O’Gorman et al reports that rainfall rate is calculated to increase by 2% – 7% per 1C, with the lower estimates GCM-based and the upper estimates based on regressions of rainfall rate on temperature. It is not water vapor per se that has the exponential dependence on temperature, but water vapor pressure and the Clausius-Clapayron calculated equilibrium water vapor; the actual does not adhere that closely to the calculated equilibrium.

      The Earth surface transfers energy to the troposphere via three mechanisms: advection/convection; evapotranspiration; radiation. The rates of all three processes increase superlinearly with surface temperature(Romps et al for advection/convection; O’Gorman for rainfall; the T^4 of Stefan-Boltzmann). If the Earth surface warms 1C, how much do these energy flow rates increase?

      Consider ocean surface, about 70% of the surface of the Earth: almost always there is wind-blowing and surface evaporation almost everywhere on that surface. What actually happens at the surface when the downwelling long-wave radiation increases by 4 W/m^2?

    • A Lacis | April 9, 2015 at 3:15 pm | Reply

      “A key point to keep in mind is that it is these non-condensing greenhouse gases that act as the principal radiative forcing agents of the climate system. Because of their thermodynamic, chemical, and radiative properties, CO2 and the minor GHGs are chemically slow-reacting with atmospheric lifetimes ranging from decades to many centuries. Once they are injected into the atmosphere these gases effectively remain there indefinitely by not condensing or precipitating at prevailing atmospheric temperatures as they continue to exert their radiative forcing.”

      No Andy, This is not true

      “You deliberately omit for CO2 that it has other properties that lead to massive injection and removal from the earths atmosphere every year.
      You can inject your CO2 into a test tube with no life and your observation would be slightly correct.
      The bulk of the CO2 is produced every year by degradation of dying tissues and biomass. dead plants, leaves animals, Forest fires. Respiration of all living matter.
      The bulk of the CO2 is then absorbed by plants every year
      Human production you say , adds about 5 ppm CO2 to the atmosphere a year
      Nature removes 95% of this 5 PPM every year
      After 10 years the 5 ppm is down to an invisibly small number

      There is no substitute for appealing directly to basic physics for physical insight and better understanding.”

      This is true

      Human production you say ,” adds about 5 ppm CO2 to the atmosphere a year half of which remains there for many centuries”
      Fear mongering par excellence. True* in a test tube,
      Incredibly misleading in real life.

    • Lacis
      “Of greater interest is the “unforced” variability of the climate system on decadal time scales that arises from changes in ocean circulation patterns that are effectively un-influenced by changes in atmospheric radiative forcing. ”

      For ATTP

      Physicists should not be confused by these random-looking quasi-chaotic fluctuations about the local climate equilibrium point.
      Radiative forcing takes place over the entire atmosphere, and not just at the ground surface.
      The atmospheric distribution of water vapor (and clouds) can change rapidly on a time scale of hours and days in response to changing weather conditions.
      Physicists should also appreciate the nature of the Clausius-Clapeyron relation, and the fact that it is exponential in temperature.

      There is a Stefan Boltzmann black body formula which in essence mandates radiation in must equal radiation out.
      This means that the ocean circulation patterns are influenced constantly by changes in atmospheric radiative forcing.
      Mr Lacis is being severely ingenuous in denying the effect of physics as we know it.
      Heart is not trapped for decades in deep sea currents to emerge later as new energy. [Clausius-Clapeyron ]
      For every KW that goes under another KW has to irradiate to space leaving colder surface currents over the top hence/overall balance.
      To claim Gaia can manufacture or retain heat due to circulation is appealing but not scientific.

      • angech,
        I don’t think you understood what Andy Lacis was saying.

        Mr Lacis is being severely ingenuous in denying the effect of physics as we know it.

        He isn’t.

      • ATTP ” I don’t think you understood what Andy Lacis was saying”

        Quite right.

        Andy is saying that heat, instead of having to radiate out at the rate it comes in can decide to stay, of it’s own accord in a breathe of hot air in the desert, in a deep welling current in the sea or in an air pocket in a cave.

        Then, when it wishes to reemerge, months or decades longer it adds new heat to the atmosphere.

        In the meantime the rest of the earth, sans this Hansen heat, somehow forgot to radiate out the energy that came in ????

        This is not physics as we have been taught.
        This is wishful thinking

      • Andy is saying that heat, instead of having to radiate out at the rate it comes in can decide to stay, of it’s own accord in a breathe of hot air in the desert, in a deep welling current in the sea or in an air pocket in a cave.

        No he did not. What he was suggesting was that if a pocket of cold water were to come to the surface of the ocean, it would – obviously – reduce the average outgoing flux (since it would reduce the average surface temperature slightly). We would therefore be out of energy balance until this pocket had warmed to the point where we had returned to energy balance. Hence variability, but a tendence to always tend back towards the equilibrium point.

      • If exactly the same amount of LW has to exit as SW entered, the planet could not change temperature, and it clearly does.

        Per Stephen et al – you remember that profound landmark guy – says that the surface of the earth has 165 Wm-2 of SW absorbed and net 52.4 Wm-2 LW plus 24 Wm-2 of net sensible heat plus 88 Wm-2 of net latent heat exiting.

        This leaves .6 Wm-2 behind on the surface, and he even gets specific about where impost of it is being left. He says most of it is being left in the southern oceans.

        Which means that profound landmark skeptic guy agrees 100% with Andy Lacis.

      • ATTP,

        if a pocket of cold water were to come to the surface, or if a miracle were to happen, blah, blah, blah.

        There is no energy balance. None. In the absence of Sun, the surface cools. In sunlight, the surface warms. No balance, no conservation of energy as far as the Earth is concerned.

        No equilibrium, even. The Earth is cooler than it was, and the Sun will eventually go out, so to speak. The final temperature of all matter in the absence of external energy is obviously 0K, or ultimate equilibrium temperature if you will.

        At the heat death of the universe, given physics as she are currently known, the final equilibrium temperature is currently undefined.

        Maybe you can educate us all, and tell us why energy must be balanced in respect to the surface temperature of the Earth. Is the energy balanced in Winter and Summer? Day and night? What do you mean?

      • Upwelling and downwelling could be miracles. Sideways welling is in the bible.

      • JCH,

        What the heck are you talking about? What’s the Bible got to do with AGW? Please explain!

      • Zipper,
        I think it’s called “responding in kind”.

      • Dr. Rice, what about the water vapor? A smart guy like you should be able to find evidence of the powerful positive and rapid water vapor feedback. Have you read Solomon 2010? Why would she make that stuff up?

        You would like to be the one to come up with a water vapor hockeystick, but I am guessing you know that it ain’t happening. Water vapor has become the third rail for CAGW dogmatists. They will chat your head off about the Keeling curve, radiative physics, paleo proxy prattle, but don’t ask them about the missing water vapor. No savvy. No speaky. No integrity.

      • Oops, sorry, those are the same link. (My browser fooled me by going to the wrong comment.)

  47. “A NEW LITTLE ICE AGE HAS STARTED” in the title of a book I am about to publish. I am looking for someone familiar with solar science who could write a short, say 4 line review of this popular book, which is based on the research of Dr. Habibullo Abdussamatov, Dr. George Kukla, Dr. Tim Ball, Dr. Ian Plimer, Dr. Roy Spencer, Dr. Rhodes Fairbridge, Dr. Jan-Erick Solheim, Dr. Jack Sauers, Dr. Ross McKitrick, Dr. Edward Wegman, Steve McIntyre, David Archibald, Donna La Framboise, and Brian Fagan.
    I need to post the review on my Amazon page. The trolls will be out in full force, and I want some protection!!

  48. Human influence may be small but the problem is that even small changes in the climate could have serious consequences for society Sea level rise could lead to costly remedial actions. More frequent droughts in could lead to serious consequences for the economics of farming in certain area.

    • richardswarthout

      Tom

      Serious? Like NYC going underwater? Polar bear extinction? Colorado ski resorts closing? Ya De Ya.

      Richard

    • The problem is that nobody knows for sure what the effect of human influence might be, and what changes might ensue.

      For example, if GHG levels were to be held at their present levels, would there be any reduction in droughts, floods, hurricanes, or changes in climate generally?

      Would sea levels drop, and require new port facilities to be built all around the world? It all seems a bit odd.

  49. Turb Ed,
    Don’t think the orbital theory suggests Greenland de iced for 100,000 years. Greenland ice cores in Earth’s Climate, past and future by W Ruddiman, imply for O18 levels that we are warm now and may go into a renewed cold spell. Overdue now in orbital mechanics. So the whole reponse is unsettled. He thinks we may already have delayed an ice age due to agriculture.
    Scott

  50. There has been a global warming debate. The debate is not over and I question whether any of it has advanced science.

    So far I see only one obvious conclusion. There are two sides to the debate.

    On one side — populated primarily by Left-thinking folks — is the belief that mere simulation of reality is, reality.

    On the other side, we see simulated reality as mere shadows on the walls of Plato’s prison cave.

    The real question has become something different — no longer simply whether humanity is causing global warming. Rather, need ask why Left wants so badly for this to be so: that humanity is guilty of causing the climate to change because, so long is that is what they really want, facts won’t matter.

    • Ancient shamanic practice: Seize power with guilt and fear.
      ============================

  51. I suppose we could say 2% is also not a big deal, it’s only an ice-age.

  52. I genuinely appreciate Andy Lacis’ contribution (he *does* try), although the heart of it seems to be another attempt at explaining the current atmospheric temperature stasis – a “blob” of upwelled very cold deep ocean water is currently equilibrating with ambient surface temperature and when that’s done, the icecaps will continue their inexorable melt

    From this, one can infer that that the abysses are not actually heating (another stasis explanation). For this I’m grateful …

    As for no pole ice if CO2 levels > 450ppm, that’s just wrong. There have been glacials as measured from the ice cores where CO2 > 800ppm

    The issue as I understand it with increased water vapour from CO2-induced 1oC temperature rise is that there has been very little to almost no increase in global humidity as measured by satellite since 1979

    I’m an empiricist.

  53. Interesting post and discussion.

  54. I think the increases in amount of radiation the CO2 can absorb in its wavelength absorption bands is exaggerated even in the graph you show. The band widening mechanism is an alarmist’s false hope. Essentially all the radiation from the earth’s surface that can be absorbed in the GHG wavelength range is being absorbed in the thick layers of GHG absorbing gases now.

    Either way–with the scenario in the graph you show, or with full saturation–this brings up the point of how much changes in the earth’s emissivity, due to land-use changes, can affect the global temperature. Since the current CO2 and H2O are absorbing nearly all the radiation reflected from the earth’s surface in their absorption ranges now, a small change in the emissivity of the earth–which would cause a change in the amount of radiation sent back to the atmosphere–will have a much, much greater influence on global temperature than changes in the amount of CO2 in the atmosphere.

    • The mechanism for band widening is very simple. The rotational transitions on the sides of the bands correspond to weak absorptions. So while a photon at a wavelength in the middle of the band would travel on average only a few meters/yards before being absorbed, one on the edge might travel twice as far and one on the far edge might travel 100 meters/yards before being absorbed.

      Now double the amount of CO2 in the atmosphere and the absorption distance falls so that more of the photons emitted from the surface are absorbed along the edges before reaching space.

  55. Steven E. Koonin wrote:
    “First, it means precision observations are required to see the climate response.”

    “Observational determination of surface radiative forcing by CO2 from 2000 to 2010,” D. R. Feldman et al, Nature 519, 339–343 (19 March 2015)
    http://www.nature.com/nature/journal/v519/n7543/full/nature14240.html

    • Danny Thomas

      David,
      FYI only (in case you’d not see this previously).
      I asked why the study produced in 2015 had data ending in 2010 and asked. The response: “Here is the response from Dan Feldman, lead scientist on this research:

      Good question. We need a lot of different datasets in order to do this analysis. Some of those datasets currently do not extend past the beginning of 2011. Specifically the dataset with which we screen for clouds (called Active Remote Sensing of Cloud Locations). However, that dataset should be updated and available soon and extend to the present, so we can extend the analysis forward in time. “

  56. Pingback: Andy Lacis responds to Steve Koonin | …and Then There's Physics

  57. David Springer

    Koonin writes:

    For example, human additions to carbon dioxide in the atmosphere by the middle of the 21st century are expected to directly shift the atmosphere’s natural greenhouse effect by only 1% to 2%.

    An even simpler indication of the percentish influence is to note that a 3 C mean global surface temperature increase on a base of 288 K is also about a 1% effect.

    That’s unconscionably stoopid or unconscionably dishonest. Either way it exposes Koonin as not credible.

    The greenhouse effect doesn’t raise earth’s temperature from absolute zero to 288K. It raises it from 255K to 288K. Thus 3C mean global surface temperature increase is 10% not 1%.

    How Curry could present Koonin as an educated voice within APS climate statement committee makes her judgement questionable as well.

    In fact if Curry doesn’t weigh in on this, which Eli Rabbett also mentioned as beyond contempt on Koonin’s part then she’s being silent for political reasons. Being willing to let something pass by without correction like makes Curry look very tribal, willing to compromise the facts based in math and science for political considerations. BOO … HISS. Bad scientist.

    • Given that the global temperature swing in the last million years or so is about 6 C and over the last 10 million years about 10 C, a 3 C change is somewhere between 30 and 50 per cent.

      http://upload.wikimedia.org/wikipedia/commons/f/f5/All_palaeotemps.png

      • So, what are the time resolutions back in the 4th and 5th decades?

        log-scale? Silly wabbit…bwevity doesn’t give the warm fuzzies. Ambigiwous, vewy, vewy twicky gwaf.

      • David Springer

        Yabbut 3C warmer was very good times for living things while 3C colder really sucked for living things. The primary producers in the food chain don’t grow well in ice. I’m tree hugger not an ice hugger.

      • These various percentages would appear to be influenced by what is allowed to go in the denominator.

        A list of ground rules constraining the permissible denominators would be appreciated.

      • That Paleotemperature chart from Eli is not scaled properly at all.

        The Eocene temps were only about +6.0C. The Cretaceous and Permian were 4.0C or 5.0C warmer than the Eocene for example.

        The entire period from 3 million to 67 million years ago, needs to be scaled downward by about 60%.

        The author of this chart conceded that this period is wrong (and he should now revise it so that people don’t keep posting it all over).

      • So, it’s still a variation of about 10 K.

      • David Springer

        @Pratt

        According to consensus narrative what clearly can’t go in the denominator is greenhouse warming of 288K. The sun is responsible for the first 255K leaving 33K for the greenhouse effect.

    • Actually, the valid comparison is with natural variation and the starting point is the prevailing global temperature before it was being affected by human CO2 emissions. Let’s call that 1950.

      Rabetticus halpernicus says natural variability is 10C. And according to his lovely chart “Temperature of Planet Earth” in 1950, we had been in a long cold spell. So 3C should be good for us. If this damn pause ever ends, we will be rolling in the high clover.

  58. On a yearly basis, that is.

  59. A Lacis | April 9, 2015 at 3:15 pm | Reply

    “A key point to keep in mind is that it is these non-condensing greenhouse gases that act as the principal radiative forcing agents of the climate system. Because of their thermodynamic, chemical, and radiative properties, CO2 and the minor GHGs are chemically slow-reacting with atmospheric lifetimes ranging from decades to many centuries. Once they are injected into the atmosphere these gases effectively remain there indefinitely by not condensing or precipitating at prevailing atmospheric temperatures as they continue to exert their radiative forcing.”

    No Andy, This is not true

    “You deliberately omit for CO2 that it has other properties that lead to massive injection and removal from the earths atmosphere every year.
    You can inject your CO2 into a test tube with no life and your observation would be slightly correct.
    The bulk of the CO2 is produced every year by degradation of dying tissues and biomass. dead plants, leaves animals, Forest fires. Respiration of all living matter.
    The bulk of the CO2 is then absorbed by plants every year
    Human production you say , adds about 5 ppm CO2 to the atmosphere a year
    Nature removes 95% of this 5 PPM every year
    After 10 years the 5 ppm is down to an invisibly small number

    There is no substitute for appealing directly to basic physics for physical insight and better understanding.”

    This is true

    Human production you say ,” adds about 5 ppm CO2 to the atmosphere a year half of which remains there for many centuries”
    Fear mongering par excellence. True* in a test tube,
    Incredibly misleading in real life.

    So good I thought I would post it twice

    • After 10 years the 5 ppm is down to an invisibly small number

      Well, the people who work on this disagree. You can play around with this carbon cycle simulator. Plug in 10 Gton C for the transition CO2 spike and see how long it takes to returns to the pre-spike value. It’s a good deal longer than 10 years. I suspect that you’re confusing the time it takes for an individual CO2 molecule to move from one reservoir to another (atmosphere to biosphere, for example) with the time it would take for an increase in atmospheric CO2 concentration to return to the pre-increase value.

      • Thanks ATTP Good graph
        Looked at it, a bit complex. Little windows pop up for explanations which is good.

        This worried me

        “Plants, if they exist on land, affect the rate of weathering by pumping carbon dioxide into the soil” with a yes/no box.

        4 criticisms.
        Plants do exist.
        They do not explain how they assigned a value to this effect
        The CC simulator however is set up to completely exclude their effect
        [ choice of no plants as an answer]
        There are a lot of them in the sea as well which they conveniently forget about [It is estimated that marine plants produce between 70 and 80 percent of the oxygen in the atmosphere. Nearly all marine plants are single celled, photosynthetic algae]
        Hence they are ignoring 80% of the effect plants have.

        My criticism of Lacis et al has not been rebuffed.
        Your CC simulator and his explanation both work for injecting CO2 into a sealed test tube earth with no plants and no CO2 turnover.
        There is massive CO2 turnover.

        Further the extra 10 Gton C is not the human component.
        The human component is 10 out of 400 or 2.5 percent of the total CO2
        so only 0.25 GT of the 10 GT that is decreasing in the simulation is due to the injected CO2. 9.75GT is turned over by the vegetation of the earth in 1 year alone and 97.5% reduction of the remainder occurs each year after that.
        I guess “an individual CO2 molecule” might be present some years later but it would be of no consequence.

      • and Then There’s Biology.
        ====================

      • [ choice of no plants as an answer]

        Select “yes” then?

        Your CC simulator and his explanation both work for injecting CO2 into a sealed test tube earth with no plants and no CO2 turnover.

        No, the CO2 turnover is included.

        There is massive CO2 turnover.

        Again, I think you’re confusing the fast fluxes (between the atmosphere, oceans, biosphere) and the slow fluxes (sequestering carbon in the deep ocean/rocks/etc). The rate at which we would reduce an increase in atmospheric CO2 is related to the slow fluxes (i.e., the rate at which we essentially remove carbon from the upper ocean/atmosphere/biosphere) not the rate at which it cycles through the ocean/atmosphere/biosphere system.

        Further the extra 10 Gton C is not the human component.
        The human component is 10 out of 400 or 2.5 percent of the

        Well, my point was just an illustration, but we are injecting 10Gton C per year into the atmosphere, therefore a 10Gton C transition spike is a fair representation of what we do annually. What the simulation is showing is that it would take hundreds of years to completely remove the influence of a single year of human emissions.

      • There is no fantasy biology.

      • Again, I think you’re confusing the fast fluxes (between the atmosphere, oceans, biosphere) and the slow fluxes (sequestering carbon in the deep ocean/rocks/etc). The rate at which we would reduce an increase in atmospheric CO2 is related to the slow fluxes (i.e., the rate at which we essentially remove carbon from the upper ocean/atmosphere/biosphere) not the rate at which it cycles through the ocean/atmosphere/biosphere system.

        No
        You continue to confuse test tubes of CO2 with real life CO2 WHICH IS REMOVED FROM THE ATMOSPHERE EVERY YEAR IN MASSIVE AMOUNTS.
        The CO2 retention rates Lacis describes are only obtainable in a dry , sterile unreal world.
        CO2 is constantly turned over in large amounts and is removed from the atmosphere and sequestered in larger amounts when they occur by the biologic process you ignore.
        Sorry about the italics , Capital lock by mistake and too tired to change it.

      • @ATTP: The rate at which we would reduce an increase in atmospheric CO2 is related to the slow fluxes (i.e., the rate at which we essentially remove carbon from the upper ocean/atmosphere/biosphere) not the rate at which it cycles through the ocean/atmosphere/biosphere system.

        Doesn’t this depend in an essential way on vegetation biomass not increasing in response to increased CO2?

        Preindustrially, plants absorbed some 120 GtC via photosynthesis during sunlight hours, returning half via respiration 24/7 and the other half to soil. If increased CO2 results in say a 10% increase in plant biomass, plants will absorb a further 12 GtC, returning 6 to the atmosphere and 6 to soil for a net drawndown of 6 GtC per year.

        Some of that 6 GtC sent to the soil may find its way back into the atmosphere. However essentially all of it would have to do so if our contribution to atmospheric CO2 is to remain high for thousands of years after we stop emitting it, as David Archer, Susan Solomon, and Ray Pierrehumbert would have it. This seems to me to be a big question mark; our own Mark Jacobson for example, author of Fundamentals of Atmospheric Modeling, expects a CO2 residence time on the order of one or two centuries.

        Given how aggressive plants can be about taking advantage of extra CO2, maintaining say 1000 ppm for thousands of years sounds like a pretty tall order.

      • VP,

        Doesn’t this depend in an essential way on vegetation biomass not increasing in response to increased CO2?

        Well, yes, and I think it is increasing. I’m not sure what would happen were we to simply stop emitting.

        Preindustrially, plants absorbed some 120 GtC via photosynthesis during sunlight hours, returning half via respiration 24/7 and the other half to soil. If increased CO2 results in say a 10% increase in plant biomass, plants will absorb a further 12 GtC, returning 6 to the atmosphere and 6 to soil for a net drawndown of 6 GtC per year.

        As I understand it, you add 120 GtC via photosynthesis, half being returned via respiration and the other being returned via microbrial respiration and decompostion. So, even preindustrially, it was 120 GtC in and 120 GtC out. The amount actually sequestered was only a small fraction. In fact, in preindustrial times the system was in balance with the volcanic emissions, which are about 100 times smaller than ours today (0.1GtC). So the amount actually sequestered was probably around 0.1GtC per year.

        Given how aggressive plants can be about taking advantage of extra CO2, maintaining say 1000 ppm for thousands of years sounds like a pretty tall order.

        It’s not so much 1000ppm for thousands of years. It’s more like 1000 years to go from 1000ppm to around 500ppm You can play around with this Carbon cycle simulator. As much as I can see that increasing biomass could draw it down faster, I don’t really have a sense as to how much more you would need if you wanted to quickly reduce the concentration. It’s possible, I guess but according to this, the terrestrial biosphere has aout 2000-3000 GtC. So it would seem to need to be substantially larger if it were to rapidly reduce atmospheric concentrations from 1000 ppm back to pre-industrial levels.

      • angech,

        You continue to confuse test tubes of CO2 with real life CO2 WHICH IS REMOVED FROM THE ATMOSPHERE EVERY YEAR IN MASSIVE AMOUNTS.

        You continue to forget that – in real life – CO2 is also returned to the atmosphere in massive amounts.

        CO2 is constantly turned over in large amounts and is removed from the atmosphere and sequestered in larger amounts when they occur by the biologic process you ignore.

        I’m not forgetting that. As I was pointing out to Vaughan, the biosphere absorbs around 120GtC per year and then returns 120GtC per year (half via respiration and half via bacterial respiration and decompostion).

        You seem to be ignoring that it’s a cycle. There is CO2 coming out of the biosphere and oceans as well as going in. In preindustrial times the system was in equilibrium with the volcanic emissions. In other words, the amount that was actually sequestered and removed from the fast cycle was the same as being emitted by volcanoes each year. Volcanoes emit around 0.1 GtC per year. It’s this slow cycle that would control how atmospheric concentrations would recover, not the fast one.

      • @ATTP: So, even preindustrially, it was 120 GtC in and 120 GtC out.

        That’s fine for equilibrium. Furthermore if you ramp up CO2 quickly and the plants respond with 10% growth equally quickly, it is reasonable to assume that the 60 GtC of respiration (from various sources on the carbon cycle) will increase at essentially the same rate as the photosynthetic downdraw of CO2.

        But why should the increasing amount of CO2, starting at 60 GtC, that enters the soil be returned to the atmosphere just as quickly? There could be a substantial lag, and furthermore some of it may be sequestered more permanently.

        While I’m not claiming this is what happens, where is the proof that it doesn’t happen?

      • Lacis said
        “Because of their thermodynamic, chemical, and radiative properties, CO2 and the minor GHGs are chemically slow-reacting with atmospheric lifetimes ranging from decades to many centuries. Once they are injected into the atmosphere these gases effectively remain there indefinitely by not condensing or precipitating at prevailing atmospheric temperatures as they continue to exert their radiative forcing.”

        ATTP disagrees with Lacis and agrees with me
        “I’m not forgetting that. As I was pointing out to Vaughan, the biosphere absorbs around 120GtC per year and then returns 120GtC per year (half via respiration and half via bacterial respiration and decomposition). ”

        So the gases do not and cannot remain there indefinitely.
        They move in and out of the atmosphere at massive rates of turnover.

        Vaughan Pratt sums up one of the difficulties well
        “Pre industrially, plants absorbed some 120 GtC via photosynthesis during sunlight hours, returning half via respiration 24/7 and the other half to soil. If increased CO2 results in say a 10% increase in plant biomass, plants will absorb a further 12 GtC, returning 6 to the atmosphere and 6 to soil for a net drawdown of 6 GtC per year.”

        You cannot discuss Carbon lifetimes in the atmosphere as if it is an unchanging block of chemicals in a neutered test tube.

      • VP,

        While I’m not claiming this is what happens, where is the proof that it doesn’t happen?

        Well, I don’t know, but am not sure how it could. To date our emissions are just over 500 GtC, about one-quarter of which has gone into the terrestrial biosphere (150 GtC). So, this has presumably increased the mass of the terrestrial biosphere by about 10%. This presumably also means that it will increase respiration from plants, bacterial respiration and decomposition. It seems hard to me that it can somehow increase the uptake without also increasing the emission back into the atmosphere.

        If, however, you look at the carbon calculator, the rate of sequestration is higher when the atmospheric concentrations are higher, so presumably this does mean that you can have faster sequestration when the total concentrations are higher. For example, starting at 1000ppm, the concentration drops at around 3ppm per year for the first 50 years. However, this slows as the concentration drops and when the concentration reaches 500ppm, it is back to a few tenths of a ppm per year.

      • angech,

        ATTP disagrees with Lacis and agrees with me

        This is irritatingly deceptive.

        So the gases do not and cannot remain there indefinitely.
        They move in and out of the atmosphere at massive rates of turnover.

        Andy Lacis didn’t say this doesn’t happen. He’s referring to the time it would take for an enhanced concentration to decay, not the time it would take for a specific gas molecule to leave the atmosphere. You need to understand what someone is saying before claiming that what they’ve said is wrong.

      • …and Then There’s Physics | April 12, 2015 at 4:28 am |
        “angech,ATTP disagrees with Lacis and agrees with me
        This is irritatingly deceptive.” I apologise

        “So the gases do not and cannot remain there indefinitely.
        They move in and out of the atmosphere at massive rates of turnover.”

        “Andy Lacis didn’t say this doesn’t happen.”

        meaning Andy said this does happen?

        No, he made no mention of CO2 turning over at rapid rates due to chemical[biochemical] reaction. None at all.

        He talks about the rate of CO2 decay in the equivalent of a test tube not a real atmosphere.
        “Ïnject some CO2 which is chemically slow-reacting he says.”

        But you said in contrast ” the biosphere absorbs around 120GtC per year and then returns 120GtC per year (half via respiration and half via bacterial respiration and decompostion).

        This is not, repeat not chemically slow reacting CO2 in any way, shape or form

      • angech,

        No, he made no mention of CO2 turning over at rapid rates due to chemical[biochemical] reaction. None at all.

        He said Once they are injected into the atmosphere these gases effectively remain there indefinitely by not condensing or precipitating at prevailing atmospheric temperatures as they continue to exert their radiative forcing. Now, I can’t say for certain that what he meant but that he meant that these specific molecules remain in the atmosphere, rather than the enhancement remains, is so ridiculous that I can’t believe it is what he meant.

        Here’s something you should consider. Certainly there are some things that I don’t spell out in exquisite detail when commenting on a climate blog because I assume that they are so obvious that everyone who regularly comments there realises this. I am often mistaken. It’s my guess that Andy Lacis did not consider that anyone would be silly enough to think that he meant that these specific molecules would remain in the atmosphere. It seems that he was mistaken.

        This is not, repeat not chemically slow reacting CO2 in any way, shape or form

        I know, I didn’t say it was slow. I said it was fast. That’s the point. The decay of an enhancement in concentration depends on the slow processes, which are much slower than these fast processes.

      • David Springer

        More ignorance from the ATTP drone on carbon cycle.

        The fact is that only half the CO2 humans release into the atmosphere each year stays in the atmosphere. A growing sink pulls half of it back out each year.

        Should humans stop releasing CO2 the sink will keep going but will instead start shrinking every year instead of growing, effectively removing CO2 at the same rate it had been growing before.

        That is how equilibrium systems far out of equilibrium work. The carbon cycle is an equilibrium system. Each year we drive it further from equilibrium and each year the system responds by increasing pressure to return to equilibrium. Absent humans driving it out of equilibrium it will return to equilibrium at the same rate it was driven out.

      • Well, I’ve seen Pekka doubt sequestration of heat in the deep, and I’ve seen Vaughn doubt the long persistence of the anthropogenic aliquot of CO2.

        What else do the alarmists have?
        ======================

      • David Springer

        …and Then There’s Physics | April 12, 2015 at 4:28 am |

        “This is irritatingly deceptive.”

        So are you. Karma in action. LOL

      • Also, David, that ‘growing sink’ is taking a larger percentage of human emissions every year. Don’t look now, but there is a big clue to the power of the biome.
        ===============

      • JCH
        great picture. Judy should keep it and use it every few months at the right times.
        ATTP, thanks for the facts you have provided, very helpful. We will have to disagree on the meaning unfortunately.

      • Vaughn Pratt asks

        While I’m not claiming this is what happens, where is the proof that it doesn’t happen?

        A good place to start is the AR4 WGI Chapter 7.3 Cmon Vaughn, this is an obvious thing to study and yes, it has been. There are all sorts of FACE experiments and information from them if nothing else, and no, miracles don’t happen, and the information is absorbed into models at all scales.

        Now some, not Eli to be sure, might point out that if CO2 is the magic plant elixir then we should be growing crops next to coal power plants.

      • OMG! The wittle wabbette is tussling with the big kangaroo, whose rateaprofessor scores tower over those of the wabbette. You two should stop fussing and put your giant brains together to come up with an excuse that explains why the big strong rapid positive water vapor feedback has shown no signs of actually causing a detectable rise in water vapor in the atmosphere. It can’t be because there hasn’t been a multiple decade warming trend with each decade warmer than the last and year after year of WARMEST YEAR EVAH!

    • Respiration recycles carbon, it does not create it.

      • “Respiration recycles carbon, it does not create it.”
        I presume by carbon you are meaning CO2.
        David every bit of fossil fuel is “Carbon” that has been locked away out of circulation but was created by photosynthesis in the past.
        “Photosynthetic organisms also convert around 100–115 thousand million metric tonnes of “carbon” into biomass per year.” Wiki
        I would say that fossil fuels existing disprove your point entirely in that life does create more and more stores of CO2

      • What I think is really cool is that even vulcanism is recycling old carbon, bringing it from carbonates.

        CO2 is a precious resource, which the biome and the sun conspire to nearly permanently sequester. If man did not exist, it would be useful to invent him.
        =================

  60. @AL: An even simpler indication of the percentish influence is to note that a 3 C mean global surface temperature increase on a base of 288 K is also about a 1% effect.

    How does 1% follow? I would believe a 0.25% effect on temperature resulting from a 1% increase in radiative forcing, on the ground that

    (a) radiation is proportional to the fourth power of temperature, and

    (b) 1.0025^4 = 1.01 (to five digits of precision).

    • An even simpler indication of the percentish influence is to note that a 3 C mean global surface temperature increase on a base of 288 K is also about a 1% effect. … – Steven E. Koonin

    • There is an analogy in physics that does give a 1% temperature response to a 1% radiative forcing change. This is a system that adjusts to forcing changes mostly by convection, but the problem is that it has to be convecting to space maintained near 0 Kelvin. In that system net radiation provides the heat, and sensible heat flux which is proportional to the temperature difference between the surface and space removes it from the surface. You get this proportionality if the net radiation change is balanced by the heat flux change. Not a real system, but a system with those properties.

    • “I would believe a 0.25% effect on temperature resulting from a 1% increase in radiative forcing, on the ground that”

      Ummm…. it doesn’t work like that.

  61. ATTP is quite right (at least for sufficiently small values of dF and dT; only in the infinitesimal limit is the factor exactly 4). And I would also be fine with “Ummm…. it doesn’t work like that.” if you said how it does work. Simply saying 1% of radiative forcing will warm the planet by 1% is not enough.

  62. Angech2014

    CO2 is constantly turned over in large amounts and is removed from the atmosphere and sequestered in larger amounts when they occur by the biologic process you ignore.

    Ah, let’s not ignore biological processes, then. The seasonal removal and return of CO2 to the atmosphere is seen in the Keeling Curve. The prevailing hypothesis is that removal by photosynthesis exceeds respiration + emissions during the northern hemisphere spring and summer, but that is reversed in the NH autumn and winter.

    The seasonal variation is about 6ppm. Compare that with the secular trend of +85ppm since 1958. Clearly, biological sequestration isn’t keeping up. Thanks for drawing our attention to that, Angech2014.

    • The percentage of new emissions which are being sequestered has been rising, in the face of rising magnitude of emissions. ‘Splain that.
      ====================

    • Mal Adapted | April 12, 2015 at 4:36 pm | Reply
      “Angech2014 CO2 is constantly turned over in large amounts and is removed from the atmosphere and sequestered in larger amounts when they occur by the biologic process you ignore.
      Ah, let’s not ignore biological processes, then”

      Quite right, thanks for agreeing with me and providing the graph

      “The seasonal removal and return of CO2 to the atmosphere is seen in the Keeling Curve”
      The graph shows the amount of CO2 in the atmosphere, it shows a variation in the amount of CO2.
      It says nothing about the amount or rate of CO2 being removed and returned by the biological processes which is very fast by the way.
      Which disproves Lacis’s argument.
      CO2 does not persist for long periods of time in the atmosphere.
      ATTP is trying to argue that there will be more CO2 overall because of extra human input but this is a different argument to that of Lacis who is pretending that CO2 is practically inert and will stay in the atmosphere for thousands of years once introduced.
      I appreciate ATTP’s line of argument much more than that of Lacis.

      • angech,
        I would be willing to bet that Lacis’s argument is actually the same as my argument. What he was – I think – trying to do was to distinguish between water vapour, that precipitates easily, and CO2, which does not. In other words, if we were to pump lots of water vapour into the atmosphere so as to increase the concentration, it would lead to some warming through enhancing the greenhouse effect. However, any small perturbation in temperature would cause the excess water vapour to precipitate (rain/snow), reducing the enhancement to the greenhouse effect, leading to further cooling and precipitation, and ultimately returning the water vapour concentration to what it was prior to injecting the excess water vapour into the atmosphere. This would happen quite quickly (days, I think).

        CO2 does not work like this. If we inject lots of CO2 into the atmosphere, it participates in the carbon cycle, but the enhancement in atmospheric concentration does not respond (much, at least) to temperature flutuations. Hence, once we introduced an enhanced atmospheric concentration, it will remain enhanced for a long time and decay very slowly. The rate at which it decays will be set by the rate at which we remove CO2 for the atmosphere/ocean/biosphere system through weathering, the sequestering of biological matter in the soils (peat, for example), and the sequestering of CO2 in the deep ocean. This is slow.

      • CO2 removal is happening faster than the alarmists expected, and is accelerating.
        ==========

      • “It says nothing about the amount or rate of CO2 being removed and returned by the biological processes which is very fast by the way.
        Which disproves Lacis’s argument.”

        Actually, it does show 6ppm of uptake, and the same amount returned to the atmosphere annually. It also shows that, while 400 million years ago carbon was being removed from the atmosphere and sequestered by plants, currently atmospheric CO2 is increasing because humans are digging up that fossil carbon and burning it. Since CO2 is a Tyndall (i.e. “greenhouse”) gas, increasing CO2 increases radiative forcing, leading to higher temperatures near the earth’s surface. Hence Anthropogenic, Global, Warming. Scientists have understood that for more than 100 years. By the present day, climate has already changed beyond what human society has known for thousands of years.

        Meanwhile, a few argumentative humans trade gibes on the Internet. Regardless, no sophistry can explain AGW away, because the carbon cycle doesn’t care about politics. Do you get it yet, Angech2104?

      • Well, climate always changes so we are always beyond what it was before, but it is certainly not warmer than the Holocene Max.

        Sure, I’ll give you AGW, but the whole argument is about how much.

        If it’s not much, then no catastrophe, and no one is showing me much.
        =====================

  63. “The percentage of new emissions which are being sequestered has been rising, in the face of rising magnitude of emissions. ‘Splain that.”

    Document that.

    • You do not get it, do you?
      Try the new, modern, paleo argument.
      More CO2, more biomass, more sequestration.
      Look at the fossil record. We did not get oil and coal produced in low CO2 environments.

    • Can’t link but it is documented. Big mystery for the alarmists.
      ===========

      • Well, that is an amusing study. Find lots of explanations for why increased plant productivity is not from increased CO2. Hilarious.

        But it doesn’t address the point mal adapted wants documented.
        =============

      • This study is a marvelous example of the sort of research that has sustained the delusions of catastrophe. It was designed to impact a powerful skeptic talking point, and all it does is illustrate the ignorance, or disingenuousness of the authors.

        Steve’s is the place to watch this phenomenon in action. Study after study designed to attack a skeptic talking point gets blown up in days if not hours.

        This is why I say that skepticism is driving climate research today.
        =====================

      • So is plant productivity increasing or not? Your links seem to disagree.
        ==============

      • Heh, the one claims increased productivity from more rainfall, more warmth and the other claims decreased productivity from drought.

        See? Each aimed at a skeptic talking point. Both apparently garbage.
        ================

      • I think this may be some of the dynamic that causes the fragility of the bubble others have spoken about on some other thread. In the effort to sustain the bubbling of the narrative, fundamentally uncurious(because of being in support of the narrative) work is done to keep expanding the bubble. It is curiosity that pricks the bubble.

        When will they ever wonder? When will they ever wonder?
        ===============

      • Biomass is subject to a lot of variability.

        Almost all of the land uptake of anthropogenic emissions is done by forests. Grass, food crops, weeds, etc., pretty much net out.

      • Well, JCH, I somewhat doubt that, but can’t prove it wrong.

        and Then There’s Oceans.
        ===================

      • thanks for taking the heat of Kim

        JCH
        “Almost all of the land uptake of anthropogenic emissions is done by forests. Grass, food crops, weeds, etc., pretty much net out.”

        So ignore the biomass in the sea?
        Look at Hadrian’s wall, 20 foot under sequestered biomass,
        go to a river bank and look at all the buried logs in the bank.
        Look at coal and oil
        Not all CO2 storage is slow

      • I’m not ignoring it. I do not think rising levels of CO2 in the atmosphere can provide much direct CO2 to plants in the water.

        Trees make wood. Freeman Dyson didn’t propose engineering carbon eating lettuce.

  64. Pingback: Weekly Climate and Energy News Roundup #176 | Watts Up With That?

  65. JCH | April 14, 2015 at 12:16 am |
    “I’m not ignoring it.
    I do not think rising levels of CO2 in the atmosphere can provide much direct CO2 to plants in the water
    Trees make wood. Freeman Dyson didn’t propose engineering carbon eating lettuce.”

    I’m not ignoring it???

    sorry JCH

    carbon [dioxide] is turned over by all living organisms.
    There is more biomass and CO2 in the ocean than on land.
    You are willfully ignoring science, biology, maths and physics.
    Your response only strengthens the case against.

    • Well, according to this about 1000 billion tonnes of Carbon is stored in living organisms (including in the ocean). Unless, I’m getting my units wrong, we’ve already emitted 500 billion tonnes of C and will reach 1000 billion tonnes by mid-century at this rate. So, if you want an increase in biomass to significantly reduce the impact of our emissions, you’re arguing for something like a doubling of all biomass on the planet. As far as the oceans itself are concerned, yes there is much more carbon in the oceans. However, the balance between the concentration in the oceans and atmosphere is set by Henry’s Law and it’s not hard to show that if we increase the total amount of carbon (by burning carbon that has been stored as fossils), the amount in the oceans and atmosphere goes up.

      • Actually, according to your link, those researchers reduced the esitmate of carbon storage in living organisms by about a third, to around 700 billion tonnes. (Given that they did this by reducing in one fell swoop the subseafloor microbe estimate by ~99%, I’m a little uneasy at accepting this as in any way a final authority. YMMV)

        This would tend to strenghten your point, except that I think you did, in fact, get your units wrong. The storage is expressed in C, but the 500gt is, according to IPCC AR5, of CO2. I believe that the carbon component of CO2 is less than 30% by weight, which would give about 150gt of Carbon, doubling by mid century to 300gt.

        Still worrisome, but not as immediately apocalyptic as the impression you’ve given.

      • kch,
        According to this site our cumulative emissions are 580 GtC, which would make it around 2000 GtCO2.

      • Just as with clouds, there are enough uncertainties in the carbon cycle to forge a whole new path to the truth.
        ===========================

      • kim,
        Argument from incredulity?

      • Not bad, Ken.

        There are many possible possible correct paths to the truth through uncertain comical clouds on the left, and the humourless carbon jokers on the right. I am incredulous that the catastrophic consensus version is correct.

        More, I’m fairly certain that in the long run anthropogenic release of fossilized carbon will be recognized as the great benefit to the earth, the biome, and human society that it has already, manifestly, been.

        The Science of Boon.
        ================

      • I am incredulous that the catastrophic consensus version is correct.

        I suspect that if someone were to study who uses the term “catastrophic” most (“skeptics” or “warmist”) it wouldn’t be warmists.

        Can we at least agree that by the mid-2100s humans could have emitted as much CO2 as is currently stored in all living organisms? Also, could we agree that if so, it is highly unlikely that the biosphere could grow sufficiently so as to rapidly reduce atmospheric concentrations? Or is that all too much for you?

      • aTTP-

        Sorry about that. I got my info from this: https://www.ipcc.ch/report/ar5/wg1/docs/WGI_AR5_2013_Poster.pdf

        and now that I look closely I see that the graph is titled “Cumulative total anthropogenic CO2 emissions from 1870 (GtC)”. Apparently I missed the (GtC) part. That’ll teach me to try to look things up between work calls… :)

        Does leave me some things to ponder, though, so I suppose it’s all good.

      • Variability:

        http://web.atmos.ucla.edu/~gruber/publication/images_publ/pt_02/fossil_atmco2_rate_stairs2.gif

        Astounding uncertainty so it’s not happening and oh gawd we just can’t, we just can’t. It’s like my father’s Admiral. He had these kid sailors on his ship with this new-fangled thing called radar. Lol, they were trying to tell him it could see the enemy in the dark. Unbelievable. He wanted to hear from scouts and lookouts – human eyeballs. He was uncertain. He damn near got everybody killed. Said to be the 3rd worst defeat of the United States Navy in WW2. The Navy solved this dunce by granting authority to shoot to the destroyers, and shoot they did.

        http://www.jpl.nasa.gov/images/blog/20141208/co2_graph-16.jpg

      • Ken, you misconstrue, as usual. Even given most of what you say I don’t think anthroCO2 is going to be anything but a net benefit. I think our feasible aliquot of CO2 will only be net boon, and I’m pretty sure it isn’t enough to much impact the next glaciation.

        I think, also, that you may be coming to realize that it is the catastrophic projection in which most skeptics have common ground, whether they be from scientific, economic, social, political or manifold other perspectives.

        It is amusing to find a consensus enforcer shying away from ‘Catastrophe’ with a capital C. I would too.
        ==================

      • Even given most of what you say I don’t think anthroCO2 is going to be anything but a net benefit.

        Fine, but how does that stop you from answering my questions? I think there are reasonably obvious answers, none of which would require you to say something that is inconsistent with your view that the net benefit would be positive (I disagree, but that’s beside the point).

      • Bingo, Ken, I can see why you teach, and expose your face. Increased respect.

        You see there is a lot of agreement. No catastrophe. You: net harm, me: net benefit. Given proposed and projected economic, social, and political changes to combat warming, which have damaged and will increasingly damage, don’t you think it prudent to understand the science better before we hang so much plausibly catastrophic policy change on such uncertain knowledge?
        ==================

      • kim,

        don’t you think it prudent to understand the science better before we hang so much plausibly catastrophic policy change on such uncertain knowledge?

        Here’s my problem with that basic idea. As it stands, the science has uncertainties but is indicating that there are risks associated with climate change and that these will probably depend on the emission pathway we choose to follow. The more we emit, the more likely we will experience changes that have severe and negative impacts. Not certain, maybe, but becoming more and more certain the higher the emissions. Additionally, the scientific evidence suggests that these changes will be essentially irreversible on human timescales. If we get to the point where the impacts are obvious, there will be little we can do to reverse the situation.

        Okay, so let’s say we decide to just wait a while. Well, that would then essentially be hoping for one of maybe 4 outcomes.
        1. there are risks but we’re just lucky and they don’t actually materialise.
        2. the science is wrong and even a high emission pathway doesn’t produce damaging changes.
        3. It’s not irreversible.
        4. We somehow develop alternatives that we can quickly introduce when we need to.

        So, we’re essentially hoping that our current understanding is very wrong or, we – without really trying – develop some kind of alternative that can quickly replace fossil fuels when needed. Seems like a risky strategy to me.

      • JCH, your chart only goes through 2000.

        Go here,

        http://csas.ei.columbia.edu/2014/12/23/assuring-real-progress-on-climate/

        and look at more complete data, and you can see that CO2 uptake continues to grow:

        http://csas.ei.columbia.edu/files/2014/12/Fig.-2.jpg

        This makes sense. The larger the atmospheric concentration, the larger the gradient both with the oceans and the biosphere. Consequently the larger the removal rates.

        Most of it is probably going where the CFCs are going – the deep ocean – since the polar regions is where uptake is most enabled and also where deep water gets transported downward.

      • Eddie,
        Here’s a paper suggesting that the fraction going into the biosphere/ocean has been reducing. I think I saw another paper (that I can’t find) that suggested it hadn’t changed. I haven’t found any evidence that the uptake fraction has risen. The total amount has, but the fraction has remained at around 56%.

      • It varies with ENSO.

      • Yes, Ken, I think the current understanding is likely very wrong, particularly the more extremely harmful scenarios. We do have a ready alternative. Atomkraft, ja bitte.

        JCH, tell me more about the oceanic oscillation phase dependence of that uptake.
        ========

      • In 2011 the earth greened a whole bunch. You laughed at water. Now water is laughing at you:

        http://sealevel.colorado.edu/files/2015_rel1/sl_ns_global.pdf

      • Sea level trend rate uncertainties guffaw as comically as the gods in the clouds, and from almost as many humourous anecdotes.
        =================

      • There simply is nowhere else for the vast amount of rain that fell on the land area in 2011 to have come from other than the oceans, and water makes plants grow. Farmers figured this out centuries ago.

        They also figured out things like chopping down trees so sunlight could reach their plants, or not making gardens in the shade of a cliff.

        But you can catch up.

      • Kim,
        So, you’re going for 2. You get my point, though? Delaying implies that we are hoping that we’re substantially wrong about something. We might be and, given what we’re currently doing, I hope we are.

    • I am not ignoring it. They were talking about CO2 enrichment – plants. The plants in the ocean do not directly use the atmosphere to get their CO2.The implied claim was an increased uptake of CO2 by land was unexpected by scientists, not true, and that it’s due to Brawndo CO2 enrichment, and then sunlight and water were belittled, which sounds amazingly stupid to the kid in me who grew up in a dryland farming economy in the Dakotas. CO2 enrichment is part of it, but plants have limiting factors, and studies are showing they’re hitting a ceiling.

      You are showing off. Everybody knows the oceans are full of carbon, and getting fuller.

      Dyson picked trees because trees make wood. Wood is a very good way to remove carbon from the atmosphere and to store it. Iowa corn plants, not so much.

      Did Dyson ignore the oceans and the corn?. Write him a letter.

      • No, JCH, I did not belittle sunlight and water. I ridiculed the scientists trying to paste an apologetic response over the obvious greening of the Earth from anthroCO2. Note archaic use of apologetic.

        But you may have something there on ‘reaching its limits’. Tell me more.
        ==================

      • er, ‘hitting a ceiling’. I don’t read very well, anymore.
        =============

      • Heh, a nursery greenhouse ceiling? Would that be glass, or plastic, or some sort of self healing substance, you know, a high tech one, that might modify the amount of energy in as well as the amount of energy out.

        Let’s get AK on imagining one, Peter Lang as stormy anchor.
        =================

      • The plants in the ocean do not directly use the atmosphere to get their CO2.

        It’s only as far away as a google.

  66. However, the balance between the concentration in the oceans and atmosphere is set by Henry’s Law and it’s not hard to show that if we increase the total amount of carbon (by burning carbon that has been stored as fossils), the amount in the oceans and atmosphere goes up.

    I believe the oceans are increasing in CO2, largely in the deep oceans where CO2 concentrations are naturally higher ( dissolved inorganic carbon ):

    http://www.pnas.org/content/106/30/12235/F2.large.jpg

  67. Great first google TE
    Oceans and the Carbon Cycle
    “The oceans influence the climate by absorbing and storing carbon dioxide. Climate change is caused by the accumulation of man-made carbon dioxide (CO2) and other greenhouse gases in the atmosphere. The rate of accumulation depends on how much CO2 mankind emits and how much of this excess CO2 is absorbed by plants and soil or is transported down into the ocean depths by plankton (microscopic plants and animals). Scientists believe that the oceans currently absorb 30-50% of the CO2 produced by the burning of fossil fuel. If they did not soak up any CO2, atmospheric CO2 levels would be much higher than the current level of 355 parts per million by volume (ppmv) – probably around 500-600 ppmv.”

    ATTP of interest apart from the 30-50% CO2 absorption which is mainly by plankton
    the punchline is
    “But the real danger, of course, is that manipulating biological systems that are not thoroughly understood could have negative consequences just as easily as positive ones.”

    This is the problem you are not facing up to.

    Your request for intemperate actions on climate change may be as harmful as you are postulating no action to be as the Climate system is still not thoroughly understood.

    • angech,

      Your request for intemperate actions on climate change may be as harmful as you are postulating no action to be as the Climate system is still not thoroughly understood.

      Could you please point out where I’ve requested intemperate action on climage change? It appears as though you’ve done what many seem to do which is to assume that since I broadly accept (and – I would argue – understand) the mainstream scientific position with respect to climate change, that I must be also advocating for all sorts of things that you would regard as intemperate. Maybe you could point out precisely where I’ve done so and what these intemperate things are.

      • ATTP, intemperance
        the problem is one of perception.
        Your mainstream position , as you address matters on your site, is that action needs to be taken sooner rather than later to reduce the risk of AGW causing harm to future generations.
        My perception is that we do not have enough proof to demand immediate action [which I would call intemperate] yet.
        From your point of view I understand there is no intemperance.
        I am more than happy for you to put your arguments up and discuss them
        and hope I only question points that have not been stated clearly [Lacis?] or I feel are wrong [JCH].
        You have put your views on CO2 levels in the atmosphere in a way that shows you are prepared to think about it even though we disagree on the persistence of CO2

  68. JCH | April 14, 2015 at 8:54 am | Reply

    “I am not ignoring it. They were talking about CO2 enrichment – plants. The plants in the ocean do not directly use the atmosphere to get their CO2″.

    Using the word ” directly ” is really cribbing the issue I feel.
    The CO2 is in the atmosphere and then dissolves in the sea.
    We are talking about sea life utilizing CO2 which comes from the CO2 in the air.
    Your explanation is wrong and misleading.
    Your argument has an eerie resemblance to the line
    “ I experimented with marijuana a time or two, and didn’t like it,” “I didn’t inhale, and I didn’t try it again.”